SlideShare a Scribd company logo
1 of 1056
Download to read offline
PRACTICAL APPLICATIONS
Each chapter devotes material to practical applications of the concepts covered in Fundamentals of Electric
Circuits to help the reader apply the concepts to real-life situations. Here is a sampling of the practical applications
found in the text:
• Rechargeable flashlight battery (Problem 1.11)
• Cost of operating toaster (Problem 1.25)
• Potentiometer (Section 2.8)
• Design a lighting system (Problem 2.61)
• Reading a voltmeter (Problem 2.66)
• Controlling speed of a motor (Problem 2.74)
• Electric pencil sharpener (Problem 2.78)
• Calculate voltage of transistor (Problem 3.86)
• Transducer modeling (Problem 4.87)
• Strain gauge (Problem 4.90)
• Wheatstone bridge (Problem 4.91)
• Design a six-bit DAC (Problem 5.83)
• Instrumentation amplifier (Problem 5.88)
• Design an analog computer circuit (Example 6.15)
• Design an op amp circuit (Problem 6.71)
• Design analog computer to solve differential equation (Problem 6.79)
• Electric power plant substation—capacitor bank (Problem 6.83)
• Electronic photo flash unit (Section 7.9)
• Automobile ignition circuit (Section 7.9)
• Welding machine (Problem 7.86)
• Airbag igniter (Problem 8.78)
• Electrical analog to bodily functions—study of convulsions (Problem 8.82)
• Electronic sensing device (Problem 9.87)
• Power transmission system (Problem 9.93)
• Design a Colpitts oscillator (Problem 10.94)
• Stereo amplifier circuit (Problem 13.85)
• Gyrator circuit (Problem 16.69)
• Calculate number of stations allowable in AM broadcast band (Problem 18.63)
• Voice signal—Nyquist rate (Problem 18.65)
ale29559_IFC.qxd 07/11/2008 07:40 PM Page 2
COMPUTER TOOLS promote flexibility and meet ABET requirements
• PSpice is introduced in Chapter 3 and appears in special sections throughout the text. Appendix D serves
as a tutorial on PSpice for Windows for readers not familiar with its use. The special sections contain exam-
ples and practice problems using PSpice. Additional homework problems at the end of each chapter also
provide an opportunity to use PSpice.
• MATLAB®
is introduced through a tutorial in Appendix E to show its usage in circuit analysis. A number
of examples and practice problems are presented throughout the book in a manner that will allow the student
to develop a facility with this powerful tool. A number of end-of-chapter problems will aid in understanding
how to effectively use MATLAB.
• KCIDE for Circuits is a working software environment developed at Cleveland State University. It is
designed to help the student work through circuit problems in an organized manner following the process
on problem-solving discussed in Section 1.8. Appendix F contains a description of how to use the software.
Additional examples can be found at the web site, http://kcide.fennresearch.org/. The actual software pack-
age can be downloaded for free from this site. One of the best benefits from using this package is that it
automatically generates a Word document and/or a PowerPoint presentation.
CAREERS AND HISTORY of electrical engineering pioneers
Since a course in circuit analysis may be a student’s first exposure to electrical engineering, each chapter opens
with discussions about how to enhance skills that contribute to successful problem-solving or career-oriented
talks on a sub-discipline of electrical engineering. The chapter openers are intended to help students grasp
the scope of electrical engineering and give thought to the various careers available to EE graduates. The open-
ing boxes include information on careers in electronics, instrumentation, electromagnetics, control systems,
engineering education, and the importance of good communication skills. Historicals throughout the text
provide brief biological sketches of such engineering pioneers as Faraday, Ampere, Edison, Henry, Fourier,
Volta, and Bell.
ale29559_IFC.qxd 07/11/2008 07:40 PM Page 3
OUR COMMITMENT TO ACCURACY
You have a right to expect an accurate textbook, and McGraw-Hill Engineering invests
considerable time and effort to ensure that we deliver one. Listed below are the many
steps we take in this process.
OUR ACCURACY VERIFICATION PROCESS
First Round
Step 1: Numerous college engineering instructors review the manuscript and report
errors to the editorial team. The authors review their comments and make the necessary
corrections in their manuscript.
Second Round
Step 2: An expert in the field works through every example and exercise in the final
manuscript to verify the accuracy of the examples, exercises, and solutions. The authors
review any resulting corrections and incorporate them into the final manuscript and solu-
tions manual.
Step 3: The manuscript goes to a copyeditor, who reviews the pages for grammatical and
stylistic considerations. At the same time, the expert in the field begins a second accuracy
check. All corrections are submitted simultaneously to the authors, who review and inte-
grate the editing, and then submit the manuscript pages for typesetting.
Third Round
Step 4: The authors review their page proofs for a dual purpose: 1) to make certain that
any previous corrections were properly made, and 2) to look for any errors they might
have missed.
Step 5: A proofreader is assigned to the project to examine the new page proofs, double
check the authors' work, and add a fresh, critical eye to the book. Revisions are incorpo-
rated into a new batch of pages which the authors check again.
Fourth Round
Step 6: The author team submits the solutions manual to the expert in the field, who
checks text pages against the solutions manual as a final review.
Step 7: The project manager, editorial team, and author team review the pages for a
final accuracy check.
The resulting engineering textbook has gone through several layers of quality assurance
and is verified to be as accurate and error-free as possible. Our authors and publishing
staff are confident that through this process we deliver textbooks that are industry leaders
in their correctness and technical integrity.
ale29559_IFC.qxd 07/11/2008 07:40 PM Page 4
Fundamentals of
Electric Circuits
ale29559_fm.qxd 07/28/2008 11:54 AM Page i
ale29559_fm.qxd 07/28/2008 11:54 AM Page ii
f o u r t h e d i t i o n
Fundamentals of
Electric Circuits
Charles K. Alexander
Department of Electrical and
Computer Engineering
Cleveland State University
Matthew N. O. Sadiku
Department of
Electrical Engineering
Prairie View A&M University
ale29559_fm.qxd 07/28/2008 11:54 AM Page iii
FUNDAMENTALS OF ELECTRIC CIRCUITS, FOURTH EDITION
Published by McGraw-Hill, a business unit of The McGraw-Hill Companies, Inc., 1221 Avenue of
the Americas, New York, NY 10020. Copyright © 2009 by The McGraw-Hill Companies, Inc.
All rights reserved. Previous editions © 2007, 2004, and 2000. No part of this publication may be
reproduced or distributed in any form or by any means, or stored in a database or retrieval system,
without the prior written consent of The McGraw-Hill Companies, Inc., including, but not limited to,
in any network or other electronic storage or transmission, or broadcast for distance learning.
Some ancillaries, including electronic and print components, may not be available to customers
outside the United States.
This book is printed on acid-free paper.
1 2 3 4 5 6 7 8 9 0 VNH/VNH 0 9 8
ISBN 978–0–07–352955–4
MHID 0–07–352955–9
Global Publisher: Raghothaman Srinivasan
Director of Development: Kristine Tibbetts
Developmental Editor: Lora Neyens
Senior Marketing Manager: Curt Reynolds
Project Manager: Joyce Watters
Senior Production Supervisor: Sherry L. Kane
Lead Media Project Manager: Stacy A. Patch
Associate Design Coordinator: Brenda A. Rolwes
Cover Designer: Studio Montage, St. Louis, Missouri
(USE) Cover Image: Astronauts Repairing Spacecraft: © StockTrek/Getty Images;
Printed Circuit Board: Photodisc Collection/Getty Images
Lead Photo Research Coordinator: Carrie K. Burger
Compositor: ICC Macmillan Inc.
Typeface: 10/12 Times Roman
Printer: R. R. Donnelley, Jefferson City, MO
Library of Congress Cataloging-in-Publication Data
Alexander, Charles K.
Fundamentals of electric circuits / Charles K. Alexander, Matthew N. O. Sadiku. — 4th ed.
p. cm.
Includes index.
ISBN 978–0–07–352955–4 — ISBN 0–07–352955–9 (hard copy : alk. paper) 1. Electric circuits.
I. Sadiku, Matthew N. O. II. Title.
TK454.A452 2009
621.319'24—dc22 2008023020
www.mhhe.com
ale29559_fm.qxd 07/28/2008 11:54 AM Page iv
Dedicated to our wives, Kikelomo and Hannah, whose understanding and
support have truly made this book possible.
Matthew
and
Chuck
ale29559_fm.qxd 07/28/2008 11:54 AM Page v
ale29559_fm.qxd 07/28/2008 11:54 AM Page vi
Contents
Preface xiii
Acknowledgments xviii
Guided Tour xx
A Note to the Student xxv
About the Authors xxvii
PART 1 DC Circuits 2
Chapter 1 Basic Concepts 3
1.1 Introduction 4
1.2 Systems of Units 4
1.3 Charge and Current 6
1.4 Voltage 9
1.5 Power and Energy 10
1.6 Circuit Elements 15
1.7 †
Applications 17
1.7.1 TV Picture Tube
1.7.2 Electricity Bills
1.8 †
Problem Solving 20
1.9 Summary 23
Review Questions 24
Problems 24
Comprehensive Problems 27
Chapter 2 Basic Laws 29
2.1 Introduction 30
2.2 Ohm’s Law 30
2.3 †
Nodes, Branches, and Loops 35
2.4 Kirchhoff’s Laws 37
2.5 Series Resistors and Voltage
Division 43
2.6 Parallel Resistors and Current
Division 45
2.7 †
Wye-Delta Transformations 52
2.8 †
Applications 58
2.8.1 Lighting Systems
2.8.2 Design of DC Meters
2.9 Summary 64
Review Questions 66
Problems 67
Comprehensive Problems 78
vii
Chapter 3 Methods of Analysis 81
3.1 Introduction 82
3.2 Nodal Analysis 82
3.3 Nodal Analysis with Voltage
Sources 88
3.4 Mesh Analysis 93
3.5 Mesh Analysis with Current
Sources 98
3.6 †
Nodal and Mesh Analyses
by Inspection 100
3.7 Nodal Versus Mesh Analysis 104
3.8 Circuit Analysis with PSpice 105
3.9 †
Applications: DC Transistor Circuits 107
3.10 Summary 112
Review Questions 113
Problems 114
Comprehensive Problem 126
Chapter 4 Circuit Theorems 127
4.1 Introduction 128
4.2 Linearity Property 128
4.3 Superposition 130
4.4 Source Transformation 135
4.5 Thevenin’s Theorem 139
4.6 Norton’s Theorem 145
4.7 †
Derivations of Thevenin’s and Norton’s
Theorems 149
4.8 Maximum Power Transfer 150
4.9 Verifying Circuit Theorems with
PSpice 152
4.10 †
Applications 155
4.10.1 Source Modeling
4.10.2 Resistance Measurement
4.11 Summary 160
Review Questions 161
Problems 162
Comprehensive Problems 173
Chapter 5 Operational Amplifiers 175
5.1 Introduction 176
5.2 Operational Amplifiers 176
ale29559_fm.qxd 07/28/2008 11:54 AM Page vii
viii Contents
5.3 Ideal Op Amp 179
5.4 Inverting Amplifier 181
5.5 Noninverting Amplifier 183
5.6 Summing Amplifier 185
5.7 Difference Amplifier 187
5.8 Cascaded Op Amp Circuits 191
5.9 Op Amp Circuit Analysis with PSpice 194
5.10 †
Applications 196
5.10.1 Digital-to-Analog Converter
5.10.2 Instrumentation Amplifiers
5.11 Summary 199
Review Questions 201
Problems 202
Comprehensive Problems 213
Chapter 6 Capacitors and
Inductors 215
6.1 Introduction 216
6.2 Capacitors 216
6.3 Series and Parallel Capacitors 222
6.4 Inductors 226
6.5 Series and Parallel Inductors 230
6.6 †
Applications 233
6.6.1 Integrator
6.6.2 Differentiator
6.6.3 Analog Computer
6.7 Summary 240
Review Questions 241
Problems 242
Comprehensive Problems 251
Chapter 7 First-Order Circuits 253
7.1 Introduction 254
7.2 The Source-Free RC Circuit 254
7.3 The Source-Free RL Circuit 259
7.4 Singularity Functions 265
7.5 Step Response of an RC Circuit 273
7.6 Step Response of an RL Circuit 280
7.7 †
First-Order Op Amp Circuits 284
7.8 Transient Analysis with PSpice 289
7.9 †
Applications 293
7.9.1 Delay Circuits
7.9.2 Photoflash Unit
7.9.3 Relay Circuits
7.9.4 Automobile Ignition Circuit
7.10 Summary 299
Review Questions 300
Problems 301
Comprehensive Problems 311
Chapter 8 Second-Order Circuits 313
8.1 Introduction 314
8.2 Finding Initial and Final Values 314
8.3 The Source-Free Series
RLC Circuit 319
8.4 The Source-Free Parallel RLC
Circuit 326
8.5 Step Response of a Series RLC
Circuit 331
8.6 Step Response of a Parallel RLC
Circuit 336
8.7 General Second-Order Circuits 339
8.8 Second-Order Op Amp Circuits 344
8.9 PSpice Analysis of RLC Circuits 346
8.10 †
Duality 350
8.11 †
Applications 353
8.11.1 Automobile Ignition System
8.11.2 Smoothing Circuits
8.12 Summary 356
Review Questions 357
Problems 358
Comprehensive Problems 367
PART 2 AC Circuits 368
Chapter 9 Sinusoids and Phasors 369
9.1 Introduction 370
9.2 Sinusoids 371
9.3 Phasors 376
9.4 Phasor Relationships for
Circuit Elements 385
9.5 Impedance and Admittance 387
9.6 †
Kirchhoff’s Laws in the Frequency
Domain 389
9.7 Impedance Combinations 390
9.8 †
Applications 396
9.8.1 Phase-Shifters
9.8.2 AC Bridges
9.9 Summary 402
Review Questions 403
Problems 403
Comprehensive Problems 411
Chapter 10 Sinusoidal Steady-State
Analysis 413
10.1 Introduction 414
10.2 Nodal Analysis 414
10.3 Mesh Analysis 417
ale29559_fm.qxd 07/28/2008 11:54 AM Page viii
Contents ix
12.11 Summary 543
Review Questions 543
Problems 544
Comprehensive Problems 553
Chapter 13 Magnetically Coupled
Circuits 555
13.1 Introduction 556
13.2 Mutual Inductance 557
13.3 Energy in a Coupled Circuit 564
13.4 Linear Transformers 567
13.5 Ideal Transformers 573
13.6 Ideal Autotransformers 581
13.7 †
Three-Phase Transformers 584
13.8 PSpice Analysis of Magnetically
Coupled Circuits 586
13.9 †
Applications 591
13.9.1 Transformer as an Isolation Device
13.9.2 Transformer as a Matching Device
13.9.3 Power Distribution
13.10 Summary 597
Review Questions 598
Problems 599
Comprehensive Problems 611
Chapter 14 Frequency Response 613
14.1 Introduction 614
14.2 Transfer Function 614
14.3 †
The Decibel Scale 617
14.4 Bode Plots 619
14.5 Series Resonance 629
14.6 Parallel Resonance 634
14.7 Passive Filters 637
14.7.1 Lowpass Filter
14.7.2 Highpass Filter
14.7.3 Bandpass Filter
14.7.4 Bandstop Filter
14.8 Active Filters 642
14.8.1 First-Order Lowpass Filter
14.8.2 First-Order Highpass Filter
14.8.3 Bandpass Filter
14.8.4 Bandreject (or Notch) Filter
14.9 Scaling 648
14.9.1 Magnitude Scaling
14.9.2 Frequency Scaling
14.9.3 Magnitude and Frequency Scaling
14.10 Frequency Response Using
PSpice 652
14.11 Computation Using MATLAB 655
10.4 Superposition Theorem 421
10.5 Source Transformation 424
10.6 Thevenin and Norton Equivalent
Circuits 426
10.7 Op Amp AC Circuits 431
10.8 AC Analysis Using PSpice 433
10.9 †
Applications 437
10.9.1 Capacitance Multiplier
10.9.2 Oscillators
10.10 Summary 441
Review Questions 441
Problems 443
Chapter 11 AC Power Analysis 457
11.1 Introduction 458
11.2 Instantaneous and Average
Power 458
11.3 Maximum Average Power
Transfer 464
11.4 Effective or RMS Value 467
11.5 Apparent Power and
Power Factor 470
11.6 Complex Power 473
11.7 †
Conservation of AC Power 477
11.8 Power Factor Correction 481
11.9 †
Applications 483
11.9.1 Power Measurement
11.9.2 Electricity Consumption Cost
11.10 Summary 488
Review Questions 490
Problems 490
Comprehensive Problems 500
Chapter 12 Three-Phase Circuits 503
12.1 Introduction 504
12.2 Balanced Three-Phase Voltages 505
12.3 Balanced Wye-Wye Connection 509
12.4 Balanced Wye-Delta Connection 512
12.5 Balanced Delta-Delta
Connection 514
12.6 Balanced Delta-Wye Connection 516
12.7 Power in a Balanced System 519
12.8 †
Unbalanced Three-Phase
Systems 525
12.9 PSpice for Three-Phase Circuits 529
12.10 †
Applications 534
12.10.1 Three-Phase Power Measurement
12.10.2 Residential Wiring
ale29559_fm.qxd 07/28/2008 11:54 AM Page ix
x Contents
14.12 †
Applications 657
14.12.1 Radio Receiver
14.12.2 Touch-Tone Telephone
14.12.3 Crossover Network
14.13 Summary 663
Review Questions 664
Problems 665
Comprehensive Problems 673
PART 3 Advanced Circuit
Analysis 674
Chapter 15 Introduction to the Laplace
Transform 675
15.1 Introduction 676
15.2 Definition of the Laplace Transform 677
15.3 Properties of the Laplace Transform 679
15.4 The Inverse Laplace Transform 690
15.4.1 Simple Poles
15.4.2 Repeated Poles
15.4.3 Complex Poles
15.5 The Convolution Integral 697
15.6 †
Application to Integrodifferential
Equations 705
15.7 Summary 708
Review Questions 708
Problems 709
Chapter 16 Applications of the Laplace
Transform 715
16.1 Introduction 716
16.2 Circuit Element Models 716
16.3 Circuit Analysis 722
16.4 Transfer Functions 726
16.5 State Variables 730
16.6 †
Applications 737
16.6.1 Network Stability
16.6.2 Network Synthesis
16.7 Summary 745
Review Questions 746
Problems 747
Comprehensive Problems 754
Chapter 17 The Fourier Series 755
17.1 Introduction 756
17.2 Trigonometric Fourier Series 756
17.3 Symmetry Considerations 764
17.3.1 Even Symmetry
17.3.2 Odd Symmetry
17.3.3 Half-Wave Symmetry
17.4 Circuit Applications 774
17.5 Average Power and RMS Values 778
17.6 Exponential Fourier Series 781
17.7 Fourier Analysis with PSpice 787
17.7.1 Discrete Fourier Transform
17.7.2 Fast Fourier Transform
17.8 †
Applications 793
17.8.1 Spectrum Analyzers
17.8.2 Filters
17.9 Summary 796
Review Questions 798
Problems 798
Comprehensive Problems 807
Chapter 18 Fourier Transform 809
18.1 Introduction 810
18.2 Definition of the Fourier Transform 810
18.3 Properties of the Fourier
Transform 816
18.4 Circuit Applications 829
18.5 Parseval’s Theorem 832
18.6 Comparing the Fourier and Laplace
Transforms 835
18.7 †
Applications 836
18.7.1 Amplitude Modulation
18.7.2 Sampling
18.8 Summary 839
Review Questions 840
Problems 841
Comprehensive Problems 847
Chapter 19 Two-Port Networks 849
19.1 Introduction 850
19.2 Impedance Parameters 850
19.3 Admittance Parameters 855
19.4 Hybrid Parameters 858
19.5 Transmission Parameters 863
19.6 †
Relationships Between
Parameters 868
19.7 Interconnection of Networks 871
19.8 Computing Two-Port Parameters
Using PSpice 877
19.9 †
Applications 880
19.9.1 Transistor Circuits
19.9.2 Ladder Network Synthesis
ale29559_fm.qxd 07/28/2008 11:54 AM Page x
Contents xi
19.10 Summary 889
Review Questions 890
Problems 890
Comprehensive Problems 901
Appendix A Simultaneous Equations and Matrix
Inversion A
Appendix B Complex Numbers A-9
Appendix C Mathematical Formulas A-16
Appendix D PSpice for Windows A-21
Appendix E MATLAB A-46
Appendix F KCIDE for Circuits A-65
Appendix G Answers to Odd-Numbered
Problems A-75
Selected Bibliography B-1
Index I-1
ale29559_fm.qxd 07/28/2008 11:54 AM Page xi
ale29559_fm.qxd 07/28/2008 11:54 AM Page xii
xiii
Preface
You may be wondering why we chose a photo of astronauts working
in space on the Space Station for the cover. We actually chose it for
several reasons. Obviously, it is very exciting; in fact, space represents
the most exciting frontier for the entire world! In addition, much of the
station itself consists of all kinds of circuits! One of the most signifi-
cant circuits within the station is its power distribution system. It is a
complete and self contained, modern power generation and distribution
system. That is why NASA (especially NASA-Glenn) continues to be
at the forefront of both theoretical as well as applied power system
research and development. The technology that has gone into the devel-
opment of space exploration continues to find itself impacting terres-
trial technology in many important ways. For some of you, this will be
an important career path.
FEATURES
New to This Edition
A course in circuit analysis is perhaps the first exposure students have
to electrical engineering. This is also a place where we can enhance
some of the skills that they will later need as they learn how to design.
In the fourth edition, we have included a very significant new
feature to help students enhance skills that are an important part of the
design process. We call this new feature, design a problem.
We know it is not possible to fully develop a student’s design skills
in a fundamental course like circuits. To fully develop design skills a
student needs a design experience normally reserved for their senior
year. This does not mean that some of those skills cannot be developed
and exercised in a circuits course. The text already included open-
ended questions that help students use creativity, which is an impor-
tant part of learning how to design. We already have some questions
that are open desired to add much more into our text in this important
area and have developed an approach to do just that. When we develop
problems for the student to solve our goal is that in solving the prob-
lem the student learn more about the theory and the problem solving
process. Why not have the students design problems like we do? That
is exactly what we will do in each chapter. Within the normal problem
set, we have a set of problems where we ask the student to design a
problem. This will have two very important results. The first will be a
better understanding of the basic theory and the second will be the
enhancement of some of the student’s basic design skills.
We are making effective use of the principle of learning by teach-
ing. Essentially we all learn better when we teach a subject. Design-
ing effective problems is a key part of the teaching process. Students
ale29559_fm.qxd 08/08/2008 10:16 AM Page xiii
xiv Preface
should also be encouraged to develop problems, when appropriate,
which have nice numbers and do not necessarily overemphasize com-
plicated mathematical manipulations.
Additionally we have changed almost 40% of the Practice Prob-
lems with the idea to better reflect more real component values and to
help the student better understand the problem and have added 121
design a problem problems. We have also changed and added a total
of 357 end-of-chapter problems (this number contains the new design
a problem problems). This brings up a very important advantage to our
textbook, we have a total of 2404 Examples, Practice Problems,
Review Questions, and end-of-chapter problems!
Retained from Previous Editions
The main objective of the fourth edition of this book remains the same
as the previous editions—to present circuit analysis in a manner that is
clearer, more interesting, and easier to understand than other circuit text,
and to assist the student in beginning to see the “fun” in engineering.
This objective is achieved in the following ways:
• Chapter Openers and Summaries
Each chapter opens with a discussion about how to enhance skills
which contribute to successful problem solving as well as suc-
cessful careers or a career-oriented talk on a sub-discipline of elec-
trical engineering. This is followed by an introduction that links the
chapter with the previous chapters and states the chapter objectives.
The chapter ends with a summary of key points and formulas.
• Problem Solving Methodology
Chapter 1 introduces a six-step method for solving circuit problems
which is used consistently throughout the book and media supple-
ments to promote best-practice problem-solving procedures.
• Student Friendly Writing Style
All principles are presented in a lucid, logical, step-by-step manner.
As much as possible, we avoid wordiness and giving too much
detail that could hide concepts and impede overall understanding of
the material.
• Boxed Formulas and Key Terms
Important formulas are boxed as a means of helping students sort
out what is essential from what is not. Also, to ensure that students
clearly understand the key elements of the subject matter, key
terms are defined and highlighted.
• Margin Notes
Marginal notes are used as a pedagogical aid. They serve multiple
uses such as hints, cross-references, more exposition, warnings,
reminders not to make some particular common mistakes, and
problem-solving insights.
• Worked Examples
Thoroughly worked examples are liberally given at the end of every
section. The examples are regarded as a part of the text and are
clearly explained without asking the reader to fill in missing steps.
Thoroughly worked examples give students a good understand-
ing of the solution process and the confidence to solve problems
ale29559_fm.qxd 07/28/2008 11:54 AM Page xiv
Preface xv
themselves. Some of the problems are solved in two or three dif-
ferent ways to facilitate a substantial comprehension of the subject
material as well as a comparison of different approaches.
• Practice Problems
To give students practice opportunity, each illustrative example is
immediately followed by a practice problem with the answer. The
student can follow the example step by step to aid in the solution
of the practice problem without flipping pages or looking at the
end of the book for answers. The practice problem is also intended
to test a student’s understanding of the preceding example. It will
reinforce their grasp of the material before the student can move
on to the next section. Complete solutions to the practice problems
are available to students on ARIS.
• Application Sections
The last section in each chapter is devoted to practical application
aspects of the concepts covered in the chapter. The material cov-
ered in the chapter is applied to at least one or two practical prob-
lems or devices. This helps students see how the concepts are
applied to real-life situations.
• Review Questions
Ten review questions in the form of multiple-choice objective items
are provided at the end of each chapter with answers. The review
questions are intended to cover the little “tricks” that the examples
and end-of-chapter problems may not cover. They serve as a self-
test device and help students determine how well they have mas-
tered the chapter.
• Computer Tools
In recognition of the requirements by ABET®
on integrating com-
puter tools, the use of PSpice, MATLAB, KCIDE for Circuits, and
developing design skills are encouraged in a student-friendly man-
ner. PSpice is covered early on in the text so that students can
become familiar and use it throughout the text. Appendix D serves
as a tutorial on PSpice for Windows. MATLAB is also introduced
early in the book with a tutorial available in Appendix E. KCIDE
for Circuits is a brand new, state-of-the-art software system designed
to help the students maximize their chance of success in problem
solving. It is introduced in Appendix F. Finally, design a problem
problems have been introduced, for the first time. These are meant
to help the student develop skills that will be needed in the design
process.
• Historical Tidbits
Historical sketches throughout the text provide profiles of important
pioneers and events relevant to the study of electrical engineering.
• Early Op Amp Discussion
The operational amplifier (op amp) as a basic element is introduced
early in the text.
• Fourier and Laplace Transforms Coverage
To ease the transition between the circuit course and signals and
systems courses, Fourier and Laplace transforms are covered
lucidly and thoroughly. The chapters are developed in a manner
that the interested instructor can go from solutions of first-order
ale29559_fm.qxd 07/28/2008 11:54 AM Page xv
circuits to Chapter 15. This then allows a very natural progression
from Laplace to Fourier to AC.
• Four Color Art Program
A completely redesigned interior design and four color art program
bring circuit drawings to life and enhance key pedagogical ele-
ments throughout the text.
• Extended Examples
Examples worked in detail according to the six-step problem solv-
ing method provide a roadmap for students to solve problems in a
consistent fashion. At least one example in each chapter is devel-
oped in this manner.
• EC 2000 Chapter Openers
Based on ABET’s new skill-based CRITERION 3, these chapter
openers are devoted to discussions as to how students can acquire
the skills that will lead to a significantly enhanced career as an
engineer. Because these skills are so very important to the student
while in college as well as in their career, we will use the head-
ing, “Enhancing your Skills and your Career.”
• Homework Problems
There are 358 new or changed end-of-chapter problems which will
provide students with plenty of practice as well as reinforce key
concepts.
• Homework Problem Icons
Icons are used to highlight problems that relate to engineering design
as well as problems that can be solved using PSpice or MATLAB.
• KCIDE for Circuits Appendix F
A new Appendix F provides a tutorial on the Knowledge Captur-
ing Integrated Design Environment (KCIDE for Circuits) software,
available on ARIS.
Organization
This book was written for a two-semester or three-quarter course in
linear circuit analysis. The book may also be used for a one-semester
course by a proper selection of chapters and sections by the instructor.
It is broadly divided into three parts.
• Part 1, consisting of Chapters 1 to 8, is devoted to dc circuits. It
covers the fundamental laws and theorems, circuits techniques, and
passive and active elements.
• Part 2, which contains Chapter 9 to 14, deals with ac circuits. It
introduces phasors, sinusoidal steady-state analysis, ac power, rms
values, three-phase systems, and frequency response.
• Part 3, consisting of Chapters 15 to 19, is devoted to advanced
techniques for network analysis. It provides students with a solid
introduction to the Laplace transform, Fourier series, Fourier trans-
form, and two-port network analysis.
The material in three parts is more than sufficient for a two-semester
course, so the instructor must select which chapters or sections to cover.
Sections marked with the dagger sign (†) may be skipped, explained
briefly, or assigned as homework. They can be omitted without loss of
xvi Preface
ale29559_fm.qxd 07/28/2008 11:54 AM Page xvi
Preface xvii
continuity. Each chapter has plenty of problems grouped according to
the sections of the related material and diverse enough that the instruc-
tor can choose some as examples and assign some as homework. As
stated earlier, we are using three icons with this edition. We are using
to denote problems that either require PSpice in the solution process,
where the circuit complexity is such that PSpice would make the solu-
tion process easier, and where PSpice makes a good check to see if the
problem has been solved correctly. We are using to denote problems
where MATLAB is required in the solution process, where MATLAB
makes sense because of the problem makeup and its complexity, and
where MATLAB makes a good check to see if the problem has been
solved correctly. Finally, we use to identify problems that help the
student develop skills that are needed for engineering design. More dif-
ficult problems are marked with an asterisk (*). Comprehensive prob-
lems follow the end-of-chapter problems. They are mostly applications
problems that require skills learned from that particular chapter.
Prerequisites
As with most introductory circuit courses, the main prerequisites, for
a course using the text, are physics and calculus. Although familiarity
with complex numbers is helpful in the later part of the book, it is not
required. A very important asset of this text is that ALL the mathe-
matical equations and fundamentals of physics needed by the student,
are included in the text.
Supplements
McGraw-Hill’s ARIS—Assessment, Review, and Instruction
System is a complete, online tutorial, electronic homework, and course
management system, designed for greater ease of use than any other
system available. Available on adoption, instructors can create and
share course materials and assignments with other instructors, edit ques-
tions and algorithms, import their own content, and create announce-
ments and due dates for assignments. ARIS has automatic grading and
reporting of easy-to-assign algorithmically-generated homework,
quizzing, and testing. Once a student is registered in the course, all stu-
dent activity within McGraw-Hill’s ARIS is automatically recorded and
available to the instructor through a fully integrated grade book that can
be downloaded to Excel. Also included on ARIS are a solutions man-
ual, text image files, transition guides to instructors, and Network
Analysis Tutorials, software downloads, complete solutions to text
practice problems, FE Exam questions, flashcards, and web links to stu-
dents. Visit www.mhhe.com/alexander.
Knowledge Capturing Integrated Design Environment for Circuits
(KCIDE for Circuits) This new software, developed at Cleveland State
University and funded by NASA, is designed to help the student work
through a circuits problem in an organized manner using the six-step
problem-solving methodology in the text. KCIDE for Circuits allows
students to work a circuit problem in PSpice and MATLAB, track the
ale29559_fm.qxd 07/28/2008 11:54 AM Page xvii
xviii Preface
evolution of their solution, and save a record of their process for future
reference. In addition, the software automatically generates a Word
document and/or a PowerPoint presentation. Appendix F contains a
description of how to use the software. Additional examples can be
found at the web site, http://kcide.fennresearch.org/, which is linked
from ARIS. The software package can be downloaded for free.
Problem Solving Made Almost Easy, a companion workbook to Fun-
damentals of Electric Circuits, is available on ARIS for students who
wish to practice their problem-solving techniques. The workbook con-
tains a discussion of problem-solving strategies and 150 additional
problems with complete solutions provided.
C.O.S.M.O.S This CD, available to instructors only, is a powerful solu-
tions manual tool to help instructors streamline the creation of assign-
ments, quizzes, and tests by using problems and solutions from the
textbook, as well as their own custom material. Instructors can edit
textbook end-of-chapter problems as well as track which problems have
been assigned.
Although the textbook is meant to be self-explanatory and act as
a tutor for the student, the personal contact in teaching is not forgot-
ten. It is hoped that the book and supplemental materials supply the
instructor with all the pedagogical tools necessary to effectively pres-
ent the material.
Acknowledgements
We would like to express our appreciation for the loving support we
have received from our wives (Hannah and Kikelomo), daughters
(Christina, Tamara, Jennifer, Motunrayo, Ann, and Joyce), son (Baixi),
and our extended family members.
At McGraw-Hill, we would like to thank the following editorial
and production staff: Raghu Srinivasan, publisher and senior sponsor-
ing editor; Lora Kalb-Neyens, developmental editors; Joyce Watters,
project manager; Carrie Burger, photo researcher; and Brenda Rolwes,
designer. Also, we appreciate the hard work of Tom Hartley at the Uni-
versity of Akron for his very detailed evaluation of various elements
of the text and his many valued suggestions for continued improve-
ment of this textbook.
We wish to thank Yongjian Fu and his outstanding team of stu-
dents, Bramarambha Elka and Saravaran Chinniah, for their efforts in
the development of KCIDE for Circuits. Their efforts to help us con-
tinue to improve this software are also appreciated.
The fourth edition has benefited greatly from the many outstand-
ing reviewers and symposium attendees who contributed to the success
of the first three editions! In addition, the following have made impor-
tant contributions to the fourth edition (in alphabetical order):
Tom Brewer, Georgia Tech
Andy Chan, City University of Hong Kong
Alan Tan Wee Chiat, Multimedia University
Norman Cox, University of Missouri-Rolla
Walter L. Green, University of Tennessee
ale29559_fm.qxd 07/28/2008 11:54 AM Page xviii
Preface xix
Dr. Gordon K. Lee, San Diego State University
Gary Perks, Cal Poly State University, San Luis Obispo
Dr. Raghu K. Settaluri, Oregon State University
Ramakant Srivastava, University of Florida
John Watkins, Wichita State University
Yik-Chung Wu, The University of Hong Kong
Xiao-Bang Xu, Clemson University
Finally, we appreciate the feedback received from instructors and stu-
dents who used the previous editions. We want this to continue, so please
keep sending us emails or direct them to the publisher. We can be reached
at c.alexander@ieee.org for Charles Alexander and sadiku@ieee.org for
Matthew Sadiku.
C. K. Alexander and M.N.O. Sadiku
ale29559_fm.qxd 07/28/2008 11:54 AM Page xix
xx
GUIDED TOUR
The main objective of this book is to present circuit analysis in a man-
ner that is clearer, more interesting, and easier to understand than other
texts. For you, the student, here are some features to help you study
and be successful in this course.
The four color art program brings circuit draw-
ings to life and enhances key concepts throughout
the text.
1.5 Power and Energy 11
To relate power and energy to voltage and current, we recall from
physics that:
Power is the time rate of expending or absorbing energy, measured in
watts (W).
We write this relationship as
(1.5)
where p is power in watts (W), w is energy in joules (J), and t is time
in seconds (s). From Eqs. (1.1), (1.3), and (1.5), it follows that
(1.6)
or
(1.7)
The power p in Eq. (1.7) is a time-varying quantity and is called the
instantaneous power. Thus, the power absorbed or supplied by an ele-
ment is the product of the voltage across the element and the current
through it. If the power has a sign, power is being delivered to or
absorbed by the element. If, on the other hand, the power has a sign,
power is being supplied by the element. But how do we know when
the power has a negative or a positive sign?
Current direction and voltage polarity play a major role in deter-
mining the sign of power. It is therefore important that we pay atten-
tion to the relationship between current i and voltage v in Fig. 1.8(a).
The voltage polarity and current direction must conform with those
shown in Fig. 1.8(a) in order for the power to have a positive sign.
This is known as the passive sign convention. By the passive sign con-
vention, current enters through the positive polarity of the voltage. In
this case, or implies that the element is absorbing
power. However, if or , as in Fig. 1.8(b), the element
is releasing or supplying power.
Passive sign convention is satisfied when the current enters through
the positive terminal of an element and p vi. If the current enters
through the negative terminal, p vi.
Unless otherwise stated, we will follow the passive sign conven-
tion throughout this text. For example, the element in both circuits of
Fig. 1.9 has an absorbing power of W because a positive current
enters the positive terminal in both cases. In Fig. 1.10, however, the
element is supplying power of W because a positive current enters
the negative terminal. Of course, an absorbing power of W is
equivalent to a supplying power of W. In general,
⫹Power absorbed ⫽ ⫺Power supplied
⫹12
⫺12
⫹12
⫹12
⫺
⫽
⫹
⫽
vi 6 0
p ⫽ ⫺vi
vi 7 0
p ⫽ ⫹vi
⫺
⫹
p ⫽ vi
p ⫽
dw
dt
⫽
dw
dq
ⴢ
dq
dt
⫽ vi
p ⫽
¢ dw
dt
p = +vi
(a)
v
+
−
p = −vi
(b)
v
+
−
i
i
Figure 1.8
Reference polarities for power using the
passive sign convention: (a) absorbing
power, (b) supplying power.
When the voltage and current directions
conform to Fig. 1.8(b), we have the ac-
tive sign convention and p vi.
⫹
⫽
(a)
4 V
3 A
(a)
+
−
3 A
4 V
3 A
(b)
+
−
Figure 1.9
Two cases of an element with an absorbing
power of 12 W: (a) W,
(b) W.
p ⫽ 4 ⫻ 3 ⫽ 12
p ⫽ 4 ⫻ 3 ⫽ 12
3 A
(a)
4 V
3 A
(a)
+
−
3 A
4 V
3 A
(b)
+
−
Figure 1.10
Two cases of an element with a supplying
power of 12 W: (a)
W, (b) W.
p ⫽ ⫺4 ⫻ 3 ⫽ ⫺12
⫺12
p ⫽ ⫺4 ⫻ 3 ⫽
Problem Solving
Although the problems to be solved during one’s career will vary in
complexity and magnitude, the basic principles to be followed remain
the same. The process outlined here is the one developed by the
authors over many years of problem solving with students, for the
solution of engineering problems in industry, and for problem solving
in research.
We will list the steps simply and then elaborate on them.
1. Carefully Define the problem.
2. Present everything you know about the problem.
3. Establish a set of Alternative solutions and determine the one that
promises the greatest likelihood of success.
4. Attempt a problem solution.
5. Evaluate the solution and check for accuracy.
6. Has the problem been solved Satisfactorily? If so, present the
solution; if not, then return to step 3 and continue through the
process again.
1. Carefully Define the problem. This may be the most important part
of the process, because it becomes the foundation for all the rest of the
steps. In general, the presentation of engineering problems is somewhat
incomplete. You must do all you can to make sure you understand the
problem as thoroughly as the presenter of the problem understands it.
Time spent at this point clearly identifying the problem will save you
considerable time and frustration later. As a student, you can clarify a
problem statement in a textbook by asking your professor. A problem
presented to you in industry may require that you consult several indi-
viduals. At this step, it is important to develop questions that need to
be addressed before continuing the solution process. If you have such
questions, you need to consult with the appropriate individuals or
resources to obtain the answers to those questions. With those answers,
you can now refine the problem, and use that refinement as the prob-
lem statement for the rest of the solution process.
2. Present everything you know about the problem. You are now ready
to write down everything you know about the problem and its possible
solutions. This important step will save you time and frustration later.
3. Establish a set of Alternative solutions and determine the one that
promises the greatest likelihood of success. Almost every problem will
have a number of possible paths that can lead to a solution. It is highly
desirable to identify as many of those paths as possible. At this point,
you also need to determine what tools are available to you, such as
PSpice and MATLAB and other software packages that can greatly
reduce effort and increase accuracy. Again, we want to stress that time
spent carefully defining the problem and investigating alternative
approaches to its solution will pay big dividends later. Evaluating the
alternatives and determining which promises the greatest likelihood of
success may be difficult but will be well worth the effort. Document
this process well since you will want to come back to it if the first
approach does not work.
4. Attempt a problem solution. Now is the time to actually begin
solving the problem. The process you follow must be well documented
1.8
20 Chapter 1 Basic Concepts
in order to present a detailed solution if successful, and to evaluate the
process if you are not successful. This detailed evaluation may lead to
corrections that can then lead to a successful solution. It can also lead
to new alternatives to try. Many times, it is wise to fully set up a solu-
tion before putting numbers into equations. This will help in checking
your results.
5. Evaluate the solution and check for accuracy. You now thoroughly
evaluate what you have accomplished. Decide if you have an acceptable
solution, one that you want to present to your team, boss, or professor.
6. Has the problem been solved Satisfactorily? If so, present the solu-
tion; if not, then return to step 3 and continue through the process
again. Now you need to present your solution or try another alterna-
tive. At this point, presenting your solution may bring closure to the
process. Often, however, presentation of a solution leads to further
refinement of the problem definition, and the process continues. Fol-
lowing this process will eventually lead to a satisfactory conclusion.
Now let us look at this process for a student taking an electrical
and computer engineering foundations course. (The basic process also
applies to almost every engineering course.) Keep in mind that
although the steps have been simplified to apply to academic types of
problems, the process as stated always needs to be followed. We con-
sider a simple example.
1.8 Problem Solving 21
Figure 1.19
Illustrative example.
Example 1.10
Solve for the current flowing through the resistor in Fig. 1.19.
Solution:
1. Carefully Define the problem. This is only a simple example, but
we can already see that we do not know the polarity on the 3-V source.
We have the following options. We can ask the professor what the
polarity should be. If we cannot ask, then we need to make a decision
on what to do next. If we have time to work the problem both ways,
we can solve for the current when the 3-V source is plus on top and
then plus on the bottom. If we do not have the time to work it both
ways, assume a polarity and then carefully document your decision.
Let us assume that the professor tells us that the source is plus on the
bottom as shown in Fig. 1.20.
2. Present everything you know about the problem. Presenting all that
we know about the problem involves labeling the circuit clearly so that
we define what we seek.
Given the circuit shown in Fig. 1.20, solve for .
We now check with the professor, if reasonable, to see if the prob-
lem is properly defined.
3. Establish a set of Alternative solutions and determine the one that
promises the greatest likelihood of success. There are essentially three
techniques that can be used to solve this problem. Later in the text you
will see that you can use circuit analysis (using Kirchhoff’s laws and
Ohm’s law), nodal analysis, and mesh analysis.
To solve for using circuit analysis will eventually lead to a
solution, but it will likely take more work than either nodal or mesh
i8⍀
i8⍀
8-⍀
Figure 1.20
Problem defintion.
2 Ω 4 Ω
8 Ω
5 V 3 V
+
− +
−
i8Ω
2 Ω 4 Ω
8 Ω
5 V 3 V
+
−
analysis. To solve for using mesh analysis will require writing
two simultaneous equations to find the two loop currents indicated in
Fig. 1.21. Using nodal analysis requires solving for only one unknown.
This is the easiest approach.
i8⍀
22 Chapter 1 Basic Concepts
2 Ω 4 Ω
8 Ω
5 V 3 V
+
− +
−
i2
i1 i3
+ − + −
+
−
v8Ω
v4Ω
v2Ω
Loop 1 Loop 2
v1
Figure 1.21
Using nodal analysis.
Therefore, we will solve for using nodal analysis.
4. Attempt a problem solution. We first write down all of the equa-
tions we will need in order to find .
Now we can solve for
5. Evaluate the solution and check for accuracy. We can now use
Kirchhoff’s voltage law (KVL) to check the results.
Applying KVL to loop 1,
Applying KVL to loop 2,
⫽ ⫺2 ⫹ 5 ⫺ 3 ⫽ 0 (Checks.)
⫽ ⫺(0.25 ⫻ 8) ⫹ (1.25 ⫻ 4) ⫺ 3
⫺v8⍀ ⫹ v4⍀ ⫺ 3 ⫽ ⫺(i2 ⫻ 8) ⫹ (i3 ⫻ 4) ⫺ 3
⫽ ⫺5 ⫹ 3 ⫹ 2 ⫽ 0 (Checks.)
⫽ ⫺5 ⫹ (⫺(⫺1.5)2) ⫹ (0.25 ⫻ 8)
⫺5 ⫹ v2⍀ ⫹ v8⍀ ⫽ ⫺5 ⫹ (⫺i1 ⫻ 2) ⫹ (i2 ⫻ 8)
i1 ⫹ i2 ⫹ i3 ⫽ ⫺1.5 ⫹ 0.25 ⫹ 1.25 ⫽ 0 (Checks.)
i3 ⫽
v1 ⫹ 3
4
⫽
2 ⫹ 3
4
⫽
5
4
⫽ 1.25 A
i2 ⫽ i8⍀ ⫽ 0.25 A
i1 ⫽
v1 ⫺ 5
2
⫽
2 ⫺ 5
2
⫽ ⫺
3
2
⫽ ⫺1.5 A
7v1 ⫽ ⫹14, v1 ⫽ ⫹2 V, i8⍀ ⫽
v1
8
⫽
2
8
⫽ 0.25 A
leads to (4v1 ⫺ 20) ⫹ (v1) ⫹ (2v1 ⫹ 6) ⫽ 0
8c
v1 ⫺ 5
2
⫹
v1 ⫺ 0
8
⫹
v1 ⫹ 3
4
d ⫽ 0
v1.
v1 ⫺ 5
2
⫹
v1 ⫺ 0
8
⫹
v1 ⫹ 3
4
⫽ 0
i8⍀ ⫽ i2, i2 ⫽
v1
8
, i8⍀ ⫽
v1
8
i8⍀
i8⍀
So we now have a very high degree of confidence in the accuracy
of our answer.
6. Has the problem been solved Satisfactorily? If so, present the solu-
tion; if not, then return to step 3 and continue through the process
again. This problem has been solved satisfactorily.
The current through the 8- resistor is 0.25 A flowing down through
the 8- resistor.
⍀
⍀
1.9 Summary 23
Try applying this process to some of the more difficult problems at the
end of the chapter.
Practice Problem 1.10
Summary
1. An electric circuit consists of electrical elements connected
together.
2. The International System of Units (SI) is the international mea-
surement language, which enables engineers to communicate their
results. From the six principal units, the units of other physical
quantities can be derived.
3. Current is the rate of charge flow.
4. Voltage is the energy required to move 1 C of charge through an
element.
5. Power is the energy supplied or absorbed per unit time. It is also
the product of voltage and current.
6. According to the passive sign convention, power assumes a posi-
tive sign when the current enters the positive polarity of the voltage
across an element.
7. An ideal voltage source produces a specific potential difference
across its terminals regardless of what is connected to it. An ideal
current source produces a specific current through its terminals
regardless of what is connected to it.
8. Voltage and current sources can be dependent or independent. A
dependent source is one whose value depends on some other cir-
cuit variable.
9. Two areas of application of the concepts covered in this chapter
are the TV picture tube and electricity billing procedure.
p ⫽
dw
dt
⫽ vi
v ⫽
dw
dq
i ⫽
dq
dt
1.9
A six-step problem-solving method-
ology is introduced in Chapter 1 and
incorporated into worked examples
throughout the text to promote
sound, step-by-step problem-solving
practices.
ale29559_fm.qxd 07/28/2008 11:54 AM Page xx
Guided Tour xxi
Each illustrative example is immediately followed by
a practice problem and answer to test understanding of
the preceding example.
PSpice® for Windows is a student-friendly tool intro-
duced to students early in the text and used through-
out, with discussions and examples at the end of each
appropriate chapter.
90 Chapter 3 Methods of Analysis
For the circuit shown in Fig. 3.9, find the node voltages.
Solution:
The supernode contains the 2-V source, nodes 1 and 2, and the 10-
resistor. Applying KCL to the supernode as shown in Fig. 3.10(a) gives
Expressing and in terms of the node voltages
or
(3.3.1)
To get the relationship between and we apply KVL to the circuit
in Fig. 3.10(b). Going around the loop, we obtain
(3.3.2)
From Eqs. (3.3.1) and (3.3.2), we write
or
and Note that the 10- resistor does not
make any difference because it is connected across the supernode.
⍀
v2 ⫽ v1 ⫹ 2 ⫽ ⫺5.333 V.
3v1 ⫽ ⫺22 1 v1 ⫽ ⫺7.333 V
v2 ⫽ v1 ⫹ 2 ⫽ ⫺20 ⫺ 2v1
⫺v1 ⫺ 2 ⫹ v2 ⫽ 0 1 v2 ⫽ v1 ⫹ 2
v2,
v1
v2 ⫽ ⫺20 ⫺ 2v1
2 ⫽
v1 ⫺ 0
2
⫹
v2 ⫺ 0
4
⫹ 7 1 8 ⫽ 2v1 ⫹ v2 ⫹ 28
i2
i1
2 ⫽ i1 ⫹ i2 ⫹ 7
⍀
Example 3.3
Figure 3.9
For Example 3.3.
+
−
2 A
2 V
7 A
4 Ω
10 Ω
2 Ω
v1 v2
2 A
2 A
7 A
7 A
2 Ω 4 Ω
v2
v1
i1 i2
1 2
(a)
+
−
(b)
2 V
1 2
+
+
− −
v1 v2
Figure 3.10
Applying: (a) KCL to the supernode, (b) KVL to the loop.
Figure 3.11
For Practice Prob. 3.3.
21 V
9 V
4 Ω
3 Ω 2 Ω 6 Ω
+
−
+
−
i
v
+
−
Practice Problem 3.3 Find v and i in the circuit of Fig. 3.11.
Answer: 4.2 A.
⫺0.6 V,
106 Chapter 3 Methods of Analysis
are displayed on VIEWPOINTS and also saved in output file
exam310.out. The output file includes the following:
NODE VOLTAGE NODE VOLTAGE NODE VOLTAGE
(1) 120.0000 (2) 81.2900 (3) 89.0320
indicating that V3 ⫽ 89.032 V.
V2 ⫽ 81.29 V,
V1 ⫽ 120 V,
+
−
R1 R3
20 10
120 V V1 R2 R4
30 40 I1 3 A
IDC
0
1 2 3
120.0000 81.2900 89.0320
Figure 3.32
For Example 3.10; the schematic of the circuit in Fig. 3.31.
For the circuit in Fig. 3.33, use PSpice to find the node voltages.
Practice Problem 3.10
+
−
2 A
200 V
30 Ω 60 Ω 50 Ω
100 Ω
25 Ω
1 2 3
0
Figure 3.33
For Practice Prob. 3.10.
Answer: V3 ⫽ 200 V.
V2 ⫽ 57.14 V,
V1 ⫽ ⫺40 V,
In the circuit of Fig. 3.34, determine the currents and i3.
i1, i2,
Example 3.11
+
−
+
−
24 V
1 Ω
i1 i2 i3
+
−
4 Ω 2 Ω
2 Ω 8 Ω 4 Ω
3vo
vo
Figure 3.34
For Example 3.11.
3.9 Applications: DC Transistor Circuits 107
Solution:
The schematic is shown in Fig. 3.35. (The schematic in Fig. 3.35
includes the output results, implying that it is the schematic displayed
on the screen after the simulation.) Notice that the voltage-controlled
voltage source E1 in Fig. 3.35 is connected so that its input is the
voltage across the 4- resistor; its gain is set equal to 3. In order to
display the required currents, we insert pseudocomponent IPROBES in
the appropriate branches. The schematic is saved as exam311.sch and
simulated by selecting Analysis/Simulate. The results are displayed on
IPROBES as shown in Fig. 3.35 and saved in output file exam311.out.
From the output file or the IPROBES, we obtain A and
A.
i3 ⫽ 2.667
i1 ⫽ i2 ⫽ 1.333
⍀
+
−
24 V V1
R1
4
R2 2 R3 8 R4 4
1.333E + 00 1.333E + 00 2.667E + 00
0
R6
1
R5
2
E E1
+
−
− +
Figure 3.35
The schematic of the circuit in Fig. 3.34.
Use PSpice to determine currents and in the circuit of Fig. 3.36.
Answer: A, A, A.
i3 ⫽ 2
i2 ⫽ 2.286
i1 ⫽ ⫺0.4286
i3
i1, i2, Practice Problem 3.11
+
−
2 A
10 V
2 Ω
i1
i1
i2
4 Ω
1 Ω 2 Ω
i3
Figure 3.36
For Practice Prob. 3.11.
Applications: DC Transistor Circuits
Most of us deal with electronic products on a routine basis and have
some experience with personal computers. A basic component for
the integrated circuits found in these electronics and computers is the
active, three-terminal device known as the transistor. Understanding
the transistor is essential before an engineer can start an electronic cir-
cuit design.
Figure 3.37 depicts various kinds of transistors commercially avail-
able. There are two basic types of transistors: bipolar junction transis-
tors (BJTs) and field-effect transistors (FETs). Here, we consider only
the BJTs, which were the first of the two and are still used today. Our
objective is to present enough detail about the BJT to enable us to apply
the techniques developed in this chapter to analyze dc transistor circuits.
p
3.9
369
Sinusoids and
Phasors
He who knows not, and knows not that he knows not, is a fool—
shun him. He who knows not, and knows that he knows not, is a child—
teach him. He who knows, and knows not that he knows, is asleep—wake
him up. He who knows, and knows that he knows, is wise—follow him.
—Persian Proverb
c h a p t e r
9
Enhancing Your Skills and Your Career
ABET EC 2000 criteria (3.d), “an ability to function on
multi-disciplinary teams.”
The “ability to function on multidisciplinary teams” is inherently crit-
ical for the working engineer. Engineers rarely, if ever, work by them-
selves. Engineers will always be part of some team. One of the things
I like to remind students is that you do not have to like everyone on a
team; you just have to be a successful part of that team.
Most frequently, these teams include individuals from of a variety
of engineering disciplines, as well as individuals from nonengineering
disciplines such as marketing and finance.
Students can easily develop and enhance this skill by working in
study groups in every course they take. Clearly, working in study
groups in nonengineering courses as well as engineering courses out-
side your discipline will also give you experience with multidiscipli-
nary teams.
Photo by Charles Alexander
The last section in each chapter is devoted to appli-
cations of the concepts covered in the chapter to help
students apply the concepts to real-life situations.
Icons next to the end-of-chapter homework problems
let students know which problems relate to engineer-
ing design and which problems can be solved using
PSpice or MATLAB. Appendices on these computer
programs provide tutorials for their use.
Each chapter opens with a discussion about how to
enhance skills that contribute to successful problem
solving as well as successful careers or a career-
oriented talk on a sub-discipline of electrical engi-
neering to give students some real-world applications
of what they are learning.
ale29559_fm.qxd 07/28/2008 11:54 AM Page xxi
xxii Guided Tour
Supplements for Students and Instructors
McGraw-Hill’s ARIS—Assessment, Review,
and Instruction System is a complete, online
tutorial, electronic homework, and course man-
agement system, designed for greater ease of
use than any other system available. With ARIS, instructors can create
and share course materials and assignments with other instructors, edit
questions and algorithms, import their own content, and create
announcements and due dates for assignments. ARIS has automatic
grading and reporting of easy-to-assign algorithmically-generated
homework, quizzing, and testing. Once a student is registered in the
course, all student activity within McGraw-Hill’s ARIS is automatically
recorded and available to the instructor through a fully integrated grade
book that can be downloaded to Excel.
www.mhhe.com/alexander
Knowledge Capturing Integrated Design Environment
for Circuits (KCIDE for Circuits) software, linked
from ARIS, enhances student understanding of the
six-step problem-solving methodology in the book.
KCIDE for Circuits allows students to work a circuit
problem in PSpice and MATLAB, track the evolution
of their solution, and save a record of their process for
future reference. Appendix F walks the user through
this program.
ale29559_fm.qxd 07/28/2008 11:54 AM Page xxii
Guided Tour xxiii
Other resources provided on ARIS.
For Students:
— Network Analysis Tutorials—a series of interactive quizzes to help
students practice fundamental concepts in circuits.
— FE Exam Interactive Review Quizzes—chapter based self-quizzes
provide hints for solutions and correct solution methods, and help
students prepare for the NCEES Fundamentals of Engineering
Examination.
— Problem Solving Made Almost Easy—a companion workbook to the
text, featuring 150 additional problems with complete solutions.
— Complete solutions to Practice Problems in the text
— Flashcards of key terms
— Web links
For Instructors:
— Image Sets—electronic files of text figures for easy integration into
your course presentations, exams, and assignments.
— Transition Guides—compare coverage of the third edition to other
popular circuits books at the section level to aid transition to teach-
ing from our text.
ale29559_fm.qxd 07/28/2008 11:54 AM Page xxiii
ale29559_fm.qxd 07/28/2008 11:54 AM Page xxiv
A Note to the Student
This may be your first course in electrical engineering. Although elec-
trical engineering is an exciting and challenging discipline, the course
may intimidate you. This book was written to prevent that. A good text-
book and a good professor are an advantage—but you are the one who
does the learning. If you keep the following ideas in mind, you will do
very well in this course.
• This course is the foundation on which most other courses in the
electrical engineering curriculum rest. For this reason, put in as
much effort as you can. Study the course regularly.
• Problem solving is an essential part of the learning process. Solve as
many problems as you can. Begin by solving the practice problem
following each example, and then proceed to the end-of-chapter
problems. The best way to learn is to solve a lot of problems. An
asterisk in front of a problem indicates a challenging problem.
• Spice, a computer circuit analysis program, is used throughout the
textbook. PSpice, the personal computer version of Spice, is the
popular standard circuit analysis program at most universities.
PSpice for Windows is described in Appendix D. Make an effort
to learn PSpice, because you can check any circuit problem with
PSpice and be sure you are handing in a correct problem solution.
• MATLAB is another software that is very useful in circuit analysis
and other courses you will be taking. A brief tutorial on MATLAB
is given in Appendix E to get you started. The best way to learn
MATLAB is to start working with it once you know a few commands.
• Each chapter ends with a section on how the material covered in
the chapter can be applied to real-life situations. The concepts in
this section may be new and advanced to you. No doubt, you will
learn more of the details in other courses. We are mainly interested
in gaining a general familiarity with these ideas.
• Attempt the review questions at the end of each chapter. They
will help you discover some “tricks” not revealed in class or in the
textbook.
• Clearly a lot of effort has gone into making the technical details in
this book easy to understand. It also contains all the mathematics
and physics necessary to understand the theory and will be very
useful in your other engineering courses. However, we have also
focused on creating a reference for you to use both in school as
well as when working in industry or seeking a graduate degree.
• It is very tempting to sell your book after you have completed your
classroom experience; however, our advice to you is DO NOT SELL
YOUR ENGINEERING BOOKS! Books have always been expen-
sive, however, the cost of this book is virtually the same as I paid
for my circuits text back in the early 60s in terms of real dollars.
In fact, it is actually cheaper. In addition, engineering books of
the past are no where near as complete as what is available now.
xxv
ale29559_fm.qxd 07/28/2008 11:54 AM Page xxv
When I was a student, I did not sell any of my engineering text-
books and was very glad I did not! I found that I needed most of
them throughout my career.
A short review on finding determinants is covered in Appendix A,
complex numbers in Appendix B, and mathematical formulas in Appen-
dix C. Answers to odd-numbered problems are given in Appendix G.
Have fun!
C. K. A. and M. N. O. S.
xxvi A Note to the Student
ale29559_fm.qxd 07/28/2008 11:54 AM Page xxvi
xxvii
About the Authors
Charles K. Alexander is professor of electrical and computer engi-
neering of the Fenn College of Engineering at Cleveland State Uni-
versity, Cleveland, Ohio. He is also the Director of The Center for
Research in Electronics and Aerospace Technology (CREATE), and is
the Managing Director of the Wright Center for Sensor Systems
(WCSSE). From 2002 until 2006 he was Dean of the Fenn College of
Engineering. From 2004 until 2007, he was Director of Ohio ICE, a
research center in instrumentation, controls, electronics, and sensors (a
coalition of CSU, Case, the University of Akron, and a number of Ohio
industries). From 1998 until 2002, he was interim director (2000 and
2001) of the Institute for Corrosion and Multiphase Technologies and
Stocker Visiting Professor of electrical engineering and computer sci-
ence at Ohio University. From 1994–1996 he was dean of engineering
and computer science at California State University, Northridge.
From 1989–1994 he was acting dean of the college of engineer-
ing at Temple University, and from 1986–1989 he was professor and
chairman of the department of electrical engineering at Temple. From
1980–1986 he held the same positions at Tennessee Technological
University. He was an associate professor and a professor of electrical
engineering at Youngstown State University from 1972–1980, where
he was named Distinguished Professor in 1977 in recognition of
“outstanding teaching and research.” He was assistant professor of
electrical engineering at Ohio University in 1971–1972. He received
the Ph.D. (1971) and M.S.E.E. (1967) from Ohio University and the
B.S.E.E. (1965) from Ohio Northern University.
Dr. Alexander has been a consultant to 23 companies and gov-
ernmental organizations, including the Air Force and Navy and sev-
eral law firms. He has received over $85 million in research and
development funds for projects ranging from solar energy to software
engineering. He has authored 40 publications, including a workbook
and a videotape lecture series, and is coauthor of Fundamentals of
Electric Circuits, Problem Solving Made Almost Easy, and the fifth
edition of the Standard Handbook of Electronic Engineering, with
McGraw-Hill. He has made more than 500 paper, professional, and
technical presentations.
Dr. Alexander is a fellow of the IEEE and served as its president
and CEO in 1997. In 1993 and 1994 he was IEEE vice president, pro-
fessional activities, and chair of the United States Activities Board
(USAB). In 1991–1992 he was region 2 director, serving on the
Regional Activities Board (RAB) and USAB. He has also been a mem-
ber of the Educational Activities Board. He served as chair of the USAB
Member Activities Council and vice chair of the USAB Professional
Activities Council for Engineers, and he chaired the RAB Student
Activities Committee and the USAB Student Professional Awareness
Committee.
Charles K. Alexander
ale29559_fm.qxd 07/28/2008 11:54 AM Page xxvii
In 1998 he received the Distinguished Engineering Education
Achievement Award from the Engineering Council, and in 1996 he
received the Distinguished Engineering Education Leadership Award
from the same group. When he became a fellow of the IEEE in 1994,
the citation read “for leadership in the field of engineering education
and the professional development of engineering students.” In 1984 he
received the IEEE Centennial Medal, and in 1983 he received the
IEEE/RAB Innovation Award, given to the IEEE member who best
contributes to RAB’s goals and objectives.
Matthew N. O. Sadiku is presently a professor at Prairie View A&M
University. Prior to joining Prairie View, he taught at Florida Atlantic
University, Boca Raton, and Temple University, Philadelphia. He has
also worked for Lucent/Avaya and Boeing Satellite Systems.
Dr. Sadiku is the author of over 170 professional papers and almost
30 books including Elements of Electromagnetics (Oxford University
Press, 3rd ed., 2001), Numerical Techniques in Electromagnetics (2nd
ed., CRC Press, 2000), Simulation of Local Area Networks (with M.
IIyas, CRC Press, 1994), Metropolitan Area Networks (CRC Press,
1994), and Fundamentals of Electric Circuits (with C. K. Alexander,
McGraw-Hill, 3rd ed., 2007). His books are used worldwide, and some
of them have been translated into Korean, Chinese, Italian, and Spanish.
He was the recipient of the 2000 McGraw-Hill/Jacob Millman Award for
outstanding contributions in the field of electrical engineering. He was
the IEEE region 2 Student Activities Committee chairman and is an asso-
ciate editor for IEEE “Transactions on Education.” He received his Ph.D.
at Tennessee Technological University, Cookeville.
xxviii About the Authors
Matthew N. O. Sadiku
ale29559_fm.qxd 07/28/2008 11:54 AM Page xxviii
Fundamentals of
Electric Circuits
ale29559_fm.qxd 07/28/2008 11:54 AM Page 1
P A R T O N E
DC Circuits
OUTLINE
1 Basic Concepts
2 Basic Laws
3 Methods of Analysis
4 Circuit Theorems
5 Operational Amplifiers
6 Capacitors and Inductors
7 First-Order Circuits
8 Second-Order Circuits
ale29559_ch01.qxd 07/08/2008 10:37 AM Page 2
3
Basic Concepts
Some books are to be tasted, others to be swallowed, and some few to
be chewed and digested.
—Francis Bacon
c h a p t e r
1
Enhancing Your Skills and Your Career
ABET EC 2000 criteria (3.a), “an ability to apply knowledge
of mathematics, science, and engineering.”
As students, you are required to study mathematics, science, and engi-
neering with the purpose of being able to apply that knowledge to the
solution of engineering problems. The skill here is the ability to apply
the fundamentals of these areas in the solution of a problem. So, how
do you develop and enhance this skill?
The best approach is to work as many problems as possible in all
of your courses. However, if you are really going to be successful with
this, you must spend time analyzing where and when and why you have
difficulty in easily arriving at successful solutions. You may be sur-
prised to learn that most of your problem-solving problems are with
mathematics rather than your understanding of theory. You may also
learn that you start working the problem too soon. Taking time to think
about the problem and how you should solve it will always save you
time and frustration in the end.
What I have found that works best for me is to apply our six-
step problem-solving technique. Then I carefully identify the areas
where I have difficulty solving the problem. Many times, my actual
deficiencies are in my understanding and ability to use correctly cer-
tain mathematical principles. I then return to my fundamental math
texts and carefully review the appropriate sections and in some cases
work some example problems in that text. This brings me to another
important thing you should always do: Keep nearby all your basic
mathematics, science, and engineering textbooks.
This process of continually looking up material you thought you
had acquired in earlier courses may seem very tedious at first; how-
ever, as your skills develop and your knowledge increases, this process
will become easier and easier. On a personal note, it is this very process
that led me from being a much less than average student to someone
who could earn a Ph.D. and become a successful researcher.
Photo by Charles Alexander
ale29559_ch01.qxd 07/08/2008 10:37 AM Page 3
Introduction
Electric circuit theory and electromagnetic theory are the two funda-
mental theories upon which all branches of electrical engineering are
built. Many branches of electrical engineering, such as power, electric
machines, control, electronics, communications, and instrumentation,
are based on electric circuit theory. Therefore, the basic electric circuit
theory course is the most important course for an electrical engineer-
ing student, and always an excellent starting point for a beginning stu-
dent in electrical engineering education. Circuit theory is also valuable
to students specializing in other branches of the physical sciences
because circuits are a good model for the study of energy systems in
general, and because of the applied mathematics, physics, and topol-
ogy involved.
In electrical engineering, we are often interested in communicating
or transferring energy from one point to another. To do this requires an
interconnection of electrical devices. Such interconnection is referred
to as an electric circuit, and each component of the circuit is known as
an element.
An electric circuit is an interconnection of electrical elements.
A simple electric circuit is shown in Fig. 1.1. It consists of three
basic elements: a battery, a lamp, and connecting wires. Such a simple
circuit can exist by itself; it has several applications, such as a flash-
light, a search light, and so forth.
A complicated real circuit is displayed in Fig. 1.2, representing the
schematic diagram for a radio receiver. Although it seems complicated,
this circuit can be analyzed using the techniques we cover in this book.
Our goal in this text is to learn various analytical techniques and
computer software applications for describing the behavior of a circuit
like this.
Electric circuits are used in numerous electrical systems to accom-
plish different tasks. Our objective in this book is not the study of
various uses and applications of circuits. Rather our major concern is
the analysis of the circuits. By the analysis of a circuit, we mean a
study of the behavior of the circuit: How does it respond to a given
input? How do the interconnected elements and devices in the circuit
interact?
We commence our study by defining some basic concepts. These
concepts include charge, current, voltage, circuit elements, power, and
energy. Before defining these concepts, we must first establish a sys-
tem of units that we will use throughout the text.
Systems of Units
As electrical engineers, we deal with measurable quantities. Our mea-
surement, however, must be communicated in a standard language that
virtually all professionals can understand, irrespective of the country
where the measurement is conducted. Such an international measurement
1.2
1.1
4 Chapter 1 Basic Concepts
+
−
Current
Lamp
Battery
Figure 1.1
A simple electric circuit.
ale29559_ch01.qxd 07/08/2008 10:37 AM Page 4
language is the International System of Units (SI), adopted by the
General Conference on Weights and Measures in 1960. In this system,
there are six principal units from which the units of all other physical
quantities can be derived. Table 1.1 shows the six units, their symbols,
and the physical quantities they represent. The SI units are used
throughout this text.
One great advantage of the SI unit is that it uses prefixes based on
the power of 10 to relate larger and smaller units to the basic unit.
Table 1.2 shows the SI prefixes and their symbols. For example, the
following are expressions of the same distance in meters (m):
600,000,000 mm 600,000 m 600 km
1.2 Systems of Units 5
2, 5, 6
C
Oscillator
E
B
R2
10 k
R3
10 k
R1 47
Y1
7 MHz
C6 5
L2
22.7 ␮H
(see text)
to
U1, Pin 8
R10
10 k
GAIN +
+
C16
100 ␮F
16 V
C11
100 ␮F
16 V
C10
1.0 ␮F
16 V
C9
1.0 ␮F
16 V
C15
0.47
16 V
C17
100 ␮F
16 V
+
−
12-V dc
Supply
Audio
Output
+
C18
0.1
R12
10
1
4
2
3
C14
0.0022
0.1
C13
U2A
1 2 TL072
U2B
1 2 TL072
R9
15 k
R5
100 k
R8
15 k
R6
100 k
5
6
R7
1 M
C12 0.0033
+
L3
1 mH
R11
47
C8
0.1
Q1
2N2222A
7
C3 0.1
L1
0.445 ␮H
Antenna C1
2200 pF
C2
2200 pF
1
8
7
U1
SBL-1
Mixer
3, 4
C7
532
C4
910
C5
910
R4
220
U3
LM386N
Audio power amp
5
4
6
3
2
+
+
−
+
−
+
−
+
8
Figure 1.2
Electric circuit of a radio receiver.
Reproduced with permission from QST, August 1995, p. 23.
TABLE 1.1
Six basic SI units and one derived unit relevant to this text.
Quantity Basic unit Symbol
Length meter m
Mass kilogram kg
Time second s
Electric current ampere A
Thermodynamic temperature kelvin K
Luminous intensity candela cd
Charge coulomb C
TABLE 1.2
The SI prefixes.
Multiplier Prefix Symbol
exa E
peta P
tera T
giga G
mega M
kilo k
hecto h
10 deka da
deci d
centi c
milli m
micro
nano n
pico p
femto f
atto a
1018
1015
1012
109
m
106
103
102
101
102
103
106
109
1012
1015
1018
ale29559_ch01.qxd 07/17/2008 11:15 AM Page 5
Charge and Current
The concept of electric charge is the underlying principle for explain-
ing all electrical phenomena. Also, the most basic quantity in an elec-
tric circuit is the electric charge. We all experience the effect of electric
charge when we try to remove our wool sweater and have it stick to
our body or walk across a carpet and receive a shock.
Charge is an electrical property of the atomic particles of which mat-
ter consists, measured in coulombs (C).
We know from elementary physics that all matter is made of funda-
mental building blocks known as atoms and that each atom consists of
electrons, protons, and neutrons. We also know that the charge e on an
electron is negative and equal in magnitude to C, while
a proton carries a positive charge of the same magnitude as the elec-
tron. The presence of equal numbers of protons and electrons leaves an
atom neutrally charged.
The following points should be noted about electric charge:
1. The coulomb is a large unit for charges. In 1 C of charge, there
are electrons. Thus realistic or
laboratory values of charges are on the order of pC, nC, or C.1
2. According to experimental observations, the only charges that
occur in nature are integral multiples of the electronic charge
3. The law of conservation of charge states that charge can neither
be created nor destroyed, only transferred. Thus the algebraic sum
of the electric charges in a system does not change.
We now consider the flow of electric charges. A unique feature of
electric charge or electricity is the fact that it is mobile; that is, it can
be transferred from one place to another, where it can be converted to
another form of energy.
When a conducting wire (consisting of several atoms) is con-
nected to a battery (a source of electromotive force), the charges are
compelled to move; positive charges move in one direction while neg-
ative charges move in the opposite direction. This motion of charges
creates electric current. It is conventional to take the current flow as
the movement of positive charges. That is, opposite to the flow of neg-
ative charges, as Fig. 1.3 illustrates. This convention was introduced
by Benjamin Franklin (1706–1790), the American scientist and inven-
tor. Although we now know that current in metallic conductors is due
to negatively charged electrons, we will follow the universally
accepted convention that current is the net flow of positive charges.
Thus,
Electric current is the time rate of change of charge, measured in
amperes (A).
e  1.602  1019
C.
m
1(1.602  1019
)  6.24  1018
1.602  1019
1.3
6 Chapter 1 Basic Concepts
1
However, a large power supply capacitor can store up to 0.5 C of charge.
Battery
I − −
− −
+ −
Figure 1.3
Electric current due to flow of electronic
charge in a conductor.
A convention is a standard way of
describing something so that others in
the profession can understand what
we mean. We will be using IEEE con-
ventions throughout this book.
ale29559_ch01.qxd 07/08/2008 10:38 AM Page 6
Mathematically, the relationship between current i, charge q, and time t is
(1.1)
where current is measured in amperes (A), and
The charge transferred between time and t is obtained by integrat-
ing both sides of Eq. (1.1). We obtain
(1.2)
The way we define current as i in Eq. (1.1) suggests that current need
not be a constant-valued function. As many of the examples and prob-
lems in this chapter and subsequent chapters suggest, there can be sev-
eral types of current; that is, charge can vary with time in several ways.
If the current does not change with time, but remains constant, we
call it a direct current (dc).
A direct current (dc) is a current that remains constant with time.
By convention the symbol I is used to represent such a constant current.
A time-varying current is represented by the symbol i. A common
form of time-varying current is the sinusoidal current or alternating
current (ac).
An alternating current (ac) is a current that varies sinusoidally with time.
Such current is used in your household, to run the air conditioner,
refrigerator, washing machine, and other electric appliances. Figure 1.4
Q 
¢

t
t0
i dt
t0
1 ampere  1 coulomb/second
i 
¢ dq
dt
1.3 Charge and Current 7
Andre-Marie Ampere (1775–1836), a French mathematician and
physicist, laid the foundation of electrodynamics. He defined the elec-
tric current and developed a way to measure it in the 1820s.
Born in Lyons, France, Ampere at age 12 mastered Latin in a few
weeks, as he was intensely interested in mathematics and many of the
best mathematical works were in Latin. He was a brilliant scientist and
a prolific writer. He formulated the laws of electromagnetics. He in-
vented the electromagnet and the ammeter. The unit of electric current,
the ampere, was named after him.
Historical
I
0 t
(a)
(b)
i
t
0
Figure 1.4
Two common types of current: (a) direct
current (dc), (b) alternating current (ac).
The Burndy Library Collection
at The Huntington Library,
San Marino, California.
ale29559_ch01.qxd 07/16/2008 12:22 PM Page 7
shows direct current and alternating current; these are the two most
common types of current. We will consider other types later in the
book.
Once we define current as the movement of charge, we expect cur-
rent to have an associated direction of flow. As mentioned earlier, the
direction of current flow is conventionally taken as the direction of pos-
itive charge movement. Based on this convention, a current of 5 A may
be represented positively or negatively as shown in Fig. 1.5. In other
words, a negative current of A flowing in one direction as shown
in Fig. 1.5(b) is the same as a current of A flowing in the opposite
direction.
5
5
8 Chapter 1 Basic Concepts
5 A
(a)
−5 A
(b)
Figure 1.5
Conventional current flow: (a) positive
current flow, (b) negative current flow.
How much charge is represented by 4,600 electrons?
Solution:
Each electron has C. Hence 4,600 electrons will have
1.602  1019
C/electron  4,600 electrons  7.369  1016
C
1.602  1019
Example 1.1
Practice Problem 1.2
Example 1.2
Calculate the amount of charge represented by four million protons.
Answer: C.
6.408  1013
Practice Problem 1.1
If in Example 1.2, find the current at s.
Answer: 7.36 mA.
t  0.5
q  (10  10e2t
) mC,
The total charge entering a terminal is given by mC.
Calculate the current at s.
Solution:
At
i  5 sin 2p  10p cos 2p  0  10p  31.42 mA
t  0.5,
i 
dq
dt

d
dt
(5t sin 4pt) mC/s  (5 sin 4pt  20pt cos 4pt) mA
t  0.5
q  5t sin 4pt
ale29559_ch01.qxd 07/08/2008 10:38 AM Page 8
Voltage
As explained briefly in the previous section, to move the electron in a
conductor in a particular direction requires some work or energy trans-
fer. This work is performed by an external electromotive force (emf),
typically represented by the battery in Fig. 1.3. This emf is also known
as voltage or potential difference. The voltage between two points
a and b in an electric circuit is the energy (or work) needed to move
a unit charge from a to b; mathematically,
(1.3)
where w is energy in joules (J) and q is charge in coulombs (C). The
voltage or simply v is measured in volts (V), named in honor of
the Italian physicist Alessandro Antonio Volta (1745–1827), who
invented the first voltaic battery. From Eq. (1.3), it is evident that
Thus,
Voltage (or potential difference) is the energy required to move a unit
charge through an element, measured in volts (V).
Figure 1.6 shows the voltage across an element (represented by a
rectangular block) connected to points a and b. The plus and minus
signs are used to define reference direction or voltage polarity. The
can be interpreted in two ways: (1) point a is at a potential of vab
vab
()
()
1 volt  1 joule/coulomb  1 newton-meter/coulomb
vab
vab 
¢ dw
dq
vab
1.4
1.4 Voltage 9
Example 1.3
Determine the total charge entering a terminal between and
s if the current passing the terminal is A.
Solution:
 at3

t2
2
b `
2
1
 (8  2)  a1 
1
2
b  5.5 C
Q  
2
t1
i dt  
2
1
(3t2
 t)dt
i  (3t2
 t)
t  2
t  1 s
The current flowing through an element is
Calculate the charge entering the element from to s.
Answer: 6.667 C.
t  2
t  0
i  e
2 A, 0 6 t 6 1
2t2
A, t 7 1
a
b
vab
+
−
Figure 1.6
Polarity of voltage .
vab
Practice Problem 1.3
ale29559_ch01.qxd 07/08/2008 10:38 AM Page 9
Historical
Alessandro Antonio Volta (1745–1827), an Italian physicist,
invented the electric battery—which provided the first continuous flow
of electricity—and the capacitor.
Born into a noble family in Como, Italy, Volta was performing
electrical experiments at age 18. His invention of the battery in 1796
revolutionized the use of electricity. The publication of his work in
1800 marked the beginning of electric circuit theory. Volta received
many honors during his lifetime. The unit of voltage or potential dif-
ference, the volt, was named in his honor.
volts higher than point b, or (2) the potential at point a with respect to
point b is . It follows logically that in general
(1.4)
For example, in Fig. 1.7, we have two representations of the same volt-
age. In Fig. 1.7(a), point a is V above point b; in Fig. 1.7(b), point b
is V above point a. We may say that in Fig. 1.7(a), there is a 9-V
voltage drop from a to b or equivalently a 9-V voltage rise from b to
a. In other words, a voltage drop from a to b is equivalent to a volt-
age rise from b to a.
Current and voltage are the two basic variables in electric circuits.
The common term signal is used for an electric quantity such as a cur-
rent or a voltage (or even electromagnetic wave) when it is used for
conveying information. Engineers prefer to call such variables signals
rather than mathematical functions of time because of their importance
in communications and other disciplines. Like electric current, a con-
stant voltage is called a dc voltage and is represented by V, whereas a
sinusoidally time-varying voltage is called an ac voltage and is repre-
sented by v. A dc voltage is commonly produced by a battery; ac volt-
age is produced by an electric generator.
Power and Energy
Although current and voltage are the two basic variables in an electric
circuit, they are not sufficient by themselves. For practical purposes,
we need to know how much power an electric device can handle. We
all know from experience that a 100-watt bulb gives more light than a
60-watt bulb. We also know that when we pay our bills to the electric
utility companies, we are paying for the electric energy consumed over
a certain period of time. Thus, power and energy calculations are
important in circuit analysis.
1.5
9
9
vab  vba
vab
10 Chapter 1 Basic Concepts
9 V
(a)
a
b
+
−
−9 V
(b)
a
b
+
−
Figure 1.7
Two equivalent representations of the
same voltage : (a) point a is 9 V above
point b, (b) point b is 9 V above point a.

vab
Keep in mind that electric current is
always through an element and that
electric voltage is always across the
element or between two points.
The Burndy Library Collection
at The Huntington Library,
San Marino, California.
ale29559_ch01.qxd 07/16/2008 12:23 PM Page 10
1.5 Power and Energy 11
To relate power and energy to voltage and current, we recall from
physics that:
Power is the time rate of expending or absorbing energy, measured in
watts (W).
We write this relationship as
(1.5)
where p is power in watts (W), w is energy in joules (J), and t is time
in seconds (s). From Eqs. (1.1), (1.3), and (1.5), it follows that
(1.6)
or
(1.7)
The power p in Eq. (1.7) is a time-varying quantity and is called the
instantaneous power. Thus, the power absorbed or supplied by an ele-
ment is the product of the voltage across the element and the current
through it. If the power has a sign, power is being delivered to or
absorbed by the element. If, on the other hand, the power has a sign,
power is being supplied by the element. But how do we know when
the power has a negative or a positive sign?
Current direction and voltage polarity play a major role in deter-
mining the sign of power. It is therefore important that we pay atten-
tion to the relationship between current i and voltage v in Fig. 1.8(a).
The voltage polarity and current direction must conform with those
shown in Fig. 1.8(a) in order for the power to have a positive sign.
This is known as the passive sign convention. By the passive sign con-
vention, current enters through the positive polarity of the voltage. In
this case, or implies that the element is absorbing
power. However, if or , as in Fig. 1.8(b), the element
is releasing or supplying power.
Passive sign convention is satisfied when the current enters through
the positive terminal of an element and p vi. If the current enters
through the negative terminal, p vi.
Unless otherwise stated, we will follow the passive sign conven-
tion throughout this text. For example, the element in both circuits of
Fig. 1.9 has an absorbing power of W because a positive current
enters the positive terminal in both cases. In Fig. 1.10, however, the
element is supplying power of W because a positive current enters
the negative terminal. Of course, an absorbing power of W is
equivalent to a supplying power of W. In general,
Power absorbed  Power supplied
12
12
12
12




vi 6 0
p  vi
vi 7 0
p  vi


p  vi
p 
dw
dt

dw
dq

dq
dt
 vi
p 
¢ dw
dt
p = +vi
(a)
v
+
−
p = −vi
(b)
v
+
−
i
i
Figure 1.8
Reference polarities for power using the
passive sign convention: (a) absorbing
power, (b) supplying power.
When the voltage and current directions
conform to Fig. 1.8(b), we have the ac-
tive sign convention and p vi.


(a)
4 V
3 A
(a)
+
−
3 A
4 V
3 A
(b)
+
−
Figure 1.9
Two cases of an element with an absorbing
power of 12 W: (a) W,
(b) W.
p  4  3  12
p  4  3  12
3 A
(a)
4 V
3 A
(a)
+
−
3 A
4 V
3 A
(b)
+
−
Figure 1.10
Two cases of an element with a supplying
power of 12 W: (a)
W, (b) W.
p  4  3  12
12
p  4  3 
ale29559_ch01.qxd 07/08/2008 10:38 AM Page 11
In fact, the law of conservation of energy must be obeyed in any
electric circuit. For this reason, the algebraic sum of power in a cir-
cuit, at any instant of time, must be zero:
(1.8)
This again confirms the fact that the total power supplied to the circuit
must balance the total power absorbed.
From Eq. (1.6), the energy absorbed or supplied by an element
from time to time t is
(1.9)
Energy is the capacity to do work, measured in joules (J).
The electric power utility companies measure energy in watt-hours
(Wh), where
1 Wh  3,600 J
w  
t
t0
p dt  
t
t0
vi dt
t0
a p  0
12 Chapter 1 Basic Concepts
An energy source forces a constant current of 2 A for 10 s to flow
through a lightbulb. If 2.3 kJ is given off in the form of light and heat
energy, calculate the voltage drop across the bulb.
Solution:
The total charge is
The voltage drop is
v 
¢w
¢q

2.3  103
20
 115 V
¢q  i ¢t  2  10  20 C
Example 1.4
To move charge q from point a to point b requires J. Find the
voltage drop if: (a) C, (b) C.
Answer: (a) V, (b) 5 V.
15
q  6
q  2
vab
30
Practice Problem 1.4
Find the power delivered to an element at ms if the current enter-
ing its positive terminal is
and the voltage is: (a) , (b) .
v  3 didt
v  3i
i  5 cos 60pt A
t  3
Example 1.5
ale29559_ch01.qxd 07/08/2008 10:38 AM Page 12
1.5 Power and Energy 13
Solution:
(a) The voltage is hence, the power is
At ms,
(b) We find the voltage and the power as
At ms,
 14137.167 sin 32.4 cos 32.4  6.396 kW
p  4500p sin 0.18p cos 0.18p W
t  3
p  vi  4500p sin 60pt cos 60pt W
v  3
di
dt
 3(60p)5 sin 60pt  900p sin 60pt V
p  75 cos2
(60p  3  103
)  75 cos2
0.18p  53.48 W
t  3
p  vi  75 cos2
60pt W
v  3i  15 cos 60pt;
Practice Problem 1.5
Find the power delivered to the element in Example 1.5 at ms
if the current remains the same but the voltage is: (a) V,
(b) V.
Answer: (a) 17.27 W, (b) 29.7 W.
v  a10  5
t
0
i dtb
v  2i
t  5
Example 1.6
How much energy does a 100-W electric bulb consume in two hours?
Solution:
This is the same as
w  pt  100 W  2 h  200 Wh
 720,000 J  720 kJ
w  pt  100 (W)  2 (h)  60 (min/h)  60 (s/min)
Practice Problem 1.6
A stove element draws 15 A when connected to a 240-V line. How
long does it take to consume 60 kJ?
Answer: 16.667 s.
ale29559_ch01.qxd 07/08/2008 10:38 AM Page 13
14 Chapter 1 Basic Concepts
Smithsonian Institution.
Historical
1884 Exhibition In the United States, nothing promoted the future
of electricity like the 1884 International Electrical Exhibition. Just
imagine a world without electricity, a world illuminated by candles and
gaslights, a world where the most common transportation was by walk-
ing and riding on horseback or by horse-drawn carriage. Into this world
an exhibition was created that highlighted Thomas Edison and reflected
his highly developed ability to promote his inventions and products.
His exhibit featured spectacular lighting displays powered by an impres-
sive 100-kW “Jumbo” generator.
Edward Weston’s dynamos and lamps were featured in the United
States Electric Lighting Company’s display. Weston’s well known col-
lection of scientific instruments was also shown.
Other prominent exhibitors included Frank Sprague, Elihu Thompson,
and the Brush Electric Company of Cleveland. The American Institute
of Electrical Engineers (AIEE) held its first technical meeting on Octo-
ber 7–8 at the Franklin Institute during the exhibit. AIEE merged with
the Institute of Radio Engineers (IRE) in 1964 to form the Institute of
Electrical and Electronics Engineers (IEEE).
ale29559_ch01.qxd 07/08/2008 10:38 AM Page 14
Circuit Elements
As we discussed in Section 1.1, an element is the basic building block
of a circuit. An electric circuit is simply an interconnection of the ele-
ments. Circuit analysis is the process of determining voltages across
(or the currents through) the elements of the circuit.
There are two types of elements found in electric circuits: pas-
sive elements and active elements. An active element is capable of
generating energy while a passive element is not. Examples of pas-
sive elements are resistors, capacitors, and inductors. Typical active
elements include generators, batteries, and operational amplifiers. Our
aim in this section is to gain familiarity with some important active
elements.
The most important active elements are voltage or current
sources that generally deliver power to the circuit connected to
them. There are two kinds of sources: independent and dependent
sources.
An ideal independent source is an active element that provides a
specified voltage or current that is completely independent of other
circuit elements.
In other words, an ideal independent voltage source delivers to the
circuit whatever current is necessary to maintain its terminal volt-
age. Physical sources such as batteries and generators may be
regarded as approximations to ideal voltage sources. Figure 1.11
shows the symbols for independent voltage sources. Notice that both
symbols in Fig. 1.11(a) and (b) can be used to represent a dc volt-
age source, but only the symbol in Fig. 1.11(a) can be used for a
time-varying voltage source. Similarly, an ideal independent current
source is an active element that provides a specified current com-
pletely independent of the voltage across the source. That is, the cur-
rent source delivers to the circuit whatever voltage is necessary to
maintain the designated current. The symbol for an independent cur-
rent source is displayed in Fig. 1.12, where the arrow indicates the
direction of current i.
An ideal dependent (or controlled) source is an active element in
which the source quantity is controlled by another voltage or current.
Dependent sources are usually designated by diamond-shaped symbols,
as shown in Fig. 1.13. Since the control of the dependent source is
achieved by a voltage or current of some other element in the circuit,
and the source can be voltage or current, it follows that there are four
possible types of dependent sources, namely:
1. A voltage-controlled voltage source (VCVS).
2. A current-controlled voltage source (CCVS).
3. A voltage-controlled current source (VCCS).
4. A current-controlled current source (CCCS).
1.6
1.6 Circuit Elements 15
Figure 1.11
Symbols for independent voltage sources:
(a) used for constant or time-varying volt-
age, (b) used for constant voltage (dc).
V
(b)
+
−
v
(a)
+
−
i
Figure 1.12
Symbol for independent current source.
(a) (b)
v +
− i
Figure 1.13
Symbols for: (a) dependent voltage
source, (b) dependent current source.
ale29559_ch01.qxd 07/08/2008 10:38 AM Page 15
Dependent sources are useful in modeling elements such as transis-
tors, operational amplifiers, and integrated circuits. An example of a
current-controlled voltage source is shown on the right-hand side of
Fig. 1.14, where the voltage of the voltage source depends on
the current i through element C. Students might be surprised that
the value of the dependent voltage source is V (and not A)
because it is a voltage source. The key idea to keep in mind is
that a voltage source comes with polarities in its symbol,
while a current source comes with an arrow, irrespective of what it
depends on.
It should be noted that an ideal voltage source (dependent or inde-
pendent) will produce any current required to ensure that the terminal
voltage is as stated, whereas an ideal current source will produce the
necessary voltage to ensure the stated current flow. Thus, an ideal
source could in theory supply an infinite amount of energy. It should
also be noted that not only do sources supply power to a circuit, they
can absorb power from a circuit too. For a voltage source, we know
the voltage but not the current supplied or drawn by it. By the same
token, we know the current supplied by a current source but not the
voltage across it.
( )
10i
10i
10i
16 Chapter 1 Basic Concepts
Calculate the power supplied or absorbed by each element in Fig. 1.15.
Solution:
We apply the sign convention for power shown in Figs. 1.8 and 1.9.
For , the 5-A current is out of the positive terminal (or into the
negative terminal); hence,
For and , the current flows into the positive terminal of the ele-
ment in each case.
For , we should note that the voltage is 8 V (positive at the top), the
same as the voltage for since both the passive element and the
dependent source are connected to the same terminals. (Remember that
voltage is always measured across an element in a circuit.) Since the
current flows out of the positive terminal,
We should observe that the 20-V independent voltage source and
dependent current source are supplying power to the rest of
the network, while the two passive elements are absorbing power.
Also,
In agreement with Eq. (1.8), the total power supplied equals the total
power absorbed.
p1  p2  p3  p4  100  60  48  8  0
0.2I
p4  8(0.2I)  8(0.2  5)  8 W Supplied power
p3,
p4
p3  8(6)  48 W Absorbed power
p2  12(5)  60 W Absorbed power
p3
p2
p1  20(5)  100 W Supplied power
p1
Example 1.7
p2
p3
I = 5 A
20 V
6 A
8 V 0.2I
12 V
+
−
+
−
+ −
p1 p4
Figure 1.15
For Example 1.7.
i
A B
C 10i
5 V
+
−
+
−
Figure 1.14
The source on the right-hand side is a
current-controlled voltage source.
ale29559_ch01.qxd 07/08/2008 10:38 AM Page 16
Applications2
In this section, we will consider two practical applications of the
concepts developed in this chapter. The first one deals with the TV
picture tube and the other with how electric utilities determine your
electric bill.
1.7.1 TV Picture Tube
One important application of the motion of electrons is found in both
the transmission and reception of TV signals. At the transmission end,
a TV camera reduces a scene from an optical image to an electrical
signal. Scanning is accomplished with a thin beam of electrons in an
iconoscope camera tube.
At the receiving end, the image is reconstructed by using a cathode-
ray tube (CRT) located in the TV receiver.3
The CRT is depicted in
Fig. 1.17. Unlike the iconoscope tube, which produces an electron
beam of constant intensity, the CRT beam varies in intensity accord-
ing to the incoming signal. The electron gun, maintained at a high
potential, fires the electron beam. The beam passes through two sets
of plates for vertical and horizontal deflections so that the spot on the
screen where the beam strikes can move right and left and up and
down. When the electron beam strikes the fluorescent screen, it gives
off light at that spot. Thus, the beam can be made to “paint” a picture
on the TV screen.
1.7
1.7 Applications 17
Compute the power absorbed or supplied by each component of the
circuit in Fig. 1.16.
Answer: W, W, W, W.
p4  15
p3  9
p2  16
p1  40
8 A
5 V 3 V
2 V
3 A
I = 5 A
0.6I
+
−
+ −
+
−
+
−
+
−
p2
p1 p3 p4
Figure 1.16
For Practice Prob. 1.7.
2
The dagger sign preceding a section heading indicates the section that may be skipped,
explained briefly, or assigned as homework.
3
Modern TV tubes use a different technology.
Vertical
deflection
plates
Horizontal
deflection
plates
Electron
trajectory
Bright spot on
fluorescent screen
Electron gun
Figure 1.17
Cathode-ray tube.
D. E. Tilley, Contemporary College Physics Menlo Park, CA: Benjamin/
Cummings, 1979, p. 319.
Practice Problem 1.7
ale29559_ch01.qxd 07/08/2008 10:38 AM Page 17
Fundamentals_of_Electric_Circuits_4th_Ed.pdf
Fundamentals_of_Electric_Circuits_4th_Ed.pdf
Fundamentals_of_Electric_Circuits_4th_Ed.pdf
Fundamentals_of_Electric_Circuits_4th_Ed.pdf
Fundamentals_of_Electric_Circuits_4th_Ed.pdf
Fundamentals_of_Electric_Circuits_4th_Ed.pdf
Fundamentals_of_Electric_Circuits_4th_Ed.pdf
Fundamentals_of_Electric_Circuits_4th_Ed.pdf
Fundamentals_of_Electric_Circuits_4th_Ed.pdf
Fundamentals_of_Electric_Circuits_4th_Ed.pdf
Fundamentals_of_Electric_Circuits_4th_Ed.pdf
Fundamentals_of_Electric_Circuits_4th_Ed.pdf
Fundamentals_of_Electric_Circuits_4th_Ed.pdf
Fundamentals_of_Electric_Circuits_4th_Ed.pdf
Fundamentals_of_Electric_Circuits_4th_Ed.pdf
Fundamentals_of_Electric_Circuits_4th_Ed.pdf
Fundamentals_of_Electric_Circuits_4th_Ed.pdf
Fundamentals_of_Electric_Circuits_4th_Ed.pdf
Fundamentals_of_Electric_Circuits_4th_Ed.pdf
Fundamentals_of_Electric_Circuits_4th_Ed.pdf
Fundamentals_of_Electric_Circuits_4th_Ed.pdf
Fundamentals_of_Electric_Circuits_4th_Ed.pdf
Fundamentals_of_Electric_Circuits_4th_Ed.pdf
Fundamentals_of_Electric_Circuits_4th_Ed.pdf
Fundamentals_of_Electric_Circuits_4th_Ed.pdf
Fundamentals_of_Electric_Circuits_4th_Ed.pdf
Fundamentals_of_Electric_Circuits_4th_Ed.pdf
Fundamentals_of_Electric_Circuits_4th_Ed.pdf
Fundamentals_of_Electric_Circuits_4th_Ed.pdf
Fundamentals_of_Electric_Circuits_4th_Ed.pdf
Fundamentals_of_Electric_Circuits_4th_Ed.pdf
Fundamentals_of_Electric_Circuits_4th_Ed.pdf
Fundamentals_of_Electric_Circuits_4th_Ed.pdf
Fundamentals_of_Electric_Circuits_4th_Ed.pdf
Fundamentals_of_Electric_Circuits_4th_Ed.pdf
Fundamentals_of_Electric_Circuits_4th_Ed.pdf
Fundamentals_of_Electric_Circuits_4th_Ed.pdf
Fundamentals_of_Electric_Circuits_4th_Ed.pdf
Fundamentals_of_Electric_Circuits_4th_Ed.pdf
Fundamentals_of_Electric_Circuits_4th_Ed.pdf
Fundamentals_of_Electric_Circuits_4th_Ed.pdf
Fundamentals_of_Electric_Circuits_4th_Ed.pdf
Fundamentals_of_Electric_Circuits_4th_Ed.pdf
Fundamentals_of_Electric_Circuits_4th_Ed.pdf
Fundamentals_of_Electric_Circuits_4th_Ed.pdf
Fundamentals_of_Electric_Circuits_4th_Ed.pdf
Fundamentals_of_Electric_Circuits_4th_Ed.pdf
Fundamentals_of_Electric_Circuits_4th_Ed.pdf
Fundamentals_of_Electric_Circuits_4th_Ed.pdf
Fundamentals_of_Electric_Circuits_4th_Ed.pdf
Fundamentals_of_Electric_Circuits_4th_Ed.pdf
Fundamentals_of_Electric_Circuits_4th_Ed.pdf
Fundamentals_of_Electric_Circuits_4th_Ed.pdf
Fundamentals_of_Electric_Circuits_4th_Ed.pdf
Fundamentals_of_Electric_Circuits_4th_Ed.pdf
Fundamentals_of_Electric_Circuits_4th_Ed.pdf
Fundamentals_of_Electric_Circuits_4th_Ed.pdf
Fundamentals_of_Electric_Circuits_4th_Ed.pdf
Fundamentals_of_Electric_Circuits_4th_Ed.pdf
Fundamentals_of_Electric_Circuits_4th_Ed.pdf
Fundamentals_of_Electric_Circuits_4th_Ed.pdf
Fundamentals_of_Electric_Circuits_4th_Ed.pdf
Fundamentals_of_Electric_Circuits_4th_Ed.pdf
Fundamentals_of_Electric_Circuits_4th_Ed.pdf
Fundamentals_of_Electric_Circuits_4th_Ed.pdf
Fundamentals_of_Electric_Circuits_4th_Ed.pdf
Fundamentals_of_Electric_Circuits_4th_Ed.pdf
Fundamentals_of_Electric_Circuits_4th_Ed.pdf
Fundamentals_of_Electric_Circuits_4th_Ed.pdf
Fundamentals_of_Electric_Circuits_4th_Ed.pdf
Fundamentals_of_Electric_Circuits_4th_Ed.pdf
Fundamentals_of_Electric_Circuits_4th_Ed.pdf
Fundamentals_of_Electric_Circuits_4th_Ed.pdf
Fundamentals_of_Electric_Circuits_4th_Ed.pdf
Fundamentals_of_Electric_Circuits_4th_Ed.pdf
Fundamentals_of_Electric_Circuits_4th_Ed.pdf
Fundamentals_of_Electric_Circuits_4th_Ed.pdf
Fundamentals_of_Electric_Circuits_4th_Ed.pdf
Fundamentals_of_Electric_Circuits_4th_Ed.pdf
Fundamentals_of_Electric_Circuits_4th_Ed.pdf
Fundamentals_of_Electric_Circuits_4th_Ed.pdf
Fundamentals_of_Electric_Circuits_4th_Ed.pdf
Fundamentals_of_Electric_Circuits_4th_Ed.pdf
Fundamentals_of_Electric_Circuits_4th_Ed.pdf
Fundamentals_of_Electric_Circuits_4th_Ed.pdf
Fundamentals_of_Electric_Circuits_4th_Ed.pdf
Fundamentals_of_Electric_Circuits_4th_Ed.pdf
Fundamentals_of_Electric_Circuits_4th_Ed.pdf
Fundamentals_of_Electric_Circuits_4th_Ed.pdf
Fundamentals_of_Electric_Circuits_4th_Ed.pdf
Fundamentals_of_Electric_Circuits_4th_Ed.pdf
Fundamentals_of_Electric_Circuits_4th_Ed.pdf
Fundamentals_of_Electric_Circuits_4th_Ed.pdf
Fundamentals_of_Electric_Circuits_4th_Ed.pdf
Fundamentals_of_Electric_Circuits_4th_Ed.pdf
Fundamentals_of_Electric_Circuits_4th_Ed.pdf
Fundamentals_of_Electric_Circuits_4th_Ed.pdf
Fundamentals_of_Electric_Circuits_4th_Ed.pdf
Fundamentals_of_Electric_Circuits_4th_Ed.pdf
Fundamentals_of_Electric_Circuits_4th_Ed.pdf
Fundamentals_of_Electric_Circuits_4th_Ed.pdf
Fundamentals_of_Electric_Circuits_4th_Ed.pdf
Fundamentals_of_Electric_Circuits_4th_Ed.pdf
Fundamentals_of_Electric_Circuits_4th_Ed.pdf
Fundamentals_of_Electric_Circuits_4th_Ed.pdf
Fundamentals_of_Electric_Circuits_4th_Ed.pdf
Fundamentals_of_Electric_Circuits_4th_Ed.pdf
Fundamentals_of_Electric_Circuits_4th_Ed.pdf
Fundamentals_of_Electric_Circuits_4th_Ed.pdf
Fundamentals_of_Electric_Circuits_4th_Ed.pdf
Fundamentals_of_Electric_Circuits_4th_Ed.pdf
Fundamentals_of_Electric_Circuits_4th_Ed.pdf
Fundamentals_of_Electric_Circuits_4th_Ed.pdf
Fundamentals_of_Electric_Circuits_4th_Ed.pdf
Fundamentals_of_Electric_Circuits_4th_Ed.pdf
Fundamentals_of_Electric_Circuits_4th_Ed.pdf
Fundamentals_of_Electric_Circuits_4th_Ed.pdf
Fundamentals_of_Electric_Circuits_4th_Ed.pdf
Fundamentals_of_Electric_Circuits_4th_Ed.pdf
Fundamentals_of_Electric_Circuits_4th_Ed.pdf
Fundamentals_of_Electric_Circuits_4th_Ed.pdf
Fundamentals_of_Electric_Circuits_4th_Ed.pdf
Fundamentals_of_Electric_Circuits_4th_Ed.pdf
Fundamentals_of_Electric_Circuits_4th_Ed.pdf
Fundamentals_of_Electric_Circuits_4th_Ed.pdf
Fundamentals_of_Electric_Circuits_4th_Ed.pdf
Fundamentals_of_Electric_Circuits_4th_Ed.pdf
Fundamentals_of_Electric_Circuits_4th_Ed.pdf
Fundamentals_of_Electric_Circuits_4th_Ed.pdf
Fundamentals_of_Electric_Circuits_4th_Ed.pdf
Fundamentals_of_Electric_Circuits_4th_Ed.pdf
Fundamentals_of_Electric_Circuits_4th_Ed.pdf
Fundamentals_of_Electric_Circuits_4th_Ed.pdf
Fundamentals_of_Electric_Circuits_4th_Ed.pdf
Fundamentals_of_Electric_Circuits_4th_Ed.pdf
Fundamentals_of_Electric_Circuits_4th_Ed.pdf
Fundamentals_of_Electric_Circuits_4th_Ed.pdf
Fundamentals_of_Electric_Circuits_4th_Ed.pdf
Fundamentals_of_Electric_Circuits_4th_Ed.pdf
Fundamentals_of_Electric_Circuits_4th_Ed.pdf
Fundamentals_of_Electric_Circuits_4th_Ed.pdf
Fundamentals_of_Electric_Circuits_4th_Ed.pdf
Fundamentals_of_Electric_Circuits_4th_Ed.pdf
Fundamentals_of_Electric_Circuits_4th_Ed.pdf
Fundamentals_of_Electric_Circuits_4th_Ed.pdf
Fundamentals_of_Electric_Circuits_4th_Ed.pdf
Fundamentals_of_Electric_Circuits_4th_Ed.pdf
Fundamentals_of_Electric_Circuits_4th_Ed.pdf
Fundamentals_of_Electric_Circuits_4th_Ed.pdf
Fundamentals_of_Electric_Circuits_4th_Ed.pdf
Fundamentals_of_Electric_Circuits_4th_Ed.pdf
Fundamentals_of_Electric_Circuits_4th_Ed.pdf
Fundamentals_of_Electric_Circuits_4th_Ed.pdf
Fundamentals_of_Electric_Circuits_4th_Ed.pdf
Fundamentals_of_Electric_Circuits_4th_Ed.pdf
Fundamentals_of_Electric_Circuits_4th_Ed.pdf
Fundamentals_of_Electric_Circuits_4th_Ed.pdf
Fundamentals_of_Electric_Circuits_4th_Ed.pdf
Fundamentals_of_Electric_Circuits_4th_Ed.pdf
Fundamentals_of_Electric_Circuits_4th_Ed.pdf
Fundamentals_of_Electric_Circuits_4th_Ed.pdf
Fundamentals_of_Electric_Circuits_4th_Ed.pdf
Fundamentals_of_Electric_Circuits_4th_Ed.pdf
Fundamentals_of_Electric_Circuits_4th_Ed.pdf
Fundamentals_of_Electric_Circuits_4th_Ed.pdf
Fundamentals_of_Electric_Circuits_4th_Ed.pdf
Fundamentals_of_Electric_Circuits_4th_Ed.pdf
Fundamentals_of_Electric_Circuits_4th_Ed.pdf
Fundamentals_of_Electric_Circuits_4th_Ed.pdf
Fundamentals_of_Electric_Circuits_4th_Ed.pdf
Fundamentals_of_Electric_Circuits_4th_Ed.pdf
Fundamentals_of_Electric_Circuits_4th_Ed.pdf
Fundamentals_of_Electric_Circuits_4th_Ed.pdf
Fundamentals_of_Electric_Circuits_4th_Ed.pdf
Fundamentals_of_Electric_Circuits_4th_Ed.pdf
Fundamentals_of_Electric_Circuits_4th_Ed.pdf
Fundamentals_of_Electric_Circuits_4th_Ed.pdf
Fundamentals_of_Electric_Circuits_4th_Ed.pdf
Fundamentals_of_Electric_Circuits_4th_Ed.pdf
Fundamentals_of_Electric_Circuits_4th_Ed.pdf
Fundamentals_of_Electric_Circuits_4th_Ed.pdf
Fundamentals_of_Electric_Circuits_4th_Ed.pdf
Fundamentals_of_Electric_Circuits_4th_Ed.pdf
Fundamentals_of_Electric_Circuits_4th_Ed.pdf
Fundamentals_of_Electric_Circuits_4th_Ed.pdf
Fundamentals_of_Electric_Circuits_4th_Ed.pdf
Fundamentals_of_Electric_Circuits_4th_Ed.pdf
Fundamentals_of_Electric_Circuits_4th_Ed.pdf
Fundamentals_of_Electric_Circuits_4th_Ed.pdf
Fundamentals_of_Electric_Circuits_4th_Ed.pdf
Fundamentals_of_Electric_Circuits_4th_Ed.pdf
Fundamentals_of_Electric_Circuits_4th_Ed.pdf
Fundamentals_of_Electric_Circuits_4th_Ed.pdf
Fundamentals_of_Electric_Circuits_4th_Ed.pdf
Fundamentals_of_Electric_Circuits_4th_Ed.pdf
Fundamentals_of_Electric_Circuits_4th_Ed.pdf
Fundamentals_of_Electric_Circuits_4th_Ed.pdf
Fundamentals_of_Electric_Circuits_4th_Ed.pdf
Fundamentals_of_Electric_Circuits_4th_Ed.pdf
Fundamentals_of_Electric_Circuits_4th_Ed.pdf
Fundamentals_of_Electric_Circuits_4th_Ed.pdf
Fundamentals_of_Electric_Circuits_4th_Ed.pdf
Fundamentals_of_Electric_Circuits_4th_Ed.pdf
Fundamentals_of_Electric_Circuits_4th_Ed.pdf
Fundamentals_of_Electric_Circuits_4th_Ed.pdf
Fundamentals_of_Electric_Circuits_4th_Ed.pdf
Fundamentals_of_Electric_Circuits_4th_Ed.pdf
Fundamentals_of_Electric_Circuits_4th_Ed.pdf
Fundamentals_of_Electric_Circuits_4th_Ed.pdf
Fundamentals_of_Electric_Circuits_4th_Ed.pdf
Fundamentals_of_Electric_Circuits_4th_Ed.pdf
Fundamentals_of_Electric_Circuits_4th_Ed.pdf
Fundamentals_of_Electric_Circuits_4th_Ed.pdf
Fundamentals_of_Electric_Circuits_4th_Ed.pdf
Fundamentals_of_Electric_Circuits_4th_Ed.pdf
Fundamentals_of_Electric_Circuits_4th_Ed.pdf
Fundamentals_of_Electric_Circuits_4th_Ed.pdf
Fundamentals_of_Electric_Circuits_4th_Ed.pdf
Fundamentals_of_Electric_Circuits_4th_Ed.pdf
Fundamentals_of_Electric_Circuits_4th_Ed.pdf
Fundamentals_of_Electric_Circuits_4th_Ed.pdf
Fundamentals_of_Electric_Circuits_4th_Ed.pdf
Fundamentals_of_Electric_Circuits_4th_Ed.pdf
Fundamentals_of_Electric_Circuits_4th_Ed.pdf
Fundamentals_of_Electric_Circuits_4th_Ed.pdf
Fundamentals_of_Electric_Circuits_4th_Ed.pdf
Fundamentals_of_Electric_Circuits_4th_Ed.pdf
Fundamentals_of_Electric_Circuits_4th_Ed.pdf
Fundamentals_of_Electric_Circuits_4th_Ed.pdf
Fundamentals_of_Electric_Circuits_4th_Ed.pdf
Fundamentals_of_Electric_Circuits_4th_Ed.pdf
Fundamentals_of_Electric_Circuits_4th_Ed.pdf
Fundamentals_of_Electric_Circuits_4th_Ed.pdf
Fundamentals_of_Electric_Circuits_4th_Ed.pdf
Fundamentals_of_Electric_Circuits_4th_Ed.pdf
Fundamentals_of_Electric_Circuits_4th_Ed.pdf
Fundamentals_of_Electric_Circuits_4th_Ed.pdf
Fundamentals_of_Electric_Circuits_4th_Ed.pdf
Fundamentals_of_Electric_Circuits_4th_Ed.pdf
Fundamentals_of_Electric_Circuits_4th_Ed.pdf
Fundamentals_of_Electric_Circuits_4th_Ed.pdf
Fundamentals_of_Electric_Circuits_4th_Ed.pdf
Fundamentals_of_Electric_Circuits_4th_Ed.pdf
Fundamentals_of_Electric_Circuits_4th_Ed.pdf
Fundamentals_of_Electric_Circuits_4th_Ed.pdf
Fundamentals_of_Electric_Circuits_4th_Ed.pdf
Fundamentals_of_Electric_Circuits_4th_Ed.pdf
Fundamentals_of_Electric_Circuits_4th_Ed.pdf
Fundamentals_of_Electric_Circuits_4th_Ed.pdf
Fundamentals_of_Electric_Circuits_4th_Ed.pdf
Fundamentals_of_Electric_Circuits_4th_Ed.pdf
Fundamentals_of_Electric_Circuits_4th_Ed.pdf
Fundamentals_of_Electric_Circuits_4th_Ed.pdf
Fundamentals_of_Electric_Circuits_4th_Ed.pdf
Fundamentals_of_Electric_Circuits_4th_Ed.pdf
Fundamentals_of_Electric_Circuits_4th_Ed.pdf
Fundamentals_of_Electric_Circuits_4th_Ed.pdf
Fundamentals_of_Electric_Circuits_4th_Ed.pdf
Fundamentals_of_Electric_Circuits_4th_Ed.pdf
Fundamentals_of_Electric_Circuits_4th_Ed.pdf
Fundamentals_of_Electric_Circuits_4th_Ed.pdf
Fundamentals_of_Electric_Circuits_4th_Ed.pdf
Fundamentals_of_Electric_Circuits_4th_Ed.pdf
Fundamentals_of_Electric_Circuits_4th_Ed.pdf
Fundamentals_of_Electric_Circuits_4th_Ed.pdf
Fundamentals_of_Electric_Circuits_4th_Ed.pdf
Fundamentals_of_Electric_Circuits_4th_Ed.pdf
Fundamentals_of_Electric_Circuits_4th_Ed.pdf
Fundamentals_of_Electric_Circuits_4th_Ed.pdf
Fundamentals_of_Electric_Circuits_4th_Ed.pdf
Fundamentals_of_Electric_Circuits_4th_Ed.pdf
Fundamentals_of_Electric_Circuits_4th_Ed.pdf
Fundamentals_of_Electric_Circuits_4th_Ed.pdf
Fundamentals_of_Electric_Circuits_4th_Ed.pdf
Fundamentals_of_Electric_Circuits_4th_Ed.pdf
Fundamentals_of_Electric_Circuits_4th_Ed.pdf
Fundamentals_of_Electric_Circuits_4th_Ed.pdf
Fundamentals_of_Electric_Circuits_4th_Ed.pdf
Fundamentals_of_Electric_Circuits_4th_Ed.pdf
Fundamentals_of_Electric_Circuits_4th_Ed.pdf
Fundamentals_of_Electric_Circuits_4th_Ed.pdf
Fundamentals_of_Electric_Circuits_4th_Ed.pdf
Fundamentals_of_Electric_Circuits_4th_Ed.pdf
Fundamentals_of_Electric_Circuits_4th_Ed.pdf
Fundamentals_of_Electric_Circuits_4th_Ed.pdf
Fundamentals_of_Electric_Circuits_4th_Ed.pdf
Fundamentals_of_Electric_Circuits_4th_Ed.pdf
Fundamentals_of_Electric_Circuits_4th_Ed.pdf
Fundamentals_of_Electric_Circuits_4th_Ed.pdf
Fundamentals_of_Electric_Circuits_4th_Ed.pdf
Fundamentals_of_Electric_Circuits_4th_Ed.pdf
Fundamentals_of_Electric_Circuits_4th_Ed.pdf
Fundamentals_of_Electric_Circuits_4th_Ed.pdf
Fundamentals_of_Electric_Circuits_4th_Ed.pdf
Fundamentals_of_Electric_Circuits_4th_Ed.pdf
Fundamentals_of_Electric_Circuits_4th_Ed.pdf
Fundamentals_of_Electric_Circuits_4th_Ed.pdf
Fundamentals_of_Electric_Circuits_4th_Ed.pdf
Fundamentals_of_Electric_Circuits_4th_Ed.pdf
Fundamentals_of_Electric_Circuits_4th_Ed.pdf
Fundamentals_of_Electric_Circuits_4th_Ed.pdf
Fundamentals_of_Electric_Circuits_4th_Ed.pdf
Fundamentals_of_Electric_Circuits_4th_Ed.pdf
Fundamentals_of_Electric_Circuits_4th_Ed.pdf
Fundamentals_of_Electric_Circuits_4th_Ed.pdf
Fundamentals_of_Electric_Circuits_4th_Ed.pdf
Fundamentals_of_Electric_Circuits_4th_Ed.pdf
Fundamentals_of_Electric_Circuits_4th_Ed.pdf
Fundamentals_of_Electric_Circuits_4th_Ed.pdf
Fundamentals_of_Electric_Circuits_4th_Ed.pdf
Fundamentals_of_Electric_Circuits_4th_Ed.pdf
Fundamentals_of_Electric_Circuits_4th_Ed.pdf
Fundamentals_of_Electric_Circuits_4th_Ed.pdf
Fundamentals_of_Electric_Circuits_4th_Ed.pdf
Fundamentals_of_Electric_Circuits_4th_Ed.pdf
Fundamentals_of_Electric_Circuits_4th_Ed.pdf
Fundamentals_of_Electric_Circuits_4th_Ed.pdf
Fundamentals_of_Electric_Circuits_4th_Ed.pdf
Fundamentals_of_Electric_Circuits_4th_Ed.pdf
Fundamentals_of_Electric_Circuits_4th_Ed.pdf
Fundamentals_of_Electric_Circuits_4th_Ed.pdf
Fundamentals_of_Electric_Circuits_4th_Ed.pdf
Fundamentals_of_Electric_Circuits_4th_Ed.pdf
Fundamentals_of_Electric_Circuits_4th_Ed.pdf
Fundamentals_of_Electric_Circuits_4th_Ed.pdf
Fundamentals_of_Electric_Circuits_4th_Ed.pdf
Fundamentals_of_Electric_Circuits_4th_Ed.pdf
Fundamentals_of_Electric_Circuits_4th_Ed.pdf
Fundamentals_of_Electric_Circuits_4th_Ed.pdf
Fundamentals_of_Electric_Circuits_4th_Ed.pdf
Fundamentals_of_Electric_Circuits_4th_Ed.pdf
Fundamentals_of_Electric_Circuits_4th_Ed.pdf
Fundamentals_of_Electric_Circuits_4th_Ed.pdf
Fundamentals_of_Electric_Circuits_4th_Ed.pdf
Fundamentals_of_Electric_Circuits_4th_Ed.pdf
Fundamentals_of_Electric_Circuits_4th_Ed.pdf
Fundamentals_of_Electric_Circuits_4th_Ed.pdf
Fundamentals_of_Electric_Circuits_4th_Ed.pdf
Fundamentals_of_Electric_Circuits_4th_Ed.pdf
Fundamentals_of_Electric_Circuits_4th_Ed.pdf
Fundamentals_of_Electric_Circuits_4th_Ed.pdf
Fundamentals_of_Electric_Circuits_4th_Ed.pdf
Fundamentals_of_Electric_Circuits_4th_Ed.pdf
Fundamentals_of_Electric_Circuits_4th_Ed.pdf
Fundamentals_of_Electric_Circuits_4th_Ed.pdf
Fundamentals_of_Electric_Circuits_4th_Ed.pdf
Fundamentals_of_Electric_Circuits_4th_Ed.pdf
Fundamentals_of_Electric_Circuits_4th_Ed.pdf
Fundamentals_of_Electric_Circuits_4th_Ed.pdf
Fundamentals_of_Electric_Circuits_4th_Ed.pdf
Fundamentals_of_Electric_Circuits_4th_Ed.pdf
Fundamentals_of_Electric_Circuits_4th_Ed.pdf
Fundamentals_of_Electric_Circuits_4th_Ed.pdf
Fundamentals_of_Electric_Circuits_4th_Ed.pdf
Fundamentals_of_Electric_Circuits_4th_Ed.pdf
Fundamentals_of_Electric_Circuits_4th_Ed.pdf
Fundamentals_of_Electric_Circuits_4th_Ed.pdf
Fundamentals_of_Electric_Circuits_4th_Ed.pdf
Fundamentals_of_Electric_Circuits_4th_Ed.pdf
Fundamentals_of_Electric_Circuits_4th_Ed.pdf
Fundamentals_of_Electric_Circuits_4th_Ed.pdf
Fundamentals_of_Electric_Circuits_4th_Ed.pdf
Fundamentals_of_Electric_Circuits_4th_Ed.pdf
Fundamentals_of_Electric_Circuits_4th_Ed.pdf
Fundamentals_of_Electric_Circuits_4th_Ed.pdf
Fundamentals_of_Electric_Circuits_4th_Ed.pdf
Fundamentals_of_Electric_Circuits_4th_Ed.pdf
Fundamentals_of_Electric_Circuits_4th_Ed.pdf
Fundamentals_of_Electric_Circuits_4th_Ed.pdf
Fundamentals_of_Electric_Circuits_4th_Ed.pdf
Fundamentals_of_Electric_Circuits_4th_Ed.pdf
Fundamentals_of_Electric_Circuits_4th_Ed.pdf
Fundamentals_of_Electric_Circuits_4th_Ed.pdf
Fundamentals_of_Electric_Circuits_4th_Ed.pdf
Fundamentals_of_Electric_Circuits_4th_Ed.pdf
Fundamentals_of_Electric_Circuits_4th_Ed.pdf
Fundamentals_of_Electric_Circuits_4th_Ed.pdf
Fundamentals_of_Electric_Circuits_4th_Ed.pdf
Fundamentals_of_Electric_Circuits_4th_Ed.pdf
Fundamentals_of_Electric_Circuits_4th_Ed.pdf
Fundamentals_of_Electric_Circuits_4th_Ed.pdf
Fundamentals_of_Electric_Circuits_4th_Ed.pdf
Fundamentals_of_Electric_Circuits_4th_Ed.pdf
Fundamentals_of_Electric_Circuits_4th_Ed.pdf
Fundamentals_of_Electric_Circuits_4th_Ed.pdf
Fundamentals_of_Electric_Circuits_4th_Ed.pdf
Fundamentals_of_Electric_Circuits_4th_Ed.pdf
Fundamentals_of_Electric_Circuits_4th_Ed.pdf
Fundamentals_of_Electric_Circuits_4th_Ed.pdf
Fundamentals_of_Electric_Circuits_4th_Ed.pdf
Fundamentals_of_Electric_Circuits_4th_Ed.pdf
Fundamentals_of_Electric_Circuits_4th_Ed.pdf
Fundamentals_of_Electric_Circuits_4th_Ed.pdf
Fundamentals_of_Electric_Circuits_4th_Ed.pdf
Fundamentals_of_Electric_Circuits_4th_Ed.pdf
Fundamentals_of_Electric_Circuits_4th_Ed.pdf
Fundamentals_of_Electric_Circuits_4th_Ed.pdf
Fundamentals_of_Electric_Circuits_4th_Ed.pdf
Fundamentals_of_Electric_Circuits_4th_Ed.pdf
Fundamentals_of_Electric_Circuits_4th_Ed.pdf
Fundamentals_of_Electric_Circuits_4th_Ed.pdf
Fundamentals_of_Electric_Circuits_4th_Ed.pdf
Fundamentals_of_Electric_Circuits_4th_Ed.pdf
Fundamentals_of_Electric_Circuits_4th_Ed.pdf
Fundamentals_of_Electric_Circuits_4th_Ed.pdf
Fundamentals_of_Electric_Circuits_4th_Ed.pdf
Fundamentals_of_Electric_Circuits_4th_Ed.pdf
Fundamentals_of_Electric_Circuits_4th_Ed.pdf
Fundamentals_of_Electric_Circuits_4th_Ed.pdf
Fundamentals_of_Electric_Circuits_4th_Ed.pdf
Fundamentals_of_Electric_Circuits_4th_Ed.pdf
Fundamentals_of_Electric_Circuits_4th_Ed.pdf
Fundamentals_of_Electric_Circuits_4th_Ed.pdf
Fundamentals_of_Electric_Circuits_4th_Ed.pdf
Fundamentals_of_Electric_Circuits_4th_Ed.pdf
Fundamentals_of_Electric_Circuits_4th_Ed.pdf
Fundamentals_of_Electric_Circuits_4th_Ed.pdf
Fundamentals_of_Electric_Circuits_4th_Ed.pdf
Fundamentals_of_Electric_Circuits_4th_Ed.pdf
Fundamentals_of_Electric_Circuits_4th_Ed.pdf
Fundamentals_of_Electric_Circuits_4th_Ed.pdf
Fundamentals_of_Electric_Circuits_4th_Ed.pdf
Fundamentals_of_Electric_Circuits_4th_Ed.pdf
Fundamentals_of_Electric_Circuits_4th_Ed.pdf
Fundamentals_of_Electric_Circuits_4th_Ed.pdf
Fundamentals_of_Electric_Circuits_4th_Ed.pdf
Fundamentals_of_Electric_Circuits_4th_Ed.pdf
Fundamentals_of_Electric_Circuits_4th_Ed.pdf
Fundamentals_of_Electric_Circuits_4th_Ed.pdf
Fundamentals_of_Electric_Circuits_4th_Ed.pdf
Fundamentals_of_Electric_Circuits_4th_Ed.pdf
Fundamentals_of_Electric_Circuits_4th_Ed.pdf
Fundamentals_of_Electric_Circuits_4th_Ed.pdf
Fundamentals_of_Electric_Circuits_4th_Ed.pdf
Fundamentals_of_Electric_Circuits_4th_Ed.pdf
Fundamentals_of_Electric_Circuits_4th_Ed.pdf
Fundamentals_of_Electric_Circuits_4th_Ed.pdf
Fundamentals_of_Electric_Circuits_4th_Ed.pdf
Fundamentals_of_Electric_Circuits_4th_Ed.pdf
Fundamentals_of_Electric_Circuits_4th_Ed.pdf
Fundamentals_of_Electric_Circuits_4th_Ed.pdf
Fundamentals_of_Electric_Circuits_4th_Ed.pdf
Fundamentals_of_Electric_Circuits_4th_Ed.pdf
Fundamentals_of_Electric_Circuits_4th_Ed.pdf
Fundamentals_of_Electric_Circuits_4th_Ed.pdf
Fundamentals_of_Electric_Circuits_4th_Ed.pdf
Fundamentals_of_Electric_Circuits_4th_Ed.pdf
Fundamentals_of_Electric_Circuits_4th_Ed.pdf
Fundamentals_of_Electric_Circuits_4th_Ed.pdf
Fundamentals_of_Electric_Circuits_4th_Ed.pdf
Fundamentals_of_Electric_Circuits_4th_Ed.pdf
Fundamentals_of_Electric_Circuits_4th_Ed.pdf
Fundamentals_of_Electric_Circuits_4th_Ed.pdf
Fundamentals_of_Electric_Circuits_4th_Ed.pdf
Fundamentals_of_Electric_Circuits_4th_Ed.pdf
Fundamentals_of_Electric_Circuits_4th_Ed.pdf
Fundamentals_of_Electric_Circuits_4th_Ed.pdf
Fundamentals_of_Electric_Circuits_4th_Ed.pdf
Fundamentals_of_Electric_Circuits_4th_Ed.pdf
Fundamentals_of_Electric_Circuits_4th_Ed.pdf
Fundamentals_of_Electric_Circuits_4th_Ed.pdf
Fundamentals_of_Electric_Circuits_4th_Ed.pdf
Fundamentals_of_Electric_Circuits_4th_Ed.pdf
Fundamentals_of_Electric_Circuits_4th_Ed.pdf
Fundamentals_of_Electric_Circuits_4th_Ed.pdf
Fundamentals_of_Electric_Circuits_4th_Ed.pdf
Fundamentals_of_Electric_Circuits_4th_Ed.pdf
Fundamentals_of_Electric_Circuits_4th_Ed.pdf
Fundamentals_of_Electric_Circuits_4th_Ed.pdf
Fundamentals_of_Electric_Circuits_4th_Ed.pdf
Fundamentals_of_Electric_Circuits_4th_Ed.pdf
Fundamentals_of_Electric_Circuits_4th_Ed.pdf
Fundamentals_of_Electric_Circuits_4th_Ed.pdf
Fundamentals_of_Electric_Circuits_4th_Ed.pdf
Fundamentals_of_Electric_Circuits_4th_Ed.pdf
Fundamentals_of_Electric_Circuits_4th_Ed.pdf
Fundamentals_of_Electric_Circuits_4th_Ed.pdf
Fundamentals_of_Electric_Circuits_4th_Ed.pdf
Fundamentals_of_Electric_Circuits_4th_Ed.pdf
Fundamentals_of_Electric_Circuits_4th_Ed.pdf
Fundamentals_of_Electric_Circuits_4th_Ed.pdf
Fundamentals_of_Electric_Circuits_4th_Ed.pdf
Fundamentals_of_Electric_Circuits_4th_Ed.pdf
Fundamentals_of_Electric_Circuits_4th_Ed.pdf
Fundamentals_of_Electric_Circuits_4th_Ed.pdf
Fundamentals_of_Electric_Circuits_4th_Ed.pdf
Fundamentals_of_Electric_Circuits_4th_Ed.pdf
Fundamentals_of_Electric_Circuits_4th_Ed.pdf
Fundamentals_of_Electric_Circuits_4th_Ed.pdf
Fundamentals_of_Electric_Circuits_4th_Ed.pdf
Fundamentals_of_Electric_Circuits_4th_Ed.pdf
Fundamentals_of_Electric_Circuits_4th_Ed.pdf
Fundamentals_of_Electric_Circuits_4th_Ed.pdf
Fundamentals_of_Electric_Circuits_4th_Ed.pdf
Fundamentals_of_Electric_Circuits_4th_Ed.pdf
Fundamentals_of_Electric_Circuits_4th_Ed.pdf
Fundamentals_of_Electric_Circuits_4th_Ed.pdf
Fundamentals_of_Electric_Circuits_4th_Ed.pdf
Fundamentals_of_Electric_Circuits_4th_Ed.pdf
Fundamentals_of_Electric_Circuits_4th_Ed.pdf
Fundamentals_of_Electric_Circuits_4th_Ed.pdf
Fundamentals_of_Electric_Circuits_4th_Ed.pdf
Fundamentals_of_Electric_Circuits_4th_Ed.pdf
Fundamentals_of_Electric_Circuits_4th_Ed.pdf
Fundamentals_of_Electric_Circuits_4th_Ed.pdf
Fundamentals_of_Electric_Circuits_4th_Ed.pdf
Fundamentals_of_Electric_Circuits_4th_Ed.pdf
Fundamentals_of_Electric_Circuits_4th_Ed.pdf
Fundamentals_of_Electric_Circuits_4th_Ed.pdf
Fundamentals_of_Electric_Circuits_4th_Ed.pdf
Fundamentals_of_Electric_Circuits_4th_Ed.pdf
Fundamentals_of_Electric_Circuits_4th_Ed.pdf
Fundamentals_of_Electric_Circuits_4th_Ed.pdf
Fundamentals_of_Electric_Circuits_4th_Ed.pdf
Fundamentals_of_Electric_Circuits_4th_Ed.pdf
Fundamentals_of_Electric_Circuits_4th_Ed.pdf
Fundamentals_of_Electric_Circuits_4th_Ed.pdf
Fundamentals_of_Electric_Circuits_4th_Ed.pdf
Fundamentals_of_Electric_Circuits_4th_Ed.pdf
Fundamentals_of_Electric_Circuits_4th_Ed.pdf
Fundamentals_of_Electric_Circuits_4th_Ed.pdf
Fundamentals_of_Electric_Circuits_4th_Ed.pdf
Fundamentals_of_Electric_Circuits_4th_Ed.pdf
Fundamentals_of_Electric_Circuits_4th_Ed.pdf
Fundamentals_of_Electric_Circuits_4th_Ed.pdf
Fundamentals_of_Electric_Circuits_4th_Ed.pdf
Fundamentals_of_Electric_Circuits_4th_Ed.pdf
Fundamentals_of_Electric_Circuits_4th_Ed.pdf
Fundamentals_of_Electric_Circuits_4th_Ed.pdf
Fundamentals_of_Electric_Circuits_4th_Ed.pdf
Fundamentals_of_Electric_Circuits_4th_Ed.pdf
Fundamentals_of_Electric_Circuits_4th_Ed.pdf
Fundamentals_of_Electric_Circuits_4th_Ed.pdf
Fundamentals_of_Electric_Circuits_4th_Ed.pdf
Fundamentals_of_Electric_Circuits_4th_Ed.pdf
Fundamentals_of_Electric_Circuits_4th_Ed.pdf
Fundamentals_of_Electric_Circuits_4th_Ed.pdf
Fundamentals_of_Electric_Circuits_4th_Ed.pdf
Fundamentals_of_Electric_Circuits_4th_Ed.pdf
Fundamentals_of_Electric_Circuits_4th_Ed.pdf
Fundamentals_of_Electric_Circuits_4th_Ed.pdf
Fundamentals_of_Electric_Circuits_4th_Ed.pdf
Fundamentals_of_Electric_Circuits_4th_Ed.pdf
Fundamentals_of_Electric_Circuits_4th_Ed.pdf
Fundamentals_of_Electric_Circuits_4th_Ed.pdf
Fundamentals_of_Electric_Circuits_4th_Ed.pdf
Fundamentals_of_Electric_Circuits_4th_Ed.pdf
Fundamentals_of_Electric_Circuits_4th_Ed.pdf
Fundamentals_of_Electric_Circuits_4th_Ed.pdf
Fundamentals_of_Electric_Circuits_4th_Ed.pdf
Fundamentals_of_Electric_Circuits_4th_Ed.pdf
Fundamentals_of_Electric_Circuits_4th_Ed.pdf
Fundamentals_of_Electric_Circuits_4th_Ed.pdf
Fundamentals_of_Electric_Circuits_4th_Ed.pdf
Fundamentals_of_Electric_Circuits_4th_Ed.pdf
Fundamentals_of_Electric_Circuits_4th_Ed.pdf
Fundamentals_of_Electric_Circuits_4th_Ed.pdf
Fundamentals_of_Electric_Circuits_4th_Ed.pdf
Fundamentals_of_Electric_Circuits_4th_Ed.pdf
Fundamentals_of_Electric_Circuits_4th_Ed.pdf
Fundamentals_of_Electric_Circuits_4th_Ed.pdf
Fundamentals_of_Electric_Circuits_4th_Ed.pdf
Fundamentals_of_Electric_Circuits_4th_Ed.pdf
Fundamentals_of_Electric_Circuits_4th_Ed.pdf
Fundamentals_of_Electric_Circuits_4th_Ed.pdf
Fundamentals_of_Electric_Circuits_4th_Ed.pdf
Fundamentals_of_Electric_Circuits_4th_Ed.pdf
Fundamentals_of_Electric_Circuits_4th_Ed.pdf
Fundamentals_of_Electric_Circuits_4th_Ed.pdf
Fundamentals_of_Electric_Circuits_4th_Ed.pdf
Fundamentals_of_Electric_Circuits_4th_Ed.pdf
Fundamentals_of_Electric_Circuits_4th_Ed.pdf
Fundamentals_of_Electric_Circuits_4th_Ed.pdf
Fundamentals_of_Electric_Circuits_4th_Ed.pdf
Fundamentals_of_Electric_Circuits_4th_Ed.pdf
Fundamentals_of_Electric_Circuits_4th_Ed.pdf
Fundamentals_of_Electric_Circuits_4th_Ed.pdf
Fundamentals_of_Electric_Circuits_4th_Ed.pdf
Fundamentals_of_Electric_Circuits_4th_Ed.pdf
Fundamentals_of_Electric_Circuits_4th_Ed.pdf
Fundamentals_of_Electric_Circuits_4th_Ed.pdf
Fundamentals_of_Electric_Circuits_4th_Ed.pdf
Fundamentals_of_Electric_Circuits_4th_Ed.pdf
Fundamentals_of_Electric_Circuits_4th_Ed.pdf
Fundamentals_of_Electric_Circuits_4th_Ed.pdf
Fundamentals_of_Electric_Circuits_4th_Ed.pdf
Fundamentals_of_Electric_Circuits_4th_Ed.pdf
Fundamentals_of_Electric_Circuits_4th_Ed.pdf
Fundamentals_of_Electric_Circuits_4th_Ed.pdf
Fundamentals_of_Electric_Circuits_4th_Ed.pdf
Fundamentals_of_Electric_Circuits_4th_Ed.pdf
Fundamentals_of_Electric_Circuits_4th_Ed.pdf
Fundamentals_of_Electric_Circuits_4th_Ed.pdf
Fundamentals_of_Electric_Circuits_4th_Ed.pdf
Fundamentals_of_Electric_Circuits_4th_Ed.pdf
Fundamentals_of_Electric_Circuits_4th_Ed.pdf
Fundamentals_of_Electric_Circuits_4th_Ed.pdf
Fundamentals_of_Electric_Circuits_4th_Ed.pdf
Fundamentals_of_Electric_Circuits_4th_Ed.pdf
Fundamentals_of_Electric_Circuits_4th_Ed.pdf
Fundamentals_of_Electric_Circuits_4th_Ed.pdf
Fundamentals_of_Electric_Circuits_4th_Ed.pdf
Fundamentals_of_Electric_Circuits_4th_Ed.pdf
Fundamentals_of_Electric_Circuits_4th_Ed.pdf
Fundamentals_of_Electric_Circuits_4th_Ed.pdf
Fundamentals_of_Electric_Circuits_4th_Ed.pdf
Fundamentals_of_Electric_Circuits_4th_Ed.pdf
Fundamentals_of_Electric_Circuits_4th_Ed.pdf
Fundamentals_of_Electric_Circuits_4th_Ed.pdf
Fundamentals_of_Electric_Circuits_4th_Ed.pdf
Fundamentals_of_Electric_Circuits_4th_Ed.pdf
Fundamentals_of_Electric_Circuits_4th_Ed.pdf
Fundamentals_of_Electric_Circuits_4th_Ed.pdf
Fundamentals_of_Electric_Circuits_4th_Ed.pdf
Fundamentals_of_Electric_Circuits_4th_Ed.pdf
Fundamentals_of_Electric_Circuits_4th_Ed.pdf
Fundamentals_of_Electric_Circuits_4th_Ed.pdf
Fundamentals_of_Electric_Circuits_4th_Ed.pdf
Fundamentals_of_Electric_Circuits_4th_Ed.pdf
Fundamentals_of_Electric_Circuits_4th_Ed.pdf
Fundamentals_of_Electric_Circuits_4th_Ed.pdf
Fundamentals_of_Electric_Circuits_4th_Ed.pdf
Fundamentals_of_Electric_Circuits_4th_Ed.pdf
Fundamentals_of_Electric_Circuits_4th_Ed.pdf
Fundamentals_of_Electric_Circuits_4th_Ed.pdf
Fundamentals_of_Electric_Circuits_4th_Ed.pdf
Fundamentals_of_Electric_Circuits_4th_Ed.pdf
Fundamentals_of_Electric_Circuits_4th_Ed.pdf
Fundamentals_of_Electric_Circuits_4th_Ed.pdf
Fundamentals_of_Electric_Circuits_4th_Ed.pdf
Fundamentals_of_Electric_Circuits_4th_Ed.pdf
Fundamentals_of_Electric_Circuits_4th_Ed.pdf
Fundamentals_of_Electric_Circuits_4th_Ed.pdf
Fundamentals_of_Electric_Circuits_4th_Ed.pdf
Fundamentals_of_Electric_Circuits_4th_Ed.pdf
Fundamentals_of_Electric_Circuits_4th_Ed.pdf
Fundamentals_of_Electric_Circuits_4th_Ed.pdf
Fundamentals_of_Electric_Circuits_4th_Ed.pdf
Fundamentals_of_Electric_Circuits_4th_Ed.pdf
Fundamentals_of_Electric_Circuits_4th_Ed.pdf
Fundamentals_of_Electric_Circuits_4th_Ed.pdf
Fundamentals_of_Electric_Circuits_4th_Ed.pdf
Fundamentals_of_Electric_Circuits_4th_Ed.pdf
Fundamentals_of_Electric_Circuits_4th_Ed.pdf
Fundamentals_of_Electric_Circuits_4th_Ed.pdf
Fundamentals_of_Electric_Circuits_4th_Ed.pdf
Fundamentals_of_Electric_Circuits_4th_Ed.pdf
Fundamentals_of_Electric_Circuits_4th_Ed.pdf
Fundamentals_of_Electric_Circuits_4th_Ed.pdf
Fundamentals_of_Electric_Circuits_4th_Ed.pdf
Fundamentals_of_Electric_Circuits_4th_Ed.pdf
Fundamentals_of_Electric_Circuits_4th_Ed.pdf
Fundamentals_of_Electric_Circuits_4th_Ed.pdf
Fundamentals_of_Electric_Circuits_4th_Ed.pdf
Fundamentals_of_Electric_Circuits_4th_Ed.pdf
Fundamentals_of_Electric_Circuits_4th_Ed.pdf
Fundamentals_of_Electric_Circuits_4th_Ed.pdf
Fundamentals_of_Electric_Circuits_4th_Ed.pdf
Fundamentals_of_Electric_Circuits_4th_Ed.pdf
Fundamentals_of_Electric_Circuits_4th_Ed.pdf
Fundamentals_of_Electric_Circuits_4th_Ed.pdf
Fundamentals_of_Electric_Circuits_4th_Ed.pdf
Fundamentals_of_Electric_Circuits_4th_Ed.pdf
Fundamentals_of_Electric_Circuits_4th_Ed.pdf
Fundamentals_of_Electric_Circuits_4th_Ed.pdf
Fundamentals_of_Electric_Circuits_4th_Ed.pdf
Fundamentals_of_Electric_Circuits_4th_Ed.pdf
Fundamentals_of_Electric_Circuits_4th_Ed.pdf
Fundamentals_of_Electric_Circuits_4th_Ed.pdf
Fundamentals_of_Electric_Circuits_4th_Ed.pdf
Fundamentals_of_Electric_Circuits_4th_Ed.pdf
Fundamentals_of_Electric_Circuits_4th_Ed.pdf
Fundamentals_of_Electric_Circuits_4th_Ed.pdf
Fundamentals_of_Electric_Circuits_4th_Ed.pdf
Fundamentals_of_Electric_Circuits_4th_Ed.pdf
Fundamentals_of_Electric_Circuits_4th_Ed.pdf
Fundamentals_of_Electric_Circuits_4th_Ed.pdf
Fundamentals_of_Electric_Circuits_4th_Ed.pdf
Fundamentals_of_Electric_Circuits_4th_Ed.pdf
Fundamentals_of_Electric_Circuits_4th_Ed.pdf
Fundamentals_of_Electric_Circuits_4th_Ed.pdf
Fundamentals_of_Electric_Circuits_4th_Ed.pdf
Fundamentals_of_Electric_Circuits_4th_Ed.pdf
Fundamentals_of_Electric_Circuits_4th_Ed.pdf
Fundamentals_of_Electric_Circuits_4th_Ed.pdf
Fundamentals_of_Electric_Circuits_4th_Ed.pdf
Fundamentals_of_Electric_Circuits_4th_Ed.pdf
Fundamentals_of_Electric_Circuits_4th_Ed.pdf
Fundamentals_of_Electric_Circuits_4th_Ed.pdf
Fundamentals_of_Electric_Circuits_4th_Ed.pdf
Fundamentals_of_Electric_Circuits_4th_Ed.pdf
Fundamentals_of_Electric_Circuits_4th_Ed.pdf
Fundamentals_of_Electric_Circuits_4th_Ed.pdf
Fundamentals_of_Electric_Circuits_4th_Ed.pdf
Fundamentals_of_Electric_Circuits_4th_Ed.pdf
Fundamentals_of_Electric_Circuits_4th_Ed.pdf
Fundamentals_of_Electric_Circuits_4th_Ed.pdf
Fundamentals_of_Electric_Circuits_4th_Ed.pdf
Fundamentals_of_Electric_Circuits_4th_Ed.pdf
Fundamentals_of_Electric_Circuits_4th_Ed.pdf
Fundamentals_of_Electric_Circuits_4th_Ed.pdf
Fundamentals_of_Electric_Circuits_4th_Ed.pdf
Fundamentals_of_Electric_Circuits_4th_Ed.pdf
Fundamentals_of_Electric_Circuits_4th_Ed.pdf
Fundamentals_of_Electric_Circuits_4th_Ed.pdf
Fundamentals_of_Electric_Circuits_4th_Ed.pdf
Fundamentals_of_Electric_Circuits_4th_Ed.pdf
Fundamentals_of_Electric_Circuits_4th_Ed.pdf
Fundamentals_of_Electric_Circuits_4th_Ed.pdf
Fundamentals_of_Electric_Circuits_4th_Ed.pdf
Fundamentals_of_Electric_Circuits_4th_Ed.pdf
Fundamentals_of_Electric_Circuits_4th_Ed.pdf
Fundamentals_of_Electric_Circuits_4th_Ed.pdf
Fundamentals_of_Electric_Circuits_4th_Ed.pdf
Fundamentals_of_Electric_Circuits_4th_Ed.pdf
Fundamentals_of_Electric_Circuits_4th_Ed.pdf
Fundamentals_of_Electric_Circuits_4th_Ed.pdf
Fundamentals_of_Electric_Circuits_4th_Ed.pdf
Fundamentals_of_Electric_Circuits_4th_Ed.pdf
Fundamentals_of_Electric_Circuits_4th_Ed.pdf
Fundamentals_of_Electric_Circuits_4th_Ed.pdf
Fundamentals_of_Electric_Circuits_4th_Ed.pdf
Fundamentals_of_Electric_Circuits_4th_Ed.pdf
Fundamentals_of_Electric_Circuits_4th_Ed.pdf
Fundamentals_of_Electric_Circuits_4th_Ed.pdf
Fundamentals_of_Electric_Circuits_4th_Ed.pdf
Fundamentals_of_Electric_Circuits_4th_Ed.pdf
Fundamentals_of_Electric_Circuits_4th_Ed.pdf
Fundamentals_of_Electric_Circuits_4th_Ed.pdf
Fundamentals_of_Electric_Circuits_4th_Ed.pdf
Fundamentals_of_Electric_Circuits_4th_Ed.pdf
Fundamentals_of_Electric_Circuits_4th_Ed.pdf
Fundamentals_of_Electric_Circuits_4th_Ed.pdf
Fundamentals_of_Electric_Circuits_4th_Ed.pdf
Fundamentals_of_Electric_Circuits_4th_Ed.pdf
Fundamentals_of_Electric_Circuits_4th_Ed.pdf
Fundamentals_of_Electric_Circuits_4th_Ed.pdf
Fundamentals_of_Electric_Circuits_4th_Ed.pdf
Fundamentals_of_Electric_Circuits_4th_Ed.pdf
Fundamentals_of_Electric_Circuits_4th_Ed.pdf
Fundamentals_of_Electric_Circuits_4th_Ed.pdf
Fundamentals_of_Electric_Circuits_4th_Ed.pdf
Fundamentals_of_Electric_Circuits_4th_Ed.pdf
Fundamentals_of_Electric_Circuits_4th_Ed.pdf
Fundamentals_of_Electric_Circuits_4th_Ed.pdf
Fundamentals_of_Electric_Circuits_4th_Ed.pdf
Fundamentals_of_Electric_Circuits_4th_Ed.pdf
Fundamentals_of_Electric_Circuits_4th_Ed.pdf
Fundamentals_of_Electric_Circuits_4th_Ed.pdf
Fundamentals_of_Electric_Circuits_4th_Ed.pdf
Fundamentals_of_Electric_Circuits_4th_Ed.pdf
Fundamentals_of_Electric_Circuits_4th_Ed.pdf
Fundamentals_of_Electric_Circuits_4th_Ed.pdf
Fundamentals_of_Electric_Circuits_4th_Ed.pdf
Fundamentals_of_Electric_Circuits_4th_Ed.pdf
Fundamentals_of_Electric_Circuits_4th_Ed.pdf
Fundamentals_of_Electric_Circuits_4th_Ed.pdf
Fundamentals_of_Electric_Circuits_4th_Ed.pdf
Fundamentals_of_Electric_Circuits_4th_Ed.pdf
Fundamentals_of_Electric_Circuits_4th_Ed.pdf
Fundamentals_of_Electric_Circuits_4th_Ed.pdf
Fundamentals_of_Electric_Circuits_4th_Ed.pdf
Fundamentals_of_Electric_Circuits_4th_Ed.pdf
Fundamentals_of_Electric_Circuits_4th_Ed.pdf
Fundamentals_of_Electric_Circuits_4th_Ed.pdf
Fundamentals_of_Electric_Circuits_4th_Ed.pdf
Fundamentals_of_Electric_Circuits_4th_Ed.pdf
Fundamentals_of_Electric_Circuits_4th_Ed.pdf
Fundamentals_of_Electric_Circuits_4th_Ed.pdf
Fundamentals_of_Electric_Circuits_4th_Ed.pdf
Fundamentals_of_Electric_Circuits_4th_Ed.pdf
Fundamentals_of_Electric_Circuits_4th_Ed.pdf
Fundamentals_of_Electric_Circuits_4th_Ed.pdf
Fundamentals_of_Electric_Circuits_4th_Ed.pdf
Fundamentals_of_Electric_Circuits_4th_Ed.pdf
Fundamentals_of_Electric_Circuits_4th_Ed.pdf
Fundamentals_of_Electric_Circuits_4th_Ed.pdf
Fundamentals_of_Electric_Circuits_4th_Ed.pdf
Fundamentals_of_Electric_Circuits_4th_Ed.pdf
Fundamentals_of_Electric_Circuits_4th_Ed.pdf
Fundamentals_of_Electric_Circuits_4th_Ed.pdf
Fundamentals_of_Electric_Circuits_4th_Ed.pdf
Fundamentals_of_Electric_Circuits_4th_Ed.pdf
Fundamentals_of_Electric_Circuits_4th_Ed.pdf
Fundamentals_of_Electric_Circuits_4th_Ed.pdf
Fundamentals_of_Electric_Circuits_4th_Ed.pdf
Fundamentals_of_Electric_Circuits_4th_Ed.pdf
Fundamentals_of_Electric_Circuits_4th_Ed.pdf
Fundamentals_of_Electric_Circuits_4th_Ed.pdf
Fundamentals_of_Electric_Circuits_4th_Ed.pdf
Fundamentals_of_Electric_Circuits_4th_Ed.pdf
Fundamentals_of_Electric_Circuits_4th_Ed.pdf
Fundamentals_of_Electric_Circuits_4th_Ed.pdf
Fundamentals_of_Electric_Circuits_4th_Ed.pdf
Fundamentals_of_Electric_Circuits_4th_Ed.pdf
Fundamentals_of_Electric_Circuits_4th_Ed.pdf
Fundamentals_of_Electric_Circuits_4th_Ed.pdf
Fundamentals_of_Electric_Circuits_4th_Ed.pdf
Fundamentals_of_Electric_Circuits_4th_Ed.pdf
Fundamentals_of_Electric_Circuits_4th_Ed.pdf
Fundamentals_of_Electric_Circuits_4th_Ed.pdf
Fundamentals_of_Electric_Circuits_4th_Ed.pdf
Fundamentals_of_Electric_Circuits_4th_Ed.pdf
Fundamentals_of_Electric_Circuits_4th_Ed.pdf
Fundamentals_of_Electric_Circuits_4th_Ed.pdf
Fundamentals_of_Electric_Circuits_4th_Ed.pdf
Fundamentals_of_Electric_Circuits_4th_Ed.pdf
Fundamentals_of_Electric_Circuits_4th_Ed.pdf
Fundamentals_of_Electric_Circuits_4th_Ed.pdf
Fundamentals_of_Electric_Circuits_4th_Ed.pdf
Fundamentals_of_Electric_Circuits_4th_Ed.pdf
Fundamentals_of_Electric_Circuits_4th_Ed.pdf
Fundamentals_of_Electric_Circuits_4th_Ed.pdf
Fundamentals_of_Electric_Circuits_4th_Ed.pdf
Fundamentals_of_Electric_Circuits_4th_Ed.pdf
Fundamentals_of_Electric_Circuits_4th_Ed.pdf
Fundamentals_of_Electric_Circuits_4th_Ed.pdf
Fundamentals_of_Electric_Circuits_4th_Ed.pdf
Fundamentals_of_Electric_Circuits_4th_Ed.pdf
Fundamentals_of_Electric_Circuits_4th_Ed.pdf
Fundamentals_of_Electric_Circuits_4th_Ed.pdf
Fundamentals_of_Electric_Circuits_4th_Ed.pdf
Fundamentals_of_Electric_Circuits_4th_Ed.pdf
Fundamentals_of_Electric_Circuits_4th_Ed.pdf
Fundamentals_of_Electric_Circuits_4th_Ed.pdf
Fundamentals_of_Electric_Circuits_4th_Ed.pdf
Fundamentals_of_Electric_Circuits_4th_Ed.pdf
Fundamentals_of_Electric_Circuits_4th_Ed.pdf
Fundamentals_of_Electric_Circuits_4th_Ed.pdf
Fundamentals_of_Electric_Circuits_4th_Ed.pdf
Fundamentals_of_Electric_Circuits_4th_Ed.pdf
Fundamentals_of_Electric_Circuits_4th_Ed.pdf
Fundamentals_of_Electric_Circuits_4th_Ed.pdf
Fundamentals_of_Electric_Circuits_4th_Ed.pdf
Fundamentals_of_Electric_Circuits_4th_Ed.pdf
Fundamentals_of_Electric_Circuits_4th_Ed.pdf
Fundamentals_of_Electric_Circuits_4th_Ed.pdf
Fundamentals_of_Electric_Circuits_4th_Ed.pdf
Fundamentals_of_Electric_Circuits_4th_Ed.pdf
Fundamentals_of_Electric_Circuits_4th_Ed.pdf
Fundamentals_of_Electric_Circuits_4th_Ed.pdf
Fundamentals_of_Electric_Circuits_4th_Ed.pdf
Fundamentals_of_Electric_Circuits_4th_Ed.pdf
Fundamentals_of_Electric_Circuits_4th_Ed.pdf
Fundamentals_of_Electric_Circuits_4th_Ed.pdf
Fundamentals_of_Electric_Circuits_4th_Ed.pdf
Fundamentals_of_Electric_Circuits_4th_Ed.pdf
Fundamentals_of_Electric_Circuits_4th_Ed.pdf
Fundamentals_of_Electric_Circuits_4th_Ed.pdf
Fundamentals_of_Electric_Circuits_4th_Ed.pdf
Fundamentals_of_Electric_Circuits_4th_Ed.pdf
Fundamentals_of_Electric_Circuits_4th_Ed.pdf
Fundamentals_of_Electric_Circuits_4th_Ed.pdf
Fundamentals_of_Electric_Circuits_4th_Ed.pdf
Fundamentals_of_Electric_Circuits_4th_Ed.pdf
Fundamentals_of_Electric_Circuits_4th_Ed.pdf
Fundamentals_of_Electric_Circuits_4th_Ed.pdf
Fundamentals_of_Electric_Circuits_4th_Ed.pdf
Fundamentals_of_Electric_Circuits_4th_Ed.pdf
Fundamentals_of_Electric_Circuits_4th_Ed.pdf
Fundamentals_of_Electric_Circuits_4th_Ed.pdf
Fundamentals_of_Electric_Circuits_4th_Ed.pdf
Fundamentals_of_Electric_Circuits_4th_Ed.pdf
Fundamentals_of_Electric_Circuits_4th_Ed.pdf
Fundamentals_of_Electric_Circuits_4th_Ed.pdf
Fundamentals_of_Electric_Circuits_4th_Ed.pdf
Fundamentals_of_Electric_Circuits_4th_Ed.pdf
Fundamentals_of_Electric_Circuits_4th_Ed.pdf
Fundamentals_of_Electric_Circuits_4th_Ed.pdf
Fundamentals_of_Electric_Circuits_4th_Ed.pdf
Fundamentals_of_Electric_Circuits_4th_Ed.pdf
Fundamentals_of_Electric_Circuits_4th_Ed.pdf
Fundamentals_of_Electric_Circuits_4th_Ed.pdf
Fundamentals_of_Electric_Circuits_4th_Ed.pdf
Fundamentals_of_Electric_Circuits_4th_Ed.pdf
Fundamentals_of_Electric_Circuits_4th_Ed.pdf
Fundamentals_of_Electric_Circuits_4th_Ed.pdf
Fundamentals_of_Electric_Circuits_4th_Ed.pdf
Fundamentals_of_Electric_Circuits_4th_Ed.pdf
Fundamentals_of_Electric_Circuits_4th_Ed.pdf
Fundamentals_of_Electric_Circuits_4th_Ed.pdf
Fundamentals_of_Electric_Circuits_4th_Ed.pdf
Fundamentals_of_Electric_Circuits_4th_Ed.pdf
Fundamentals_of_Electric_Circuits_4th_Ed.pdf
Fundamentals_of_Electric_Circuits_4th_Ed.pdf
Fundamentals_of_Electric_Circuits_4th_Ed.pdf
Fundamentals_of_Electric_Circuits_4th_Ed.pdf
Fundamentals_of_Electric_Circuits_4th_Ed.pdf
Fundamentals_of_Electric_Circuits_4th_Ed.pdf
Fundamentals_of_Electric_Circuits_4th_Ed.pdf
Fundamentals_of_Electric_Circuits_4th_Ed.pdf
Fundamentals_of_Electric_Circuits_4th_Ed.pdf
Fundamentals_of_Electric_Circuits_4th_Ed.pdf
Fundamentals_of_Electric_Circuits_4th_Ed.pdf
Fundamentals_of_Electric_Circuits_4th_Ed.pdf
Fundamentals_of_Electric_Circuits_4th_Ed.pdf
Fundamentals_of_Electric_Circuits_4th_Ed.pdf
Fundamentals_of_Electric_Circuits_4th_Ed.pdf
Fundamentals_of_Electric_Circuits_4th_Ed.pdf
Fundamentals_of_Electric_Circuits_4th_Ed.pdf
Fundamentals_of_Electric_Circuits_4th_Ed.pdf
Fundamentals_of_Electric_Circuits_4th_Ed.pdf
Fundamentals_of_Electric_Circuits_4th_Ed.pdf
Fundamentals_of_Electric_Circuits_4th_Ed.pdf
Fundamentals_of_Electric_Circuits_4th_Ed.pdf
Fundamentals_of_Electric_Circuits_4th_Ed.pdf
Fundamentals_of_Electric_Circuits_4th_Ed.pdf
Fundamentals_of_Electric_Circuits_4th_Ed.pdf
Fundamentals_of_Electric_Circuits_4th_Ed.pdf
Fundamentals_of_Electric_Circuits_4th_Ed.pdf
Fundamentals_of_Electric_Circuits_4th_Ed.pdf
Fundamentals_of_Electric_Circuits_4th_Ed.pdf
Fundamentals_of_Electric_Circuits_4th_Ed.pdf
Fundamentals_of_Electric_Circuits_4th_Ed.pdf
Fundamentals_of_Electric_Circuits_4th_Ed.pdf
Fundamentals_of_Electric_Circuits_4th_Ed.pdf
Fundamentals_of_Electric_Circuits_4th_Ed.pdf
Fundamentals_of_Electric_Circuits_4th_Ed.pdf
Fundamentals_of_Electric_Circuits_4th_Ed.pdf
Fundamentals_of_Electric_Circuits_4th_Ed.pdf
Fundamentals_of_Electric_Circuits_4th_Ed.pdf
Fundamentals_of_Electric_Circuits_4th_Ed.pdf
Fundamentals_of_Electric_Circuits_4th_Ed.pdf
Fundamentals_of_Electric_Circuits_4th_Ed.pdf
Fundamentals_of_Electric_Circuits_4th_Ed.pdf
Fundamentals_of_Electric_Circuits_4th_Ed.pdf
Fundamentals_of_Electric_Circuits_4th_Ed.pdf
Fundamentals_of_Electric_Circuits_4th_Ed.pdf
Fundamentals_of_Electric_Circuits_4th_Ed.pdf
Fundamentals_of_Electric_Circuits_4th_Ed.pdf
Fundamentals_of_Electric_Circuits_4th_Ed.pdf
Fundamentals_of_Electric_Circuits_4th_Ed.pdf
Fundamentals_of_Electric_Circuits_4th_Ed.pdf
Fundamentals_of_Electric_Circuits_4th_Ed.pdf
Fundamentals_of_Electric_Circuits_4th_Ed.pdf
Fundamentals_of_Electric_Circuits_4th_Ed.pdf
Fundamentals_of_Electric_Circuits_4th_Ed.pdf
Fundamentals_of_Electric_Circuits_4th_Ed.pdf
Fundamentals_of_Electric_Circuits_4th_Ed.pdf
Fundamentals_of_Electric_Circuits_4th_Ed.pdf
Fundamentals_of_Electric_Circuits_4th_Ed.pdf
Fundamentals_of_Electric_Circuits_4th_Ed.pdf
Fundamentals_of_Electric_Circuits_4th_Ed.pdf
Fundamentals_of_Electric_Circuits_4th_Ed.pdf
Fundamentals_of_Electric_Circuits_4th_Ed.pdf
Fundamentals_of_Electric_Circuits_4th_Ed.pdf
Fundamentals_of_Electric_Circuits_4th_Ed.pdf
Fundamentals_of_Electric_Circuits_4th_Ed.pdf
Fundamentals_of_Electric_Circuits_4th_Ed.pdf
Fundamentals_of_Electric_Circuits_4th_Ed.pdf
Fundamentals_of_Electric_Circuits_4th_Ed.pdf
Fundamentals_of_Electric_Circuits_4th_Ed.pdf
Fundamentals_of_Electric_Circuits_4th_Ed.pdf
Fundamentals_of_Electric_Circuits_4th_Ed.pdf
Fundamentals_of_Electric_Circuits_4th_Ed.pdf
Fundamentals_of_Electric_Circuits_4th_Ed.pdf
Fundamentals_of_Electric_Circuits_4th_Ed.pdf
Fundamentals_of_Electric_Circuits_4th_Ed.pdf
Fundamentals_of_Electric_Circuits_4th_Ed.pdf
Fundamentals_of_Electric_Circuits_4th_Ed.pdf
Fundamentals_of_Electric_Circuits_4th_Ed.pdf
Fundamentals_of_Electric_Circuits_4th_Ed.pdf
Fundamentals_of_Electric_Circuits_4th_Ed.pdf
Fundamentals_of_Electric_Circuits_4th_Ed.pdf
Fundamentals_of_Electric_Circuits_4th_Ed.pdf
Fundamentals_of_Electric_Circuits_4th_Ed.pdf
Fundamentals_of_Electric_Circuits_4th_Ed.pdf
Fundamentals_of_Electric_Circuits_4th_Ed.pdf
Fundamentals_of_Electric_Circuits_4th_Ed.pdf
Fundamentals_of_Electric_Circuits_4th_Ed.pdf
Fundamentals_of_Electric_Circuits_4th_Ed.pdf
Fundamentals_of_Electric_Circuits_4th_Ed.pdf
Fundamentals_of_Electric_Circuits_4th_Ed.pdf
Fundamentals_of_Electric_Circuits_4th_Ed.pdf
Fundamentals_of_Electric_Circuits_4th_Ed.pdf
Fundamentals_of_Electric_Circuits_4th_Ed.pdf
Fundamentals_of_Electric_Circuits_4th_Ed.pdf
Fundamentals_of_Electric_Circuits_4th_Ed.pdf
Fundamentals_of_Electric_Circuits_4th_Ed.pdf
Fundamentals_of_Electric_Circuits_4th_Ed.pdf
Fundamentals_of_Electric_Circuits_4th_Ed.pdf
Fundamentals_of_Electric_Circuits_4th_Ed.pdf
Fundamentals_of_Electric_Circuits_4th_Ed.pdf
Fundamentals_of_Electric_Circuits_4th_Ed.pdf
Fundamentals_of_Electric_Circuits_4th_Ed.pdf
Fundamentals_of_Electric_Circuits_4th_Ed.pdf
Fundamentals_of_Electric_Circuits_4th_Ed.pdf
Fundamentals_of_Electric_Circuits_4th_Ed.pdf
Fundamentals_of_Electric_Circuits_4th_Ed.pdf
Fundamentals_of_Electric_Circuits_4th_Ed.pdf
Fundamentals_of_Electric_Circuits_4th_Ed.pdf
Fundamentals_of_Electric_Circuits_4th_Ed.pdf
Fundamentals_of_Electric_Circuits_4th_Ed.pdf
Fundamentals_of_Electric_Circuits_4th_Ed.pdf
Fundamentals_of_Electric_Circuits_4th_Ed.pdf
Fundamentals_of_Electric_Circuits_4th_Ed.pdf
Fundamentals_of_Electric_Circuits_4th_Ed.pdf
Fundamentals_of_Electric_Circuits_4th_Ed.pdf
Fundamentals_of_Electric_Circuits_4th_Ed.pdf
Fundamentals_of_Electric_Circuits_4th_Ed.pdf
Fundamentals_of_Electric_Circuits_4th_Ed.pdf
Fundamentals_of_Electric_Circuits_4th_Ed.pdf
Fundamentals_of_Electric_Circuits_4th_Ed.pdf
Fundamentals_of_Electric_Circuits_4th_Ed.pdf
Fundamentals_of_Electric_Circuits_4th_Ed.pdf
Fundamentals_of_Electric_Circuits_4th_Ed.pdf
Fundamentals_of_Electric_Circuits_4th_Ed.pdf

More Related Content

Similar to Fundamentals_of_Electric_Circuits_4th_Ed.pdf

Mis 589 Success Begins / snaptutorial.com
Mis 589  Success Begins / snaptutorial.comMis 589  Success Begins / snaptutorial.com
Mis 589 Success Begins / snaptutorial.comWilliamsTaylor44
 
Mis 589 Massive Success / snaptutorial.com
Mis 589 Massive Success / snaptutorial.comMis 589 Massive Success / snaptutorial.com
Mis 589 Massive Success / snaptutorial.comStephenson185
 
Gr9 electricity
Gr9 electricityGr9 electricity
Gr9 electricityatong ang
 
The Impact of Applying LabVIEW Simulation Platform in Teaching Power Electron...
The Impact of Applying LabVIEW Simulation Platform in Teaching Power Electron...The Impact of Applying LabVIEW Simulation Platform in Teaching Power Electron...
The Impact of Applying LabVIEW Simulation Platform in Teaching Power Electron...Yashodhan Mohan Bhatnagar
 
Collecting Economic Data in Power Quality Survey
Collecting Economic Data in Power Quality SurveyCollecting Economic Data in Power Quality Survey
Collecting Economic Data in Power Quality SurveyLeonardo ENERGY
 
ElectronicDevicesAndCircuits_J.J.C..pdf
ElectronicDevicesAndCircuits_J.J.C..pdfElectronicDevicesAndCircuits_J.J.C..pdf
ElectronicDevicesAndCircuits_J.J.C..pdfivan ion
 
Power Electronics Handbook Second Edition Devices Circuits and Applications.pdf
Power Electronics Handbook Second Edition Devices Circuits and Applications.pdfPower Electronics Handbook Second Edition Devices Circuits and Applications.pdf
Power Electronics Handbook Second Edition Devices Circuits and Applications.pdfGollapalli Sreenivasulu
 
B.E. (EEE) Project Report Preparation Template-Rev (1).docx
B.E. (EEE) Project Report Preparation Template-Rev (1).docxB.E. (EEE) Project Report Preparation Template-Rev (1).docx
B.E. (EEE) Project Report Preparation Template-Rev (1).docxdivyeshparmar927
 
Please include MutiSIM fileIntroductionWrite one to two p.docx
Please include MutiSIM fileIntroductionWrite one to two p.docxPlease include MutiSIM fileIntroductionWrite one to two p.docx
Please include MutiSIM fileIntroductionWrite one to two p.docxmattjtoni51554
 
ECET 375 Effective Communication/tutorialrank.com
 ECET 375 Effective Communication/tutorialrank.com ECET 375 Effective Communication/tutorialrank.com
ECET 375 Effective Communication/tutorialrank.comjonhson203
 

Similar to Fundamentals_of_Electric_Circuits_4th_Ed.pdf (20)

Mis 589 Success Begins / snaptutorial.com
Mis 589  Success Begins / snaptutorial.comMis 589  Success Begins / snaptutorial.com
Mis 589 Success Begins / snaptutorial.com
 
Mis 589 Massive Success / snaptutorial.com
Mis 589 Massive Success / snaptutorial.comMis 589 Massive Success / snaptutorial.com
Mis 589 Massive Success / snaptutorial.com
 
Gr9 electricity
Gr9 electricityGr9 electricity
Gr9 electricity
 
The Impact of Applying LabVIEW Simulation Platform in Teaching Power Electron...
The Impact of Applying LabVIEW Simulation Platform in Teaching Power Electron...The Impact of Applying LabVIEW Simulation Platform in Teaching Power Electron...
The Impact of Applying LabVIEW Simulation Platform in Teaching Power Electron...
 
Collecting Economic Data in Power Quality Survey
Collecting Economic Data in Power Quality SurveyCollecting Economic Data in Power Quality Survey
Collecting Economic Data in Power Quality Survey
 
It 241 it241
It 241 it241It 241 it241
It 241 it241
 
EDC_WW8.pdf
EDC_WW8.pdfEDC_WW8.pdf
EDC_WW8.pdf
 
ElectronicDevicesAndCircuits_J.J.C..pdf
ElectronicDevicesAndCircuits_J.J.C..pdfElectronicDevicesAndCircuits_J.J.C..pdf
ElectronicDevicesAndCircuits_J.J.C..pdf
 
Power Electronics Handbook Second Edition Devices Circuits and Applications.pdf
Power Electronics Handbook Second Edition Devices Circuits and Applications.pdfPower Electronics Handbook Second Edition Devices Circuits and Applications.pdf
Power Electronics Handbook Second Edition Devices Circuits and Applications.pdf
 
B.E. (EEE) Project Report Preparation Template-Rev (1).docx
B.E. (EEE) Project Report Preparation Template-Rev (1).docxB.E. (EEE) Project Report Preparation Template-Rev (1).docx
B.E. (EEE) Project Report Preparation Template-Rev (1).docx
 
Antenna Balanis
Antenna BalanisAntenna Balanis
Antenna Balanis
 
pdf.pdf
pdf.pdfpdf.pdf
pdf.pdf
 
pdf.pdf
pdf.pdfpdf.pdf
pdf.pdf
 
pdf.pdf
pdf.pdfpdf.pdf
pdf.pdf
 
pdf.pdf
pdf.pdfpdf.pdf
pdf.pdf
 
Machine learning
Machine learningMachine learning
Machine learning
 
Please include MutiSIM fileIntroductionWrite one to two p.docx
Please include MutiSIM fileIntroductionWrite one to two p.docxPlease include MutiSIM fileIntroductionWrite one to two p.docx
Please include MutiSIM fileIntroductionWrite one to two p.docx
 
Thebook
ThebookThebook
Thebook
 
dadir mohmwed
dadir mohmweddadir mohmwed
dadir mohmwed
 
ECET 375 Effective Communication/tutorialrank.com
 ECET 375 Effective Communication/tutorialrank.com ECET 375 Effective Communication/tutorialrank.com
ECET 375 Effective Communication/tutorialrank.com
 

Recently uploaded

The Most Excellent Way | 1 Corinthians 13
The Most Excellent Way | 1 Corinthians 13The Most Excellent Way | 1 Corinthians 13
The Most Excellent Way | 1 Corinthians 13Steve Thomason
 
Interactive Powerpoint_How to Master effective communication
Interactive Powerpoint_How to Master effective communicationInteractive Powerpoint_How to Master effective communication
Interactive Powerpoint_How to Master effective communicationnomboosow
 
MENTAL STATUS EXAMINATION format.docx
MENTAL     STATUS EXAMINATION format.docxMENTAL     STATUS EXAMINATION format.docx
MENTAL STATUS EXAMINATION format.docxPoojaSen20
 
Paris 2024 Olympic Geographies - an activity
Paris 2024 Olympic Geographies - an activityParis 2024 Olympic Geographies - an activity
Paris 2024 Olympic Geographies - an activityGeoBlogs
 
Science 7 - LAND and SEA BREEZE and its Characteristics
Science 7 - LAND and SEA BREEZE and its CharacteristicsScience 7 - LAND and SEA BREEZE and its Characteristics
Science 7 - LAND and SEA BREEZE and its CharacteristicsKarinaGenton
 
Mastering the Unannounced Regulatory Inspection
Mastering the Unannounced Regulatory InspectionMastering the Unannounced Regulatory Inspection
Mastering the Unannounced Regulatory InspectionSafetyChain Software
 
Organic Name Reactions for the students and aspirants of Chemistry12th.pptx
Organic Name Reactions  for the students and aspirants of Chemistry12th.pptxOrganic Name Reactions  for the students and aspirants of Chemistry12th.pptx
Organic Name Reactions for the students and aspirants of Chemistry12th.pptxVS Mahajan Coaching Centre
 
Solving Puzzles Benefits Everyone (English).pptx
Solving Puzzles Benefits Everyone (English).pptxSolving Puzzles Benefits Everyone (English).pptx
Solving Puzzles Benefits Everyone (English).pptxOH TEIK BIN
 
“Oh GOSH! Reflecting on Hackteria's Collaborative Practices in a Global Do-It...
“Oh GOSH! Reflecting on Hackteria's Collaborative Practices in a Global Do-It...“Oh GOSH! Reflecting on Hackteria's Collaborative Practices in a Global Do-It...
“Oh GOSH! Reflecting on Hackteria's Collaborative Practices in a Global Do-It...Marc Dusseiller Dusjagr
 
ECONOMIC CONTEXT - LONG FORM TV DRAMA - PPT
ECONOMIC CONTEXT - LONG FORM TV DRAMA - PPTECONOMIC CONTEXT - LONG FORM TV DRAMA - PPT
ECONOMIC CONTEXT - LONG FORM TV DRAMA - PPTiammrhaywood
 
18-04-UA_REPORT_MEDIALITERAСY_INDEX-DM_23-1-final-eng.pdf
18-04-UA_REPORT_MEDIALITERAСY_INDEX-DM_23-1-final-eng.pdf18-04-UA_REPORT_MEDIALITERAСY_INDEX-DM_23-1-final-eng.pdf
18-04-UA_REPORT_MEDIALITERAСY_INDEX-DM_23-1-final-eng.pdfssuser54595a
 
SOCIAL AND HISTORICAL CONTEXT - LFTVD.pptx
SOCIAL AND HISTORICAL CONTEXT - LFTVD.pptxSOCIAL AND HISTORICAL CONTEXT - LFTVD.pptx
SOCIAL AND HISTORICAL CONTEXT - LFTVD.pptxiammrhaywood
 
Hybridoma Technology ( Production , Purification , and Application )
Hybridoma Technology  ( Production , Purification , and Application  ) Hybridoma Technology  ( Production , Purification , and Application  )
Hybridoma Technology ( Production , Purification , and Application ) Sakshi Ghasle
 
BASLIQ CURRENT LOOKBOOK LOOKBOOK(1) (1).pdf
BASLIQ CURRENT LOOKBOOK  LOOKBOOK(1) (1).pdfBASLIQ CURRENT LOOKBOOK  LOOKBOOK(1) (1).pdf
BASLIQ CURRENT LOOKBOOK LOOKBOOK(1) (1).pdfSoniaTolstoy
 
Presiding Officer Training module 2024 lok sabha elections
Presiding Officer Training module 2024 lok sabha electionsPresiding Officer Training module 2024 lok sabha elections
Presiding Officer Training module 2024 lok sabha electionsanshu789521
 
Presentation by Andreas Schleicher Tackling the School Absenteeism Crisis 30 ...
Presentation by Andreas Schleicher Tackling the School Absenteeism Crisis 30 ...Presentation by Andreas Schleicher Tackling the School Absenteeism Crisis 30 ...
Presentation by Andreas Schleicher Tackling the School Absenteeism Crisis 30 ...EduSkills OECD
 
Employee wellbeing at the workplace.pptx
Employee wellbeing at the workplace.pptxEmployee wellbeing at the workplace.pptx
Employee wellbeing at the workplace.pptxNirmalaLoungPoorunde1
 
mini mental status format.docx
mini    mental       status     format.docxmini    mental       status     format.docx
mini mental status format.docxPoojaSen20
 
Sanyam Choudhary Chemistry practical.pdf
Sanyam Choudhary Chemistry practical.pdfSanyam Choudhary Chemistry practical.pdf
Sanyam Choudhary Chemistry practical.pdfsanyamsingh5019
 

Recently uploaded (20)

The Most Excellent Way | 1 Corinthians 13
The Most Excellent Way | 1 Corinthians 13The Most Excellent Way | 1 Corinthians 13
The Most Excellent Way | 1 Corinthians 13
 
Interactive Powerpoint_How to Master effective communication
Interactive Powerpoint_How to Master effective communicationInteractive Powerpoint_How to Master effective communication
Interactive Powerpoint_How to Master effective communication
 
MENTAL STATUS EXAMINATION format.docx
MENTAL     STATUS EXAMINATION format.docxMENTAL     STATUS EXAMINATION format.docx
MENTAL STATUS EXAMINATION format.docx
 
Paris 2024 Olympic Geographies - an activity
Paris 2024 Olympic Geographies - an activityParis 2024 Olympic Geographies - an activity
Paris 2024 Olympic Geographies - an activity
 
Science 7 - LAND and SEA BREEZE and its Characteristics
Science 7 - LAND and SEA BREEZE and its CharacteristicsScience 7 - LAND and SEA BREEZE and its Characteristics
Science 7 - LAND and SEA BREEZE and its Characteristics
 
Model Call Girl in Bikash Puri Delhi reach out to us at 🔝9953056974🔝
Model Call Girl in Bikash Puri  Delhi reach out to us at 🔝9953056974🔝Model Call Girl in Bikash Puri  Delhi reach out to us at 🔝9953056974🔝
Model Call Girl in Bikash Puri Delhi reach out to us at 🔝9953056974🔝
 
Mastering the Unannounced Regulatory Inspection
Mastering the Unannounced Regulatory InspectionMastering the Unannounced Regulatory Inspection
Mastering the Unannounced Regulatory Inspection
 
Organic Name Reactions for the students and aspirants of Chemistry12th.pptx
Organic Name Reactions  for the students and aspirants of Chemistry12th.pptxOrganic Name Reactions  for the students and aspirants of Chemistry12th.pptx
Organic Name Reactions for the students and aspirants of Chemistry12th.pptx
 
Solving Puzzles Benefits Everyone (English).pptx
Solving Puzzles Benefits Everyone (English).pptxSolving Puzzles Benefits Everyone (English).pptx
Solving Puzzles Benefits Everyone (English).pptx
 
“Oh GOSH! Reflecting on Hackteria's Collaborative Practices in a Global Do-It...
“Oh GOSH! Reflecting on Hackteria's Collaborative Practices in a Global Do-It...“Oh GOSH! Reflecting on Hackteria's Collaborative Practices in a Global Do-It...
“Oh GOSH! Reflecting on Hackteria's Collaborative Practices in a Global Do-It...
 
ECONOMIC CONTEXT - LONG FORM TV DRAMA - PPT
ECONOMIC CONTEXT - LONG FORM TV DRAMA - PPTECONOMIC CONTEXT - LONG FORM TV DRAMA - PPT
ECONOMIC CONTEXT - LONG FORM TV DRAMA - PPT
 
18-04-UA_REPORT_MEDIALITERAСY_INDEX-DM_23-1-final-eng.pdf
18-04-UA_REPORT_MEDIALITERAСY_INDEX-DM_23-1-final-eng.pdf18-04-UA_REPORT_MEDIALITERAСY_INDEX-DM_23-1-final-eng.pdf
18-04-UA_REPORT_MEDIALITERAСY_INDEX-DM_23-1-final-eng.pdf
 
SOCIAL AND HISTORICAL CONTEXT - LFTVD.pptx
SOCIAL AND HISTORICAL CONTEXT - LFTVD.pptxSOCIAL AND HISTORICAL CONTEXT - LFTVD.pptx
SOCIAL AND HISTORICAL CONTEXT - LFTVD.pptx
 
Hybridoma Technology ( Production , Purification , and Application )
Hybridoma Technology  ( Production , Purification , and Application  ) Hybridoma Technology  ( Production , Purification , and Application  )
Hybridoma Technology ( Production , Purification , and Application )
 
BASLIQ CURRENT LOOKBOOK LOOKBOOK(1) (1).pdf
BASLIQ CURRENT LOOKBOOK  LOOKBOOK(1) (1).pdfBASLIQ CURRENT LOOKBOOK  LOOKBOOK(1) (1).pdf
BASLIQ CURRENT LOOKBOOK LOOKBOOK(1) (1).pdf
 
Presiding Officer Training module 2024 lok sabha elections
Presiding Officer Training module 2024 lok sabha electionsPresiding Officer Training module 2024 lok sabha elections
Presiding Officer Training module 2024 lok sabha elections
 
Presentation by Andreas Schleicher Tackling the School Absenteeism Crisis 30 ...
Presentation by Andreas Schleicher Tackling the School Absenteeism Crisis 30 ...Presentation by Andreas Schleicher Tackling the School Absenteeism Crisis 30 ...
Presentation by Andreas Schleicher Tackling the School Absenteeism Crisis 30 ...
 
Employee wellbeing at the workplace.pptx
Employee wellbeing at the workplace.pptxEmployee wellbeing at the workplace.pptx
Employee wellbeing at the workplace.pptx
 
mini mental status format.docx
mini    mental       status     format.docxmini    mental       status     format.docx
mini mental status format.docx
 
Sanyam Choudhary Chemistry practical.pdf
Sanyam Choudhary Chemistry practical.pdfSanyam Choudhary Chemistry practical.pdf
Sanyam Choudhary Chemistry practical.pdf
 

Fundamentals_of_Electric_Circuits_4th_Ed.pdf

  • 1.
  • 2. PRACTICAL APPLICATIONS Each chapter devotes material to practical applications of the concepts covered in Fundamentals of Electric Circuits to help the reader apply the concepts to real-life situations. Here is a sampling of the practical applications found in the text: • Rechargeable flashlight battery (Problem 1.11) • Cost of operating toaster (Problem 1.25) • Potentiometer (Section 2.8) • Design a lighting system (Problem 2.61) • Reading a voltmeter (Problem 2.66) • Controlling speed of a motor (Problem 2.74) • Electric pencil sharpener (Problem 2.78) • Calculate voltage of transistor (Problem 3.86) • Transducer modeling (Problem 4.87) • Strain gauge (Problem 4.90) • Wheatstone bridge (Problem 4.91) • Design a six-bit DAC (Problem 5.83) • Instrumentation amplifier (Problem 5.88) • Design an analog computer circuit (Example 6.15) • Design an op amp circuit (Problem 6.71) • Design analog computer to solve differential equation (Problem 6.79) • Electric power plant substation—capacitor bank (Problem 6.83) • Electronic photo flash unit (Section 7.9) • Automobile ignition circuit (Section 7.9) • Welding machine (Problem 7.86) • Airbag igniter (Problem 8.78) • Electrical analog to bodily functions—study of convulsions (Problem 8.82) • Electronic sensing device (Problem 9.87) • Power transmission system (Problem 9.93) • Design a Colpitts oscillator (Problem 10.94) • Stereo amplifier circuit (Problem 13.85) • Gyrator circuit (Problem 16.69) • Calculate number of stations allowable in AM broadcast band (Problem 18.63) • Voice signal—Nyquist rate (Problem 18.65) ale29559_IFC.qxd 07/11/2008 07:40 PM Page 2
  • 3. COMPUTER TOOLS promote flexibility and meet ABET requirements • PSpice is introduced in Chapter 3 and appears in special sections throughout the text. Appendix D serves as a tutorial on PSpice for Windows for readers not familiar with its use. The special sections contain exam- ples and practice problems using PSpice. Additional homework problems at the end of each chapter also provide an opportunity to use PSpice. • MATLAB® is introduced through a tutorial in Appendix E to show its usage in circuit analysis. A number of examples and practice problems are presented throughout the book in a manner that will allow the student to develop a facility with this powerful tool. A number of end-of-chapter problems will aid in understanding how to effectively use MATLAB. • KCIDE for Circuits is a working software environment developed at Cleveland State University. It is designed to help the student work through circuit problems in an organized manner following the process on problem-solving discussed in Section 1.8. Appendix F contains a description of how to use the software. Additional examples can be found at the web site, http://kcide.fennresearch.org/. The actual software pack- age can be downloaded for free from this site. One of the best benefits from using this package is that it automatically generates a Word document and/or a PowerPoint presentation. CAREERS AND HISTORY of electrical engineering pioneers Since a course in circuit analysis may be a student’s first exposure to electrical engineering, each chapter opens with discussions about how to enhance skills that contribute to successful problem-solving or career-oriented talks on a sub-discipline of electrical engineering. The chapter openers are intended to help students grasp the scope of electrical engineering and give thought to the various careers available to EE graduates. The open- ing boxes include information on careers in electronics, instrumentation, electromagnetics, control systems, engineering education, and the importance of good communication skills. Historicals throughout the text provide brief biological sketches of such engineering pioneers as Faraday, Ampere, Edison, Henry, Fourier, Volta, and Bell. ale29559_IFC.qxd 07/11/2008 07:40 PM Page 3
  • 4. OUR COMMITMENT TO ACCURACY You have a right to expect an accurate textbook, and McGraw-Hill Engineering invests considerable time and effort to ensure that we deliver one. Listed below are the many steps we take in this process. OUR ACCURACY VERIFICATION PROCESS First Round Step 1: Numerous college engineering instructors review the manuscript and report errors to the editorial team. The authors review their comments and make the necessary corrections in their manuscript. Second Round Step 2: An expert in the field works through every example and exercise in the final manuscript to verify the accuracy of the examples, exercises, and solutions. The authors review any resulting corrections and incorporate them into the final manuscript and solu- tions manual. Step 3: The manuscript goes to a copyeditor, who reviews the pages for grammatical and stylistic considerations. At the same time, the expert in the field begins a second accuracy check. All corrections are submitted simultaneously to the authors, who review and inte- grate the editing, and then submit the manuscript pages for typesetting. Third Round Step 4: The authors review their page proofs for a dual purpose: 1) to make certain that any previous corrections were properly made, and 2) to look for any errors they might have missed. Step 5: A proofreader is assigned to the project to examine the new page proofs, double check the authors' work, and add a fresh, critical eye to the book. Revisions are incorpo- rated into a new batch of pages which the authors check again. Fourth Round Step 6: The author team submits the solutions manual to the expert in the field, who checks text pages against the solutions manual as a final review. Step 7: The project manager, editorial team, and author team review the pages for a final accuracy check. The resulting engineering textbook has gone through several layers of quality assurance and is verified to be as accurate and error-free as possible. Our authors and publishing staff are confident that through this process we deliver textbooks that are industry leaders in their correctness and technical integrity. ale29559_IFC.qxd 07/11/2008 07:40 PM Page 4
  • 7. f o u r t h e d i t i o n Fundamentals of Electric Circuits Charles K. Alexander Department of Electrical and Computer Engineering Cleveland State University Matthew N. O. Sadiku Department of Electrical Engineering Prairie View A&M University ale29559_fm.qxd 07/28/2008 11:54 AM Page iii
  • 8. FUNDAMENTALS OF ELECTRIC CIRCUITS, FOURTH EDITION Published by McGraw-Hill, a business unit of The McGraw-Hill Companies, Inc., 1221 Avenue of the Americas, New York, NY 10020. Copyright © 2009 by The McGraw-Hill Companies, Inc. All rights reserved. Previous editions © 2007, 2004, and 2000. No part of this publication may be reproduced or distributed in any form or by any means, or stored in a database or retrieval system, without the prior written consent of The McGraw-Hill Companies, Inc., including, but not limited to, in any network or other electronic storage or transmission, or broadcast for distance learning. Some ancillaries, including electronic and print components, may not be available to customers outside the United States. This book is printed on acid-free paper. 1 2 3 4 5 6 7 8 9 0 VNH/VNH 0 9 8 ISBN 978–0–07–352955–4 MHID 0–07–352955–9 Global Publisher: Raghothaman Srinivasan Director of Development: Kristine Tibbetts Developmental Editor: Lora Neyens Senior Marketing Manager: Curt Reynolds Project Manager: Joyce Watters Senior Production Supervisor: Sherry L. Kane Lead Media Project Manager: Stacy A. Patch Associate Design Coordinator: Brenda A. Rolwes Cover Designer: Studio Montage, St. Louis, Missouri (USE) Cover Image: Astronauts Repairing Spacecraft: © StockTrek/Getty Images; Printed Circuit Board: Photodisc Collection/Getty Images Lead Photo Research Coordinator: Carrie K. Burger Compositor: ICC Macmillan Inc. Typeface: 10/12 Times Roman Printer: R. R. Donnelley, Jefferson City, MO Library of Congress Cataloging-in-Publication Data Alexander, Charles K. Fundamentals of electric circuits / Charles K. Alexander, Matthew N. O. Sadiku. — 4th ed. p. cm. Includes index. ISBN 978–0–07–352955–4 — ISBN 0–07–352955–9 (hard copy : alk. paper) 1. Electric circuits. I. Sadiku, Matthew N. O. II. Title. TK454.A452 2009 621.319'24—dc22 2008023020 www.mhhe.com ale29559_fm.qxd 07/28/2008 11:54 AM Page iv
  • 9. Dedicated to our wives, Kikelomo and Hannah, whose understanding and support have truly made this book possible. Matthew and Chuck ale29559_fm.qxd 07/28/2008 11:54 AM Page v
  • 11. Contents Preface xiii Acknowledgments xviii Guided Tour xx A Note to the Student xxv About the Authors xxvii PART 1 DC Circuits 2 Chapter 1 Basic Concepts 3 1.1 Introduction 4 1.2 Systems of Units 4 1.3 Charge and Current 6 1.4 Voltage 9 1.5 Power and Energy 10 1.6 Circuit Elements 15 1.7 † Applications 17 1.7.1 TV Picture Tube 1.7.2 Electricity Bills 1.8 † Problem Solving 20 1.9 Summary 23 Review Questions 24 Problems 24 Comprehensive Problems 27 Chapter 2 Basic Laws 29 2.1 Introduction 30 2.2 Ohm’s Law 30 2.3 † Nodes, Branches, and Loops 35 2.4 Kirchhoff’s Laws 37 2.5 Series Resistors and Voltage Division 43 2.6 Parallel Resistors and Current Division 45 2.7 † Wye-Delta Transformations 52 2.8 † Applications 58 2.8.1 Lighting Systems 2.8.2 Design of DC Meters 2.9 Summary 64 Review Questions 66 Problems 67 Comprehensive Problems 78 vii Chapter 3 Methods of Analysis 81 3.1 Introduction 82 3.2 Nodal Analysis 82 3.3 Nodal Analysis with Voltage Sources 88 3.4 Mesh Analysis 93 3.5 Mesh Analysis with Current Sources 98 3.6 † Nodal and Mesh Analyses by Inspection 100 3.7 Nodal Versus Mesh Analysis 104 3.8 Circuit Analysis with PSpice 105 3.9 † Applications: DC Transistor Circuits 107 3.10 Summary 112 Review Questions 113 Problems 114 Comprehensive Problem 126 Chapter 4 Circuit Theorems 127 4.1 Introduction 128 4.2 Linearity Property 128 4.3 Superposition 130 4.4 Source Transformation 135 4.5 Thevenin’s Theorem 139 4.6 Norton’s Theorem 145 4.7 † Derivations of Thevenin’s and Norton’s Theorems 149 4.8 Maximum Power Transfer 150 4.9 Verifying Circuit Theorems with PSpice 152 4.10 † Applications 155 4.10.1 Source Modeling 4.10.2 Resistance Measurement 4.11 Summary 160 Review Questions 161 Problems 162 Comprehensive Problems 173 Chapter 5 Operational Amplifiers 175 5.1 Introduction 176 5.2 Operational Amplifiers 176 ale29559_fm.qxd 07/28/2008 11:54 AM Page vii
  • 12. viii Contents 5.3 Ideal Op Amp 179 5.4 Inverting Amplifier 181 5.5 Noninverting Amplifier 183 5.6 Summing Amplifier 185 5.7 Difference Amplifier 187 5.8 Cascaded Op Amp Circuits 191 5.9 Op Amp Circuit Analysis with PSpice 194 5.10 † Applications 196 5.10.1 Digital-to-Analog Converter 5.10.2 Instrumentation Amplifiers 5.11 Summary 199 Review Questions 201 Problems 202 Comprehensive Problems 213 Chapter 6 Capacitors and Inductors 215 6.1 Introduction 216 6.2 Capacitors 216 6.3 Series and Parallel Capacitors 222 6.4 Inductors 226 6.5 Series and Parallel Inductors 230 6.6 † Applications 233 6.6.1 Integrator 6.6.2 Differentiator 6.6.3 Analog Computer 6.7 Summary 240 Review Questions 241 Problems 242 Comprehensive Problems 251 Chapter 7 First-Order Circuits 253 7.1 Introduction 254 7.2 The Source-Free RC Circuit 254 7.3 The Source-Free RL Circuit 259 7.4 Singularity Functions 265 7.5 Step Response of an RC Circuit 273 7.6 Step Response of an RL Circuit 280 7.7 † First-Order Op Amp Circuits 284 7.8 Transient Analysis with PSpice 289 7.9 † Applications 293 7.9.1 Delay Circuits 7.9.2 Photoflash Unit 7.9.3 Relay Circuits 7.9.4 Automobile Ignition Circuit 7.10 Summary 299 Review Questions 300 Problems 301 Comprehensive Problems 311 Chapter 8 Second-Order Circuits 313 8.1 Introduction 314 8.2 Finding Initial and Final Values 314 8.3 The Source-Free Series RLC Circuit 319 8.4 The Source-Free Parallel RLC Circuit 326 8.5 Step Response of a Series RLC Circuit 331 8.6 Step Response of a Parallel RLC Circuit 336 8.7 General Second-Order Circuits 339 8.8 Second-Order Op Amp Circuits 344 8.9 PSpice Analysis of RLC Circuits 346 8.10 † Duality 350 8.11 † Applications 353 8.11.1 Automobile Ignition System 8.11.2 Smoothing Circuits 8.12 Summary 356 Review Questions 357 Problems 358 Comprehensive Problems 367 PART 2 AC Circuits 368 Chapter 9 Sinusoids and Phasors 369 9.1 Introduction 370 9.2 Sinusoids 371 9.3 Phasors 376 9.4 Phasor Relationships for Circuit Elements 385 9.5 Impedance and Admittance 387 9.6 † Kirchhoff’s Laws in the Frequency Domain 389 9.7 Impedance Combinations 390 9.8 † Applications 396 9.8.1 Phase-Shifters 9.8.2 AC Bridges 9.9 Summary 402 Review Questions 403 Problems 403 Comprehensive Problems 411 Chapter 10 Sinusoidal Steady-State Analysis 413 10.1 Introduction 414 10.2 Nodal Analysis 414 10.3 Mesh Analysis 417 ale29559_fm.qxd 07/28/2008 11:54 AM Page viii
  • 13. Contents ix 12.11 Summary 543 Review Questions 543 Problems 544 Comprehensive Problems 553 Chapter 13 Magnetically Coupled Circuits 555 13.1 Introduction 556 13.2 Mutual Inductance 557 13.3 Energy in a Coupled Circuit 564 13.4 Linear Transformers 567 13.5 Ideal Transformers 573 13.6 Ideal Autotransformers 581 13.7 † Three-Phase Transformers 584 13.8 PSpice Analysis of Magnetically Coupled Circuits 586 13.9 † Applications 591 13.9.1 Transformer as an Isolation Device 13.9.2 Transformer as a Matching Device 13.9.3 Power Distribution 13.10 Summary 597 Review Questions 598 Problems 599 Comprehensive Problems 611 Chapter 14 Frequency Response 613 14.1 Introduction 614 14.2 Transfer Function 614 14.3 † The Decibel Scale 617 14.4 Bode Plots 619 14.5 Series Resonance 629 14.6 Parallel Resonance 634 14.7 Passive Filters 637 14.7.1 Lowpass Filter 14.7.2 Highpass Filter 14.7.3 Bandpass Filter 14.7.4 Bandstop Filter 14.8 Active Filters 642 14.8.1 First-Order Lowpass Filter 14.8.2 First-Order Highpass Filter 14.8.3 Bandpass Filter 14.8.4 Bandreject (or Notch) Filter 14.9 Scaling 648 14.9.1 Magnitude Scaling 14.9.2 Frequency Scaling 14.9.3 Magnitude and Frequency Scaling 14.10 Frequency Response Using PSpice 652 14.11 Computation Using MATLAB 655 10.4 Superposition Theorem 421 10.5 Source Transformation 424 10.6 Thevenin and Norton Equivalent Circuits 426 10.7 Op Amp AC Circuits 431 10.8 AC Analysis Using PSpice 433 10.9 † Applications 437 10.9.1 Capacitance Multiplier 10.9.2 Oscillators 10.10 Summary 441 Review Questions 441 Problems 443 Chapter 11 AC Power Analysis 457 11.1 Introduction 458 11.2 Instantaneous and Average Power 458 11.3 Maximum Average Power Transfer 464 11.4 Effective or RMS Value 467 11.5 Apparent Power and Power Factor 470 11.6 Complex Power 473 11.7 † Conservation of AC Power 477 11.8 Power Factor Correction 481 11.9 † Applications 483 11.9.1 Power Measurement 11.9.2 Electricity Consumption Cost 11.10 Summary 488 Review Questions 490 Problems 490 Comprehensive Problems 500 Chapter 12 Three-Phase Circuits 503 12.1 Introduction 504 12.2 Balanced Three-Phase Voltages 505 12.3 Balanced Wye-Wye Connection 509 12.4 Balanced Wye-Delta Connection 512 12.5 Balanced Delta-Delta Connection 514 12.6 Balanced Delta-Wye Connection 516 12.7 Power in a Balanced System 519 12.8 † Unbalanced Three-Phase Systems 525 12.9 PSpice for Three-Phase Circuits 529 12.10 † Applications 534 12.10.1 Three-Phase Power Measurement 12.10.2 Residential Wiring ale29559_fm.qxd 07/28/2008 11:54 AM Page ix
  • 14. x Contents 14.12 † Applications 657 14.12.1 Radio Receiver 14.12.2 Touch-Tone Telephone 14.12.3 Crossover Network 14.13 Summary 663 Review Questions 664 Problems 665 Comprehensive Problems 673 PART 3 Advanced Circuit Analysis 674 Chapter 15 Introduction to the Laplace Transform 675 15.1 Introduction 676 15.2 Definition of the Laplace Transform 677 15.3 Properties of the Laplace Transform 679 15.4 The Inverse Laplace Transform 690 15.4.1 Simple Poles 15.4.2 Repeated Poles 15.4.3 Complex Poles 15.5 The Convolution Integral 697 15.6 † Application to Integrodifferential Equations 705 15.7 Summary 708 Review Questions 708 Problems 709 Chapter 16 Applications of the Laplace Transform 715 16.1 Introduction 716 16.2 Circuit Element Models 716 16.3 Circuit Analysis 722 16.4 Transfer Functions 726 16.5 State Variables 730 16.6 † Applications 737 16.6.1 Network Stability 16.6.2 Network Synthesis 16.7 Summary 745 Review Questions 746 Problems 747 Comprehensive Problems 754 Chapter 17 The Fourier Series 755 17.1 Introduction 756 17.2 Trigonometric Fourier Series 756 17.3 Symmetry Considerations 764 17.3.1 Even Symmetry 17.3.2 Odd Symmetry 17.3.3 Half-Wave Symmetry 17.4 Circuit Applications 774 17.5 Average Power and RMS Values 778 17.6 Exponential Fourier Series 781 17.7 Fourier Analysis with PSpice 787 17.7.1 Discrete Fourier Transform 17.7.2 Fast Fourier Transform 17.8 † Applications 793 17.8.1 Spectrum Analyzers 17.8.2 Filters 17.9 Summary 796 Review Questions 798 Problems 798 Comprehensive Problems 807 Chapter 18 Fourier Transform 809 18.1 Introduction 810 18.2 Definition of the Fourier Transform 810 18.3 Properties of the Fourier Transform 816 18.4 Circuit Applications 829 18.5 Parseval’s Theorem 832 18.6 Comparing the Fourier and Laplace Transforms 835 18.7 † Applications 836 18.7.1 Amplitude Modulation 18.7.2 Sampling 18.8 Summary 839 Review Questions 840 Problems 841 Comprehensive Problems 847 Chapter 19 Two-Port Networks 849 19.1 Introduction 850 19.2 Impedance Parameters 850 19.3 Admittance Parameters 855 19.4 Hybrid Parameters 858 19.5 Transmission Parameters 863 19.6 † Relationships Between Parameters 868 19.7 Interconnection of Networks 871 19.8 Computing Two-Port Parameters Using PSpice 877 19.9 † Applications 880 19.9.1 Transistor Circuits 19.9.2 Ladder Network Synthesis ale29559_fm.qxd 07/28/2008 11:54 AM Page x
  • 15. Contents xi 19.10 Summary 889 Review Questions 890 Problems 890 Comprehensive Problems 901 Appendix A Simultaneous Equations and Matrix Inversion A Appendix B Complex Numbers A-9 Appendix C Mathematical Formulas A-16 Appendix D PSpice for Windows A-21 Appendix E MATLAB A-46 Appendix F KCIDE for Circuits A-65 Appendix G Answers to Odd-Numbered Problems A-75 Selected Bibliography B-1 Index I-1 ale29559_fm.qxd 07/28/2008 11:54 AM Page xi
  • 17. xiii Preface You may be wondering why we chose a photo of astronauts working in space on the Space Station for the cover. We actually chose it for several reasons. Obviously, it is very exciting; in fact, space represents the most exciting frontier for the entire world! In addition, much of the station itself consists of all kinds of circuits! One of the most signifi- cant circuits within the station is its power distribution system. It is a complete and self contained, modern power generation and distribution system. That is why NASA (especially NASA-Glenn) continues to be at the forefront of both theoretical as well as applied power system research and development. The technology that has gone into the devel- opment of space exploration continues to find itself impacting terres- trial technology in many important ways. For some of you, this will be an important career path. FEATURES New to This Edition A course in circuit analysis is perhaps the first exposure students have to electrical engineering. This is also a place where we can enhance some of the skills that they will later need as they learn how to design. In the fourth edition, we have included a very significant new feature to help students enhance skills that are an important part of the design process. We call this new feature, design a problem. We know it is not possible to fully develop a student’s design skills in a fundamental course like circuits. To fully develop design skills a student needs a design experience normally reserved for their senior year. This does not mean that some of those skills cannot be developed and exercised in a circuits course. The text already included open- ended questions that help students use creativity, which is an impor- tant part of learning how to design. We already have some questions that are open desired to add much more into our text in this important area and have developed an approach to do just that. When we develop problems for the student to solve our goal is that in solving the prob- lem the student learn more about the theory and the problem solving process. Why not have the students design problems like we do? That is exactly what we will do in each chapter. Within the normal problem set, we have a set of problems where we ask the student to design a problem. This will have two very important results. The first will be a better understanding of the basic theory and the second will be the enhancement of some of the student’s basic design skills. We are making effective use of the principle of learning by teach- ing. Essentially we all learn better when we teach a subject. Design- ing effective problems is a key part of the teaching process. Students ale29559_fm.qxd 08/08/2008 10:16 AM Page xiii
  • 18. xiv Preface should also be encouraged to develop problems, when appropriate, which have nice numbers and do not necessarily overemphasize com- plicated mathematical manipulations. Additionally we have changed almost 40% of the Practice Prob- lems with the idea to better reflect more real component values and to help the student better understand the problem and have added 121 design a problem problems. We have also changed and added a total of 357 end-of-chapter problems (this number contains the new design a problem problems). This brings up a very important advantage to our textbook, we have a total of 2404 Examples, Practice Problems, Review Questions, and end-of-chapter problems! Retained from Previous Editions The main objective of the fourth edition of this book remains the same as the previous editions—to present circuit analysis in a manner that is clearer, more interesting, and easier to understand than other circuit text, and to assist the student in beginning to see the “fun” in engineering. This objective is achieved in the following ways: • Chapter Openers and Summaries Each chapter opens with a discussion about how to enhance skills which contribute to successful problem solving as well as suc- cessful careers or a career-oriented talk on a sub-discipline of elec- trical engineering. This is followed by an introduction that links the chapter with the previous chapters and states the chapter objectives. The chapter ends with a summary of key points and formulas. • Problem Solving Methodology Chapter 1 introduces a six-step method for solving circuit problems which is used consistently throughout the book and media supple- ments to promote best-practice problem-solving procedures. • Student Friendly Writing Style All principles are presented in a lucid, logical, step-by-step manner. As much as possible, we avoid wordiness and giving too much detail that could hide concepts and impede overall understanding of the material. • Boxed Formulas and Key Terms Important formulas are boxed as a means of helping students sort out what is essential from what is not. Also, to ensure that students clearly understand the key elements of the subject matter, key terms are defined and highlighted. • Margin Notes Marginal notes are used as a pedagogical aid. They serve multiple uses such as hints, cross-references, more exposition, warnings, reminders not to make some particular common mistakes, and problem-solving insights. • Worked Examples Thoroughly worked examples are liberally given at the end of every section. The examples are regarded as a part of the text and are clearly explained without asking the reader to fill in missing steps. Thoroughly worked examples give students a good understand- ing of the solution process and the confidence to solve problems ale29559_fm.qxd 07/28/2008 11:54 AM Page xiv
  • 19. Preface xv themselves. Some of the problems are solved in two or three dif- ferent ways to facilitate a substantial comprehension of the subject material as well as a comparison of different approaches. • Practice Problems To give students practice opportunity, each illustrative example is immediately followed by a practice problem with the answer. The student can follow the example step by step to aid in the solution of the practice problem without flipping pages or looking at the end of the book for answers. The practice problem is also intended to test a student’s understanding of the preceding example. It will reinforce their grasp of the material before the student can move on to the next section. Complete solutions to the practice problems are available to students on ARIS. • Application Sections The last section in each chapter is devoted to practical application aspects of the concepts covered in the chapter. The material cov- ered in the chapter is applied to at least one or two practical prob- lems or devices. This helps students see how the concepts are applied to real-life situations. • Review Questions Ten review questions in the form of multiple-choice objective items are provided at the end of each chapter with answers. The review questions are intended to cover the little “tricks” that the examples and end-of-chapter problems may not cover. They serve as a self- test device and help students determine how well they have mas- tered the chapter. • Computer Tools In recognition of the requirements by ABET® on integrating com- puter tools, the use of PSpice, MATLAB, KCIDE for Circuits, and developing design skills are encouraged in a student-friendly man- ner. PSpice is covered early on in the text so that students can become familiar and use it throughout the text. Appendix D serves as a tutorial on PSpice for Windows. MATLAB is also introduced early in the book with a tutorial available in Appendix E. KCIDE for Circuits is a brand new, state-of-the-art software system designed to help the students maximize their chance of success in problem solving. It is introduced in Appendix F. Finally, design a problem problems have been introduced, for the first time. These are meant to help the student develop skills that will be needed in the design process. • Historical Tidbits Historical sketches throughout the text provide profiles of important pioneers and events relevant to the study of electrical engineering. • Early Op Amp Discussion The operational amplifier (op amp) as a basic element is introduced early in the text. • Fourier and Laplace Transforms Coverage To ease the transition between the circuit course and signals and systems courses, Fourier and Laplace transforms are covered lucidly and thoroughly. The chapters are developed in a manner that the interested instructor can go from solutions of first-order ale29559_fm.qxd 07/28/2008 11:54 AM Page xv
  • 20. circuits to Chapter 15. This then allows a very natural progression from Laplace to Fourier to AC. • Four Color Art Program A completely redesigned interior design and four color art program bring circuit drawings to life and enhance key pedagogical ele- ments throughout the text. • Extended Examples Examples worked in detail according to the six-step problem solv- ing method provide a roadmap for students to solve problems in a consistent fashion. At least one example in each chapter is devel- oped in this manner. • EC 2000 Chapter Openers Based on ABET’s new skill-based CRITERION 3, these chapter openers are devoted to discussions as to how students can acquire the skills that will lead to a significantly enhanced career as an engineer. Because these skills are so very important to the student while in college as well as in their career, we will use the head- ing, “Enhancing your Skills and your Career.” • Homework Problems There are 358 new or changed end-of-chapter problems which will provide students with plenty of practice as well as reinforce key concepts. • Homework Problem Icons Icons are used to highlight problems that relate to engineering design as well as problems that can be solved using PSpice or MATLAB. • KCIDE for Circuits Appendix F A new Appendix F provides a tutorial on the Knowledge Captur- ing Integrated Design Environment (KCIDE for Circuits) software, available on ARIS. Organization This book was written for a two-semester or three-quarter course in linear circuit analysis. The book may also be used for a one-semester course by a proper selection of chapters and sections by the instructor. It is broadly divided into three parts. • Part 1, consisting of Chapters 1 to 8, is devoted to dc circuits. It covers the fundamental laws and theorems, circuits techniques, and passive and active elements. • Part 2, which contains Chapter 9 to 14, deals with ac circuits. It introduces phasors, sinusoidal steady-state analysis, ac power, rms values, three-phase systems, and frequency response. • Part 3, consisting of Chapters 15 to 19, is devoted to advanced techniques for network analysis. It provides students with a solid introduction to the Laplace transform, Fourier series, Fourier trans- form, and two-port network analysis. The material in three parts is more than sufficient for a two-semester course, so the instructor must select which chapters or sections to cover. Sections marked with the dagger sign (†) may be skipped, explained briefly, or assigned as homework. They can be omitted without loss of xvi Preface ale29559_fm.qxd 07/28/2008 11:54 AM Page xvi
  • 21. Preface xvii continuity. Each chapter has plenty of problems grouped according to the sections of the related material and diverse enough that the instruc- tor can choose some as examples and assign some as homework. As stated earlier, we are using three icons with this edition. We are using to denote problems that either require PSpice in the solution process, where the circuit complexity is such that PSpice would make the solu- tion process easier, and where PSpice makes a good check to see if the problem has been solved correctly. We are using to denote problems where MATLAB is required in the solution process, where MATLAB makes sense because of the problem makeup and its complexity, and where MATLAB makes a good check to see if the problem has been solved correctly. Finally, we use to identify problems that help the student develop skills that are needed for engineering design. More dif- ficult problems are marked with an asterisk (*). Comprehensive prob- lems follow the end-of-chapter problems. They are mostly applications problems that require skills learned from that particular chapter. Prerequisites As with most introductory circuit courses, the main prerequisites, for a course using the text, are physics and calculus. Although familiarity with complex numbers is helpful in the later part of the book, it is not required. A very important asset of this text is that ALL the mathe- matical equations and fundamentals of physics needed by the student, are included in the text. Supplements McGraw-Hill’s ARIS—Assessment, Review, and Instruction System is a complete, online tutorial, electronic homework, and course management system, designed for greater ease of use than any other system available. Available on adoption, instructors can create and share course materials and assignments with other instructors, edit ques- tions and algorithms, import their own content, and create announce- ments and due dates for assignments. ARIS has automatic grading and reporting of easy-to-assign algorithmically-generated homework, quizzing, and testing. Once a student is registered in the course, all stu- dent activity within McGraw-Hill’s ARIS is automatically recorded and available to the instructor through a fully integrated grade book that can be downloaded to Excel. Also included on ARIS are a solutions man- ual, text image files, transition guides to instructors, and Network Analysis Tutorials, software downloads, complete solutions to text practice problems, FE Exam questions, flashcards, and web links to stu- dents. Visit www.mhhe.com/alexander. Knowledge Capturing Integrated Design Environment for Circuits (KCIDE for Circuits) This new software, developed at Cleveland State University and funded by NASA, is designed to help the student work through a circuits problem in an organized manner using the six-step problem-solving methodology in the text. KCIDE for Circuits allows students to work a circuit problem in PSpice and MATLAB, track the ale29559_fm.qxd 07/28/2008 11:54 AM Page xvii
  • 22. xviii Preface evolution of their solution, and save a record of their process for future reference. In addition, the software automatically generates a Word document and/or a PowerPoint presentation. Appendix F contains a description of how to use the software. Additional examples can be found at the web site, http://kcide.fennresearch.org/, which is linked from ARIS. The software package can be downloaded for free. Problem Solving Made Almost Easy, a companion workbook to Fun- damentals of Electric Circuits, is available on ARIS for students who wish to practice their problem-solving techniques. The workbook con- tains a discussion of problem-solving strategies and 150 additional problems with complete solutions provided. C.O.S.M.O.S This CD, available to instructors only, is a powerful solu- tions manual tool to help instructors streamline the creation of assign- ments, quizzes, and tests by using problems and solutions from the textbook, as well as their own custom material. Instructors can edit textbook end-of-chapter problems as well as track which problems have been assigned. Although the textbook is meant to be self-explanatory and act as a tutor for the student, the personal contact in teaching is not forgot- ten. It is hoped that the book and supplemental materials supply the instructor with all the pedagogical tools necessary to effectively pres- ent the material. Acknowledgements We would like to express our appreciation for the loving support we have received from our wives (Hannah and Kikelomo), daughters (Christina, Tamara, Jennifer, Motunrayo, Ann, and Joyce), son (Baixi), and our extended family members. At McGraw-Hill, we would like to thank the following editorial and production staff: Raghu Srinivasan, publisher and senior sponsor- ing editor; Lora Kalb-Neyens, developmental editors; Joyce Watters, project manager; Carrie Burger, photo researcher; and Brenda Rolwes, designer. Also, we appreciate the hard work of Tom Hartley at the Uni- versity of Akron for his very detailed evaluation of various elements of the text and his many valued suggestions for continued improve- ment of this textbook. We wish to thank Yongjian Fu and his outstanding team of stu- dents, Bramarambha Elka and Saravaran Chinniah, for their efforts in the development of KCIDE for Circuits. Their efforts to help us con- tinue to improve this software are also appreciated. The fourth edition has benefited greatly from the many outstand- ing reviewers and symposium attendees who contributed to the success of the first three editions! In addition, the following have made impor- tant contributions to the fourth edition (in alphabetical order): Tom Brewer, Georgia Tech Andy Chan, City University of Hong Kong Alan Tan Wee Chiat, Multimedia University Norman Cox, University of Missouri-Rolla Walter L. Green, University of Tennessee ale29559_fm.qxd 07/28/2008 11:54 AM Page xviii
  • 23. Preface xix Dr. Gordon K. Lee, San Diego State University Gary Perks, Cal Poly State University, San Luis Obispo Dr. Raghu K. Settaluri, Oregon State University Ramakant Srivastava, University of Florida John Watkins, Wichita State University Yik-Chung Wu, The University of Hong Kong Xiao-Bang Xu, Clemson University Finally, we appreciate the feedback received from instructors and stu- dents who used the previous editions. We want this to continue, so please keep sending us emails or direct them to the publisher. We can be reached at c.alexander@ieee.org for Charles Alexander and sadiku@ieee.org for Matthew Sadiku. C. K. Alexander and M.N.O. Sadiku ale29559_fm.qxd 07/28/2008 11:54 AM Page xix
  • 24. xx GUIDED TOUR The main objective of this book is to present circuit analysis in a man- ner that is clearer, more interesting, and easier to understand than other texts. For you, the student, here are some features to help you study and be successful in this course. The four color art program brings circuit draw- ings to life and enhances key concepts throughout the text. 1.5 Power and Energy 11 To relate power and energy to voltage and current, we recall from physics that: Power is the time rate of expending or absorbing energy, measured in watts (W). We write this relationship as (1.5) where p is power in watts (W), w is energy in joules (J), and t is time in seconds (s). From Eqs. (1.1), (1.3), and (1.5), it follows that (1.6) or (1.7) The power p in Eq. (1.7) is a time-varying quantity and is called the instantaneous power. Thus, the power absorbed or supplied by an ele- ment is the product of the voltage across the element and the current through it. If the power has a sign, power is being delivered to or absorbed by the element. If, on the other hand, the power has a sign, power is being supplied by the element. But how do we know when the power has a negative or a positive sign? Current direction and voltage polarity play a major role in deter- mining the sign of power. It is therefore important that we pay atten- tion to the relationship between current i and voltage v in Fig. 1.8(a). The voltage polarity and current direction must conform with those shown in Fig. 1.8(a) in order for the power to have a positive sign. This is known as the passive sign convention. By the passive sign con- vention, current enters through the positive polarity of the voltage. In this case, or implies that the element is absorbing power. However, if or , as in Fig. 1.8(b), the element is releasing or supplying power. Passive sign convention is satisfied when the current enters through the positive terminal of an element and p vi. If the current enters through the negative terminal, p vi. Unless otherwise stated, we will follow the passive sign conven- tion throughout this text. For example, the element in both circuits of Fig. 1.9 has an absorbing power of W because a positive current enters the positive terminal in both cases. In Fig. 1.10, however, the element is supplying power of W because a positive current enters the negative terminal. Of course, an absorbing power of W is equivalent to a supplying power of W. In general, ⫹Power absorbed ⫽ ⫺Power supplied ⫹12 ⫺12 ⫹12 ⫹12 ⫺ ⫽ ⫹ ⫽ vi 6 0 p ⫽ ⫺vi vi 7 0 p ⫽ ⫹vi ⫺ ⫹ p ⫽ vi p ⫽ dw dt ⫽ dw dq ⴢ dq dt ⫽ vi p ⫽ ¢ dw dt p = +vi (a) v + − p = −vi (b) v + − i i Figure 1.8 Reference polarities for power using the passive sign convention: (a) absorbing power, (b) supplying power. When the voltage and current directions conform to Fig. 1.8(b), we have the ac- tive sign convention and p vi. ⫹ ⫽ (a) 4 V 3 A (a) + − 3 A 4 V 3 A (b) + − Figure 1.9 Two cases of an element with an absorbing power of 12 W: (a) W, (b) W. p ⫽ 4 ⫻ 3 ⫽ 12 p ⫽ 4 ⫻ 3 ⫽ 12 3 A (a) 4 V 3 A (a) + − 3 A 4 V 3 A (b) + − Figure 1.10 Two cases of an element with a supplying power of 12 W: (a) W, (b) W. p ⫽ ⫺4 ⫻ 3 ⫽ ⫺12 ⫺12 p ⫽ ⫺4 ⫻ 3 ⫽ Problem Solving Although the problems to be solved during one’s career will vary in complexity and magnitude, the basic principles to be followed remain the same. The process outlined here is the one developed by the authors over many years of problem solving with students, for the solution of engineering problems in industry, and for problem solving in research. We will list the steps simply and then elaborate on them. 1. Carefully Define the problem. 2. Present everything you know about the problem. 3. Establish a set of Alternative solutions and determine the one that promises the greatest likelihood of success. 4. Attempt a problem solution. 5. Evaluate the solution and check for accuracy. 6. Has the problem been solved Satisfactorily? If so, present the solution; if not, then return to step 3 and continue through the process again. 1. Carefully Define the problem. This may be the most important part of the process, because it becomes the foundation for all the rest of the steps. In general, the presentation of engineering problems is somewhat incomplete. You must do all you can to make sure you understand the problem as thoroughly as the presenter of the problem understands it. Time spent at this point clearly identifying the problem will save you considerable time and frustration later. As a student, you can clarify a problem statement in a textbook by asking your professor. A problem presented to you in industry may require that you consult several indi- viduals. At this step, it is important to develop questions that need to be addressed before continuing the solution process. If you have such questions, you need to consult with the appropriate individuals or resources to obtain the answers to those questions. With those answers, you can now refine the problem, and use that refinement as the prob- lem statement for the rest of the solution process. 2. Present everything you know about the problem. You are now ready to write down everything you know about the problem and its possible solutions. This important step will save you time and frustration later. 3. Establish a set of Alternative solutions and determine the one that promises the greatest likelihood of success. Almost every problem will have a number of possible paths that can lead to a solution. It is highly desirable to identify as many of those paths as possible. At this point, you also need to determine what tools are available to you, such as PSpice and MATLAB and other software packages that can greatly reduce effort and increase accuracy. Again, we want to stress that time spent carefully defining the problem and investigating alternative approaches to its solution will pay big dividends later. Evaluating the alternatives and determining which promises the greatest likelihood of success may be difficult but will be well worth the effort. Document this process well since you will want to come back to it if the first approach does not work. 4. Attempt a problem solution. Now is the time to actually begin solving the problem. The process you follow must be well documented 1.8 20 Chapter 1 Basic Concepts in order to present a detailed solution if successful, and to evaluate the process if you are not successful. This detailed evaluation may lead to corrections that can then lead to a successful solution. It can also lead to new alternatives to try. Many times, it is wise to fully set up a solu- tion before putting numbers into equations. This will help in checking your results. 5. Evaluate the solution and check for accuracy. You now thoroughly evaluate what you have accomplished. Decide if you have an acceptable solution, one that you want to present to your team, boss, or professor. 6. Has the problem been solved Satisfactorily? If so, present the solu- tion; if not, then return to step 3 and continue through the process again. Now you need to present your solution or try another alterna- tive. At this point, presenting your solution may bring closure to the process. Often, however, presentation of a solution leads to further refinement of the problem definition, and the process continues. Fol- lowing this process will eventually lead to a satisfactory conclusion. Now let us look at this process for a student taking an electrical and computer engineering foundations course. (The basic process also applies to almost every engineering course.) Keep in mind that although the steps have been simplified to apply to academic types of problems, the process as stated always needs to be followed. We con- sider a simple example. 1.8 Problem Solving 21 Figure 1.19 Illustrative example. Example 1.10 Solve for the current flowing through the resistor in Fig. 1.19. Solution: 1. Carefully Define the problem. This is only a simple example, but we can already see that we do not know the polarity on the 3-V source. We have the following options. We can ask the professor what the polarity should be. If we cannot ask, then we need to make a decision on what to do next. If we have time to work the problem both ways, we can solve for the current when the 3-V source is plus on top and then plus on the bottom. If we do not have the time to work it both ways, assume a polarity and then carefully document your decision. Let us assume that the professor tells us that the source is plus on the bottom as shown in Fig. 1.20. 2. Present everything you know about the problem. Presenting all that we know about the problem involves labeling the circuit clearly so that we define what we seek. Given the circuit shown in Fig. 1.20, solve for . We now check with the professor, if reasonable, to see if the prob- lem is properly defined. 3. Establish a set of Alternative solutions and determine the one that promises the greatest likelihood of success. There are essentially three techniques that can be used to solve this problem. Later in the text you will see that you can use circuit analysis (using Kirchhoff’s laws and Ohm’s law), nodal analysis, and mesh analysis. To solve for using circuit analysis will eventually lead to a solution, but it will likely take more work than either nodal or mesh i8⍀ i8⍀ 8-⍀ Figure 1.20 Problem defintion. 2 Ω 4 Ω 8 Ω 5 V 3 V + − + − i8Ω 2 Ω 4 Ω 8 Ω 5 V 3 V + − analysis. To solve for using mesh analysis will require writing two simultaneous equations to find the two loop currents indicated in Fig. 1.21. Using nodal analysis requires solving for only one unknown. This is the easiest approach. i8⍀ 22 Chapter 1 Basic Concepts 2 Ω 4 Ω 8 Ω 5 V 3 V + − + − i2 i1 i3 + − + − + − v8Ω v4Ω v2Ω Loop 1 Loop 2 v1 Figure 1.21 Using nodal analysis. Therefore, we will solve for using nodal analysis. 4. Attempt a problem solution. We first write down all of the equa- tions we will need in order to find . Now we can solve for 5. Evaluate the solution and check for accuracy. We can now use Kirchhoff’s voltage law (KVL) to check the results. Applying KVL to loop 1, Applying KVL to loop 2, ⫽ ⫺2 ⫹ 5 ⫺ 3 ⫽ 0 (Checks.) ⫽ ⫺(0.25 ⫻ 8) ⫹ (1.25 ⫻ 4) ⫺ 3 ⫺v8⍀ ⫹ v4⍀ ⫺ 3 ⫽ ⫺(i2 ⫻ 8) ⫹ (i3 ⫻ 4) ⫺ 3 ⫽ ⫺5 ⫹ 3 ⫹ 2 ⫽ 0 (Checks.) ⫽ ⫺5 ⫹ (⫺(⫺1.5)2) ⫹ (0.25 ⫻ 8) ⫺5 ⫹ v2⍀ ⫹ v8⍀ ⫽ ⫺5 ⫹ (⫺i1 ⫻ 2) ⫹ (i2 ⫻ 8) i1 ⫹ i2 ⫹ i3 ⫽ ⫺1.5 ⫹ 0.25 ⫹ 1.25 ⫽ 0 (Checks.) i3 ⫽ v1 ⫹ 3 4 ⫽ 2 ⫹ 3 4 ⫽ 5 4 ⫽ 1.25 A i2 ⫽ i8⍀ ⫽ 0.25 A i1 ⫽ v1 ⫺ 5 2 ⫽ 2 ⫺ 5 2 ⫽ ⫺ 3 2 ⫽ ⫺1.5 A 7v1 ⫽ ⫹14, v1 ⫽ ⫹2 V, i8⍀ ⫽ v1 8 ⫽ 2 8 ⫽ 0.25 A leads to (4v1 ⫺ 20) ⫹ (v1) ⫹ (2v1 ⫹ 6) ⫽ 0 8c v1 ⫺ 5 2 ⫹ v1 ⫺ 0 8 ⫹ v1 ⫹ 3 4 d ⫽ 0 v1. v1 ⫺ 5 2 ⫹ v1 ⫺ 0 8 ⫹ v1 ⫹ 3 4 ⫽ 0 i8⍀ ⫽ i2, i2 ⫽ v1 8 , i8⍀ ⫽ v1 8 i8⍀ i8⍀ So we now have a very high degree of confidence in the accuracy of our answer. 6. Has the problem been solved Satisfactorily? If so, present the solu- tion; if not, then return to step 3 and continue through the process again. This problem has been solved satisfactorily. The current through the 8- resistor is 0.25 A flowing down through the 8- resistor. ⍀ ⍀ 1.9 Summary 23 Try applying this process to some of the more difficult problems at the end of the chapter. Practice Problem 1.10 Summary 1. An electric circuit consists of electrical elements connected together. 2. The International System of Units (SI) is the international mea- surement language, which enables engineers to communicate their results. From the six principal units, the units of other physical quantities can be derived. 3. Current is the rate of charge flow. 4. Voltage is the energy required to move 1 C of charge through an element. 5. Power is the energy supplied or absorbed per unit time. It is also the product of voltage and current. 6. According to the passive sign convention, power assumes a posi- tive sign when the current enters the positive polarity of the voltage across an element. 7. An ideal voltage source produces a specific potential difference across its terminals regardless of what is connected to it. An ideal current source produces a specific current through its terminals regardless of what is connected to it. 8. Voltage and current sources can be dependent or independent. A dependent source is one whose value depends on some other cir- cuit variable. 9. Two areas of application of the concepts covered in this chapter are the TV picture tube and electricity billing procedure. p ⫽ dw dt ⫽ vi v ⫽ dw dq i ⫽ dq dt 1.9 A six-step problem-solving method- ology is introduced in Chapter 1 and incorporated into worked examples throughout the text to promote sound, step-by-step problem-solving practices. ale29559_fm.qxd 07/28/2008 11:54 AM Page xx
  • 25. Guided Tour xxi Each illustrative example is immediately followed by a practice problem and answer to test understanding of the preceding example. PSpice® for Windows is a student-friendly tool intro- duced to students early in the text and used through- out, with discussions and examples at the end of each appropriate chapter. 90 Chapter 3 Methods of Analysis For the circuit shown in Fig. 3.9, find the node voltages. Solution: The supernode contains the 2-V source, nodes 1 and 2, and the 10- resistor. Applying KCL to the supernode as shown in Fig. 3.10(a) gives Expressing and in terms of the node voltages or (3.3.1) To get the relationship between and we apply KVL to the circuit in Fig. 3.10(b). Going around the loop, we obtain (3.3.2) From Eqs. (3.3.1) and (3.3.2), we write or and Note that the 10- resistor does not make any difference because it is connected across the supernode. ⍀ v2 ⫽ v1 ⫹ 2 ⫽ ⫺5.333 V. 3v1 ⫽ ⫺22 1 v1 ⫽ ⫺7.333 V v2 ⫽ v1 ⫹ 2 ⫽ ⫺20 ⫺ 2v1 ⫺v1 ⫺ 2 ⫹ v2 ⫽ 0 1 v2 ⫽ v1 ⫹ 2 v2, v1 v2 ⫽ ⫺20 ⫺ 2v1 2 ⫽ v1 ⫺ 0 2 ⫹ v2 ⫺ 0 4 ⫹ 7 1 8 ⫽ 2v1 ⫹ v2 ⫹ 28 i2 i1 2 ⫽ i1 ⫹ i2 ⫹ 7 ⍀ Example 3.3 Figure 3.9 For Example 3.3. + − 2 A 2 V 7 A 4 Ω 10 Ω 2 Ω v1 v2 2 A 2 A 7 A 7 A 2 Ω 4 Ω v2 v1 i1 i2 1 2 (a) + − (b) 2 V 1 2 + + − − v1 v2 Figure 3.10 Applying: (a) KCL to the supernode, (b) KVL to the loop. Figure 3.11 For Practice Prob. 3.3. 21 V 9 V 4 Ω 3 Ω 2 Ω 6 Ω + − + − i v + − Practice Problem 3.3 Find v and i in the circuit of Fig. 3.11. Answer: 4.2 A. ⫺0.6 V, 106 Chapter 3 Methods of Analysis are displayed on VIEWPOINTS and also saved in output file exam310.out. The output file includes the following: NODE VOLTAGE NODE VOLTAGE NODE VOLTAGE (1) 120.0000 (2) 81.2900 (3) 89.0320 indicating that V3 ⫽ 89.032 V. V2 ⫽ 81.29 V, V1 ⫽ 120 V, + − R1 R3 20 10 120 V V1 R2 R4 30 40 I1 3 A IDC 0 1 2 3 120.0000 81.2900 89.0320 Figure 3.32 For Example 3.10; the schematic of the circuit in Fig. 3.31. For the circuit in Fig. 3.33, use PSpice to find the node voltages. Practice Problem 3.10 + − 2 A 200 V 30 Ω 60 Ω 50 Ω 100 Ω 25 Ω 1 2 3 0 Figure 3.33 For Practice Prob. 3.10. Answer: V3 ⫽ 200 V. V2 ⫽ 57.14 V, V1 ⫽ ⫺40 V, In the circuit of Fig. 3.34, determine the currents and i3. i1, i2, Example 3.11 + − + − 24 V 1 Ω i1 i2 i3 + − 4 Ω 2 Ω 2 Ω 8 Ω 4 Ω 3vo vo Figure 3.34 For Example 3.11. 3.9 Applications: DC Transistor Circuits 107 Solution: The schematic is shown in Fig. 3.35. (The schematic in Fig. 3.35 includes the output results, implying that it is the schematic displayed on the screen after the simulation.) Notice that the voltage-controlled voltage source E1 in Fig. 3.35 is connected so that its input is the voltage across the 4- resistor; its gain is set equal to 3. In order to display the required currents, we insert pseudocomponent IPROBES in the appropriate branches. The schematic is saved as exam311.sch and simulated by selecting Analysis/Simulate. The results are displayed on IPROBES as shown in Fig. 3.35 and saved in output file exam311.out. From the output file or the IPROBES, we obtain A and A. i3 ⫽ 2.667 i1 ⫽ i2 ⫽ 1.333 ⍀ + − 24 V V1 R1 4 R2 2 R3 8 R4 4 1.333E + 00 1.333E + 00 2.667E + 00 0 R6 1 R5 2 E E1 + − − + Figure 3.35 The schematic of the circuit in Fig. 3.34. Use PSpice to determine currents and in the circuit of Fig. 3.36. Answer: A, A, A. i3 ⫽ 2 i2 ⫽ 2.286 i1 ⫽ ⫺0.4286 i3 i1, i2, Practice Problem 3.11 + − 2 A 10 V 2 Ω i1 i1 i2 4 Ω 1 Ω 2 Ω i3 Figure 3.36 For Practice Prob. 3.11. Applications: DC Transistor Circuits Most of us deal with electronic products on a routine basis and have some experience with personal computers. A basic component for the integrated circuits found in these electronics and computers is the active, three-terminal device known as the transistor. Understanding the transistor is essential before an engineer can start an electronic cir- cuit design. Figure 3.37 depicts various kinds of transistors commercially avail- able. There are two basic types of transistors: bipolar junction transis- tors (BJTs) and field-effect transistors (FETs). Here, we consider only the BJTs, which were the first of the two and are still used today. Our objective is to present enough detail about the BJT to enable us to apply the techniques developed in this chapter to analyze dc transistor circuits. p 3.9 369 Sinusoids and Phasors He who knows not, and knows not that he knows not, is a fool— shun him. He who knows not, and knows that he knows not, is a child— teach him. He who knows, and knows not that he knows, is asleep—wake him up. He who knows, and knows that he knows, is wise—follow him. —Persian Proverb c h a p t e r 9 Enhancing Your Skills and Your Career ABET EC 2000 criteria (3.d), “an ability to function on multi-disciplinary teams.” The “ability to function on multidisciplinary teams” is inherently crit- ical for the working engineer. Engineers rarely, if ever, work by them- selves. Engineers will always be part of some team. One of the things I like to remind students is that you do not have to like everyone on a team; you just have to be a successful part of that team. Most frequently, these teams include individuals from of a variety of engineering disciplines, as well as individuals from nonengineering disciplines such as marketing and finance. Students can easily develop and enhance this skill by working in study groups in every course they take. Clearly, working in study groups in nonengineering courses as well as engineering courses out- side your discipline will also give you experience with multidiscipli- nary teams. Photo by Charles Alexander The last section in each chapter is devoted to appli- cations of the concepts covered in the chapter to help students apply the concepts to real-life situations. Icons next to the end-of-chapter homework problems let students know which problems relate to engineer- ing design and which problems can be solved using PSpice or MATLAB. Appendices on these computer programs provide tutorials for their use. Each chapter opens with a discussion about how to enhance skills that contribute to successful problem solving as well as successful careers or a career- oriented talk on a sub-discipline of electrical engi- neering to give students some real-world applications of what they are learning. ale29559_fm.qxd 07/28/2008 11:54 AM Page xxi
  • 26. xxii Guided Tour Supplements for Students and Instructors McGraw-Hill’s ARIS—Assessment, Review, and Instruction System is a complete, online tutorial, electronic homework, and course man- agement system, designed for greater ease of use than any other system available. With ARIS, instructors can create and share course materials and assignments with other instructors, edit questions and algorithms, import their own content, and create announcements and due dates for assignments. ARIS has automatic grading and reporting of easy-to-assign algorithmically-generated homework, quizzing, and testing. Once a student is registered in the course, all student activity within McGraw-Hill’s ARIS is automatically recorded and available to the instructor through a fully integrated grade book that can be downloaded to Excel. www.mhhe.com/alexander Knowledge Capturing Integrated Design Environment for Circuits (KCIDE for Circuits) software, linked from ARIS, enhances student understanding of the six-step problem-solving methodology in the book. KCIDE for Circuits allows students to work a circuit problem in PSpice and MATLAB, track the evolution of their solution, and save a record of their process for future reference. Appendix F walks the user through this program. ale29559_fm.qxd 07/28/2008 11:54 AM Page xxii
  • 27. Guided Tour xxiii Other resources provided on ARIS. For Students: — Network Analysis Tutorials—a series of interactive quizzes to help students practice fundamental concepts in circuits. — FE Exam Interactive Review Quizzes—chapter based self-quizzes provide hints for solutions and correct solution methods, and help students prepare for the NCEES Fundamentals of Engineering Examination. — Problem Solving Made Almost Easy—a companion workbook to the text, featuring 150 additional problems with complete solutions. — Complete solutions to Practice Problems in the text — Flashcards of key terms — Web links For Instructors: — Image Sets—electronic files of text figures for easy integration into your course presentations, exams, and assignments. — Transition Guides—compare coverage of the third edition to other popular circuits books at the section level to aid transition to teach- ing from our text. ale29559_fm.qxd 07/28/2008 11:54 AM Page xxiii
  • 29. A Note to the Student This may be your first course in electrical engineering. Although elec- trical engineering is an exciting and challenging discipline, the course may intimidate you. This book was written to prevent that. A good text- book and a good professor are an advantage—but you are the one who does the learning. If you keep the following ideas in mind, you will do very well in this course. • This course is the foundation on which most other courses in the electrical engineering curriculum rest. For this reason, put in as much effort as you can. Study the course regularly. • Problem solving is an essential part of the learning process. Solve as many problems as you can. Begin by solving the practice problem following each example, and then proceed to the end-of-chapter problems. The best way to learn is to solve a lot of problems. An asterisk in front of a problem indicates a challenging problem. • Spice, a computer circuit analysis program, is used throughout the textbook. PSpice, the personal computer version of Spice, is the popular standard circuit analysis program at most universities. PSpice for Windows is described in Appendix D. Make an effort to learn PSpice, because you can check any circuit problem with PSpice and be sure you are handing in a correct problem solution. • MATLAB is another software that is very useful in circuit analysis and other courses you will be taking. A brief tutorial on MATLAB is given in Appendix E to get you started. The best way to learn MATLAB is to start working with it once you know a few commands. • Each chapter ends with a section on how the material covered in the chapter can be applied to real-life situations. The concepts in this section may be new and advanced to you. No doubt, you will learn more of the details in other courses. We are mainly interested in gaining a general familiarity with these ideas. • Attempt the review questions at the end of each chapter. They will help you discover some “tricks” not revealed in class or in the textbook. • Clearly a lot of effort has gone into making the technical details in this book easy to understand. It also contains all the mathematics and physics necessary to understand the theory and will be very useful in your other engineering courses. However, we have also focused on creating a reference for you to use both in school as well as when working in industry or seeking a graduate degree. • It is very tempting to sell your book after you have completed your classroom experience; however, our advice to you is DO NOT SELL YOUR ENGINEERING BOOKS! Books have always been expen- sive, however, the cost of this book is virtually the same as I paid for my circuits text back in the early 60s in terms of real dollars. In fact, it is actually cheaper. In addition, engineering books of the past are no where near as complete as what is available now. xxv ale29559_fm.qxd 07/28/2008 11:54 AM Page xxv
  • 30. When I was a student, I did not sell any of my engineering text- books and was very glad I did not! I found that I needed most of them throughout my career. A short review on finding determinants is covered in Appendix A, complex numbers in Appendix B, and mathematical formulas in Appen- dix C. Answers to odd-numbered problems are given in Appendix G. Have fun! C. K. A. and M. N. O. S. xxvi A Note to the Student ale29559_fm.qxd 07/28/2008 11:54 AM Page xxvi
  • 31. xxvii About the Authors Charles K. Alexander is professor of electrical and computer engi- neering of the Fenn College of Engineering at Cleveland State Uni- versity, Cleveland, Ohio. He is also the Director of The Center for Research in Electronics and Aerospace Technology (CREATE), and is the Managing Director of the Wright Center for Sensor Systems (WCSSE). From 2002 until 2006 he was Dean of the Fenn College of Engineering. From 2004 until 2007, he was Director of Ohio ICE, a research center in instrumentation, controls, electronics, and sensors (a coalition of CSU, Case, the University of Akron, and a number of Ohio industries). From 1998 until 2002, he was interim director (2000 and 2001) of the Institute for Corrosion and Multiphase Technologies and Stocker Visiting Professor of electrical engineering and computer sci- ence at Ohio University. From 1994–1996 he was dean of engineering and computer science at California State University, Northridge. From 1989–1994 he was acting dean of the college of engineer- ing at Temple University, and from 1986–1989 he was professor and chairman of the department of electrical engineering at Temple. From 1980–1986 he held the same positions at Tennessee Technological University. He was an associate professor and a professor of electrical engineering at Youngstown State University from 1972–1980, where he was named Distinguished Professor in 1977 in recognition of “outstanding teaching and research.” He was assistant professor of electrical engineering at Ohio University in 1971–1972. He received the Ph.D. (1971) and M.S.E.E. (1967) from Ohio University and the B.S.E.E. (1965) from Ohio Northern University. Dr. Alexander has been a consultant to 23 companies and gov- ernmental organizations, including the Air Force and Navy and sev- eral law firms. He has received over $85 million in research and development funds for projects ranging from solar energy to software engineering. He has authored 40 publications, including a workbook and a videotape lecture series, and is coauthor of Fundamentals of Electric Circuits, Problem Solving Made Almost Easy, and the fifth edition of the Standard Handbook of Electronic Engineering, with McGraw-Hill. He has made more than 500 paper, professional, and technical presentations. Dr. Alexander is a fellow of the IEEE and served as its president and CEO in 1997. In 1993 and 1994 he was IEEE vice president, pro- fessional activities, and chair of the United States Activities Board (USAB). In 1991–1992 he was region 2 director, serving on the Regional Activities Board (RAB) and USAB. He has also been a mem- ber of the Educational Activities Board. He served as chair of the USAB Member Activities Council and vice chair of the USAB Professional Activities Council for Engineers, and he chaired the RAB Student Activities Committee and the USAB Student Professional Awareness Committee. Charles K. Alexander ale29559_fm.qxd 07/28/2008 11:54 AM Page xxvii
  • 32. In 1998 he received the Distinguished Engineering Education Achievement Award from the Engineering Council, and in 1996 he received the Distinguished Engineering Education Leadership Award from the same group. When he became a fellow of the IEEE in 1994, the citation read “for leadership in the field of engineering education and the professional development of engineering students.” In 1984 he received the IEEE Centennial Medal, and in 1983 he received the IEEE/RAB Innovation Award, given to the IEEE member who best contributes to RAB’s goals and objectives. Matthew N. O. Sadiku is presently a professor at Prairie View A&M University. Prior to joining Prairie View, he taught at Florida Atlantic University, Boca Raton, and Temple University, Philadelphia. He has also worked for Lucent/Avaya and Boeing Satellite Systems. Dr. Sadiku is the author of over 170 professional papers and almost 30 books including Elements of Electromagnetics (Oxford University Press, 3rd ed., 2001), Numerical Techniques in Electromagnetics (2nd ed., CRC Press, 2000), Simulation of Local Area Networks (with M. IIyas, CRC Press, 1994), Metropolitan Area Networks (CRC Press, 1994), and Fundamentals of Electric Circuits (with C. K. Alexander, McGraw-Hill, 3rd ed., 2007). His books are used worldwide, and some of them have been translated into Korean, Chinese, Italian, and Spanish. He was the recipient of the 2000 McGraw-Hill/Jacob Millman Award for outstanding contributions in the field of electrical engineering. He was the IEEE region 2 Student Activities Committee chairman and is an asso- ciate editor for IEEE “Transactions on Education.” He received his Ph.D. at Tennessee Technological University, Cookeville. xxviii About the Authors Matthew N. O. Sadiku ale29559_fm.qxd 07/28/2008 11:54 AM Page xxviii
  • 34. P A R T O N E DC Circuits OUTLINE 1 Basic Concepts 2 Basic Laws 3 Methods of Analysis 4 Circuit Theorems 5 Operational Amplifiers 6 Capacitors and Inductors 7 First-Order Circuits 8 Second-Order Circuits ale29559_ch01.qxd 07/08/2008 10:37 AM Page 2
  • 35. 3 Basic Concepts Some books are to be tasted, others to be swallowed, and some few to be chewed and digested. —Francis Bacon c h a p t e r 1 Enhancing Your Skills and Your Career ABET EC 2000 criteria (3.a), “an ability to apply knowledge of mathematics, science, and engineering.” As students, you are required to study mathematics, science, and engi- neering with the purpose of being able to apply that knowledge to the solution of engineering problems. The skill here is the ability to apply the fundamentals of these areas in the solution of a problem. So, how do you develop and enhance this skill? The best approach is to work as many problems as possible in all of your courses. However, if you are really going to be successful with this, you must spend time analyzing where and when and why you have difficulty in easily arriving at successful solutions. You may be sur- prised to learn that most of your problem-solving problems are with mathematics rather than your understanding of theory. You may also learn that you start working the problem too soon. Taking time to think about the problem and how you should solve it will always save you time and frustration in the end. What I have found that works best for me is to apply our six- step problem-solving technique. Then I carefully identify the areas where I have difficulty solving the problem. Many times, my actual deficiencies are in my understanding and ability to use correctly cer- tain mathematical principles. I then return to my fundamental math texts and carefully review the appropriate sections and in some cases work some example problems in that text. This brings me to another important thing you should always do: Keep nearby all your basic mathematics, science, and engineering textbooks. This process of continually looking up material you thought you had acquired in earlier courses may seem very tedious at first; how- ever, as your skills develop and your knowledge increases, this process will become easier and easier. On a personal note, it is this very process that led me from being a much less than average student to someone who could earn a Ph.D. and become a successful researcher. Photo by Charles Alexander ale29559_ch01.qxd 07/08/2008 10:37 AM Page 3
  • 36. Introduction Electric circuit theory and electromagnetic theory are the two funda- mental theories upon which all branches of electrical engineering are built. Many branches of electrical engineering, such as power, electric machines, control, electronics, communications, and instrumentation, are based on electric circuit theory. Therefore, the basic electric circuit theory course is the most important course for an electrical engineer- ing student, and always an excellent starting point for a beginning stu- dent in electrical engineering education. Circuit theory is also valuable to students specializing in other branches of the physical sciences because circuits are a good model for the study of energy systems in general, and because of the applied mathematics, physics, and topol- ogy involved. In electrical engineering, we are often interested in communicating or transferring energy from one point to another. To do this requires an interconnection of electrical devices. Such interconnection is referred to as an electric circuit, and each component of the circuit is known as an element. An electric circuit is an interconnection of electrical elements. A simple electric circuit is shown in Fig. 1.1. It consists of three basic elements: a battery, a lamp, and connecting wires. Such a simple circuit can exist by itself; it has several applications, such as a flash- light, a search light, and so forth. A complicated real circuit is displayed in Fig. 1.2, representing the schematic diagram for a radio receiver. Although it seems complicated, this circuit can be analyzed using the techniques we cover in this book. Our goal in this text is to learn various analytical techniques and computer software applications for describing the behavior of a circuit like this. Electric circuits are used in numerous electrical systems to accom- plish different tasks. Our objective in this book is not the study of various uses and applications of circuits. Rather our major concern is the analysis of the circuits. By the analysis of a circuit, we mean a study of the behavior of the circuit: How does it respond to a given input? How do the interconnected elements and devices in the circuit interact? We commence our study by defining some basic concepts. These concepts include charge, current, voltage, circuit elements, power, and energy. Before defining these concepts, we must first establish a sys- tem of units that we will use throughout the text. Systems of Units As electrical engineers, we deal with measurable quantities. Our mea- surement, however, must be communicated in a standard language that virtually all professionals can understand, irrespective of the country where the measurement is conducted. Such an international measurement 1.2 1.1 4 Chapter 1 Basic Concepts + − Current Lamp Battery Figure 1.1 A simple electric circuit. ale29559_ch01.qxd 07/08/2008 10:37 AM Page 4
  • 37. language is the International System of Units (SI), adopted by the General Conference on Weights and Measures in 1960. In this system, there are six principal units from which the units of all other physical quantities can be derived. Table 1.1 shows the six units, their symbols, and the physical quantities they represent. The SI units are used throughout this text. One great advantage of the SI unit is that it uses prefixes based on the power of 10 to relate larger and smaller units to the basic unit. Table 1.2 shows the SI prefixes and their symbols. For example, the following are expressions of the same distance in meters (m): 600,000,000 mm 600,000 m 600 km 1.2 Systems of Units 5 2, 5, 6 C Oscillator E B R2 10 k R3 10 k R1 47 Y1 7 MHz C6 5 L2 22.7 ␮H (see text) to U1, Pin 8 R10 10 k GAIN + + C16 100 ␮F 16 V C11 100 ␮F 16 V C10 1.0 ␮F 16 V C9 1.0 ␮F 16 V C15 0.47 16 V C17 100 ␮F 16 V + − 12-V dc Supply Audio Output + C18 0.1 R12 10 1 4 2 3 C14 0.0022 0.1 C13 U2A 1 2 TL072 U2B 1 2 TL072 R9 15 k R5 100 k R8 15 k R6 100 k 5 6 R7 1 M C12 0.0033 + L3 1 mH R11 47 C8 0.1 Q1 2N2222A 7 C3 0.1 L1 0.445 ␮H Antenna C1 2200 pF C2 2200 pF 1 8 7 U1 SBL-1 Mixer 3, 4 C7 532 C4 910 C5 910 R4 220 U3 LM386N Audio power amp 5 4 6 3 2 + + − + − + − + 8 Figure 1.2 Electric circuit of a radio receiver. Reproduced with permission from QST, August 1995, p. 23. TABLE 1.1 Six basic SI units and one derived unit relevant to this text. Quantity Basic unit Symbol Length meter m Mass kilogram kg Time second s Electric current ampere A Thermodynamic temperature kelvin K Luminous intensity candela cd Charge coulomb C TABLE 1.2 The SI prefixes. Multiplier Prefix Symbol exa E peta P tera T giga G mega M kilo k hecto h 10 deka da deci d centi c milli m micro nano n pico p femto f atto a 1018 1015 1012 109 m 106 103 102 101 102 103 106 109 1012 1015 1018 ale29559_ch01.qxd 07/17/2008 11:15 AM Page 5
  • 38. Charge and Current The concept of electric charge is the underlying principle for explain- ing all electrical phenomena. Also, the most basic quantity in an elec- tric circuit is the electric charge. We all experience the effect of electric charge when we try to remove our wool sweater and have it stick to our body or walk across a carpet and receive a shock. Charge is an electrical property of the atomic particles of which mat- ter consists, measured in coulombs (C). We know from elementary physics that all matter is made of funda- mental building blocks known as atoms and that each atom consists of electrons, protons, and neutrons. We also know that the charge e on an electron is negative and equal in magnitude to C, while a proton carries a positive charge of the same magnitude as the elec- tron. The presence of equal numbers of protons and electrons leaves an atom neutrally charged. The following points should be noted about electric charge: 1. The coulomb is a large unit for charges. In 1 C of charge, there are electrons. Thus realistic or laboratory values of charges are on the order of pC, nC, or C.1 2. According to experimental observations, the only charges that occur in nature are integral multiples of the electronic charge 3. The law of conservation of charge states that charge can neither be created nor destroyed, only transferred. Thus the algebraic sum of the electric charges in a system does not change. We now consider the flow of electric charges. A unique feature of electric charge or electricity is the fact that it is mobile; that is, it can be transferred from one place to another, where it can be converted to another form of energy. When a conducting wire (consisting of several atoms) is con- nected to a battery (a source of electromotive force), the charges are compelled to move; positive charges move in one direction while neg- ative charges move in the opposite direction. This motion of charges creates electric current. It is conventional to take the current flow as the movement of positive charges. That is, opposite to the flow of neg- ative charges, as Fig. 1.3 illustrates. This convention was introduced by Benjamin Franklin (1706–1790), the American scientist and inven- tor. Although we now know that current in metallic conductors is due to negatively charged electrons, we will follow the universally accepted convention that current is the net flow of positive charges. Thus, Electric current is the time rate of change of charge, measured in amperes (A). e 1.602 1019 C. m 1(1.602 1019 ) 6.24 1018 1.602 1019 1.3 6 Chapter 1 Basic Concepts 1 However, a large power supply capacitor can store up to 0.5 C of charge. Battery I − − − − + − Figure 1.3 Electric current due to flow of electronic charge in a conductor. A convention is a standard way of describing something so that others in the profession can understand what we mean. We will be using IEEE con- ventions throughout this book. ale29559_ch01.qxd 07/08/2008 10:38 AM Page 6
  • 39. Mathematically, the relationship between current i, charge q, and time t is (1.1) where current is measured in amperes (A), and The charge transferred between time and t is obtained by integrat- ing both sides of Eq. (1.1). We obtain (1.2) The way we define current as i in Eq. (1.1) suggests that current need not be a constant-valued function. As many of the examples and prob- lems in this chapter and subsequent chapters suggest, there can be sev- eral types of current; that is, charge can vary with time in several ways. If the current does not change with time, but remains constant, we call it a direct current (dc). A direct current (dc) is a current that remains constant with time. By convention the symbol I is used to represent such a constant current. A time-varying current is represented by the symbol i. A common form of time-varying current is the sinusoidal current or alternating current (ac). An alternating current (ac) is a current that varies sinusoidally with time. Such current is used in your household, to run the air conditioner, refrigerator, washing machine, and other electric appliances. Figure 1.4 Q ¢ t t0 i dt t0 1 ampere 1 coulomb/second i ¢ dq dt 1.3 Charge and Current 7 Andre-Marie Ampere (1775–1836), a French mathematician and physicist, laid the foundation of electrodynamics. He defined the elec- tric current and developed a way to measure it in the 1820s. Born in Lyons, France, Ampere at age 12 mastered Latin in a few weeks, as he was intensely interested in mathematics and many of the best mathematical works were in Latin. He was a brilliant scientist and a prolific writer. He formulated the laws of electromagnetics. He in- vented the electromagnet and the ammeter. The unit of electric current, the ampere, was named after him. Historical I 0 t (a) (b) i t 0 Figure 1.4 Two common types of current: (a) direct current (dc), (b) alternating current (ac). The Burndy Library Collection at The Huntington Library, San Marino, California. ale29559_ch01.qxd 07/16/2008 12:22 PM Page 7
  • 40. shows direct current and alternating current; these are the two most common types of current. We will consider other types later in the book. Once we define current as the movement of charge, we expect cur- rent to have an associated direction of flow. As mentioned earlier, the direction of current flow is conventionally taken as the direction of pos- itive charge movement. Based on this convention, a current of 5 A may be represented positively or negatively as shown in Fig. 1.5. In other words, a negative current of A flowing in one direction as shown in Fig. 1.5(b) is the same as a current of A flowing in the opposite direction. 5 5 8 Chapter 1 Basic Concepts 5 A (a) −5 A (b) Figure 1.5 Conventional current flow: (a) positive current flow, (b) negative current flow. How much charge is represented by 4,600 electrons? Solution: Each electron has C. Hence 4,600 electrons will have 1.602 1019 C/electron 4,600 electrons 7.369 1016 C 1.602 1019 Example 1.1 Practice Problem 1.2 Example 1.2 Calculate the amount of charge represented by four million protons. Answer: C. 6.408 1013 Practice Problem 1.1 If in Example 1.2, find the current at s. Answer: 7.36 mA. t 0.5 q (10 10e2t ) mC, The total charge entering a terminal is given by mC. Calculate the current at s. Solution: At i 5 sin 2p 10p cos 2p 0 10p 31.42 mA t 0.5, i dq dt d dt (5t sin 4pt) mC/s (5 sin 4pt 20pt cos 4pt) mA t 0.5 q 5t sin 4pt ale29559_ch01.qxd 07/08/2008 10:38 AM Page 8
  • 41. Voltage As explained briefly in the previous section, to move the electron in a conductor in a particular direction requires some work or energy trans- fer. This work is performed by an external electromotive force (emf), typically represented by the battery in Fig. 1.3. This emf is also known as voltage or potential difference. The voltage between two points a and b in an electric circuit is the energy (or work) needed to move a unit charge from a to b; mathematically, (1.3) where w is energy in joules (J) and q is charge in coulombs (C). The voltage or simply v is measured in volts (V), named in honor of the Italian physicist Alessandro Antonio Volta (1745–1827), who invented the first voltaic battery. From Eq. (1.3), it is evident that Thus, Voltage (or potential difference) is the energy required to move a unit charge through an element, measured in volts (V). Figure 1.6 shows the voltage across an element (represented by a rectangular block) connected to points a and b. The plus and minus signs are used to define reference direction or voltage polarity. The can be interpreted in two ways: (1) point a is at a potential of vab vab () () 1 volt 1 joule/coulomb 1 newton-meter/coulomb vab vab ¢ dw dq vab 1.4 1.4 Voltage 9 Example 1.3 Determine the total charge entering a terminal between and s if the current passing the terminal is A. Solution: at3 t2 2 b ` 2 1 (8 2) a1 1 2 b 5.5 C Q 2 t1 i dt 2 1 (3t2 t)dt i (3t2 t) t 2 t 1 s The current flowing through an element is Calculate the charge entering the element from to s. Answer: 6.667 C. t 2 t 0 i e 2 A, 0 6 t 6 1 2t2 A, t 7 1 a b vab + − Figure 1.6 Polarity of voltage . vab Practice Problem 1.3 ale29559_ch01.qxd 07/08/2008 10:38 AM Page 9
  • 42. Historical Alessandro Antonio Volta (1745–1827), an Italian physicist, invented the electric battery—which provided the first continuous flow of electricity—and the capacitor. Born into a noble family in Como, Italy, Volta was performing electrical experiments at age 18. His invention of the battery in 1796 revolutionized the use of electricity. The publication of his work in 1800 marked the beginning of electric circuit theory. Volta received many honors during his lifetime. The unit of voltage or potential dif- ference, the volt, was named in his honor. volts higher than point b, or (2) the potential at point a with respect to point b is . It follows logically that in general (1.4) For example, in Fig. 1.7, we have two representations of the same volt- age. In Fig. 1.7(a), point a is V above point b; in Fig. 1.7(b), point b is V above point a. We may say that in Fig. 1.7(a), there is a 9-V voltage drop from a to b or equivalently a 9-V voltage rise from b to a. In other words, a voltage drop from a to b is equivalent to a volt- age rise from b to a. Current and voltage are the two basic variables in electric circuits. The common term signal is used for an electric quantity such as a cur- rent or a voltage (or even electromagnetic wave) when it is used for conveying information. Engineers prefer to call such variables signals rather than mathematical functions of time because of their importance in communications and other disciplines. Like electric current, a con- stant voltage is called a dc voltage and is represented by V, whereas a sinusoidally time-varying voltage is called an ac voltage and is repre- sented by v. A dc voltage is commonly produced by a battery; ac volt- age is produced by an electric generator. Power and Energy Although current and voltage are the two basic variables in an electric circuit, they are not sufficient by themselves. For practical purposes, we need to know how much power an electric device can handle. We all know from experience that a 100-watt bulb gives more light than a 60-watt bulb. We also know that when we pay our bills to the electric utility companies, we are paying for the electric energy consumed over a certain period of time. Thus, power and energy calculations are important in circuit analysis. 1.5 9 9 vab vba vab 10 Chapter 1 Basic Concepts 9 V (a) a b + − −9 V (b) a b + − Figure 1.7 Two equivalent representations of the same voltage : (a) point a is 9 V above point b, (b) point b is 9 V above point a. vab Keep in mind that electric current is always through an element and that electric voltage is always across the element or between two points. The Burndy Library Collection at The Huntington Library, San Marino, California. ale29559_ch01.qxd 07/16/2008 12:23 PM Page 10
  • 43. 1.5 Power and Energy 11 To relate power and energy to voltage and current, we recall from physics that: Power is the time rate of expending or absorbing energy, measured in watts (W). We write this relationship as (1.5) where p is power in watts (W), w is energy in joules (J), and t is time in seconds (s). From Eqs. (1.1), (1.3), and (1.5), it follows that (1.6) or (1.7) The power p in Eq. (1.7) is a time-varying quantity and is called the instantaneous power. Thus, the power absorbed or supplied by an ele- ment is the product of the voltage across the element and the current through it. If the power has a sign, power is being delivered to or absorbed by the element. If, on the other hand, the power has a sign, power is being supplied by the element. But how do we know when the power has a negative or a positive sign? Current direction and voltage polarity play a major role in deter- mining the sign of power. It is therefore important that we pay atten- tion to the relationship between current i and voltage v in Fig. 1.8(a). The voltage polarity and current direction must conform with those shown in Fig. 1.8(a) in order for the power to have a positive sign. This is known as the passive sign convention. By the passive sign con- vention, current enters through the positive polarity of the voltage. In this case, or implies that the element is absorbing power. However, if or , as in Fig. 1.8(b), the element is releasing or supplying power. Passive sign convention is satisfied when the current enters through the positive terminal of an element and p vi. If the current enters through the negative terminal, p vi. Unless otherwise stated, we will follow the passive sign conven- tion throughout this text. For example, the element in both circuits of Fig. 1.9 has an absorbing power of W because a positive current enters the positive terminal in both cases. In Fig. 1.10, however, the element is supplying power of W because a positive current enters the negative terminal. Of course, an absorbing power of W is equivalent to a supplying power of W. In general, Power absorbed Power supplied 12 12 12 12 vi 6 0 p vi vi 7 0 p vi p vi p dw dt dw dq dq dt vi p ¢ dw dt p = +vi (a) v + − p = −vi (b) v + − i i Figure 1.8 Reference polarities for power using the passive sign convention: (a) absorbing power, (b) supplying power. When the voltage and current directions conform to Fig. 1.8(b), we have the ac- tive sign convention and p vi. (a) 4 V 3 A (a) + − 3 A 4 V 3 A (b) + − Figure 1.9 Two cases of an element with an absorbing power of 12 W: (a) W, (b) W. p 4 3 12 p 4 3 12 3 A (a) 4 V 3 A (a) + − 3 A 4 V 3 A (b) + − Figure 1.10 Two cases of an element with a supplying power of 12 W: (a) W, (b) W. p 4 3 12 12 p 4 3 ale29559_ch01.qxd 07/08/2008 10:38 AM Page 11
  • 44. In fact, the law of conservation of energy must be obeyed in any electric circuit. For this reason, the algebraic sum of power in a cir- cuit, at any instant of time, must be zero: (1.8) This again confirms the fact that the total power supplied to the circuit must balance the total power absorbed. From Eq. (1.6), the energy absorbed or supplied by an element from time to time t is (1.9) Energy is the capacity to do work, measured in joules (J). The electric power utility companies measure energy in watt-hours (Wh), where 1 Wh 3,600 J w t t0 p dt t t0 vi dt t0 a p 0 12 Chapter 1 Basic Concepts An energy source forces a constant current of 2 A for 10 s to flow through a lightbulb. If 2.3 kJ is given off in the form of light and heat energy, calculate the voltage drop across the bulb. Solution: The total charge is The voltage drop is v ¢w ¢q 2.3 103 20 115 V ¢q i ¢t 2 10 20 C Example 1.4 To move charge q from point a to point b requires J. Find the voltage drop if: (a) C, (b) C. Answer: (a) V, (b) 5 V. 15 q 6 q 2 vab 30 Practice Problem 1.4 Find the power delivered to an element at ms if the current enter- ing its positive terminal is and the voltage is: (a) , (b) . v 3 didt v 3i i 5 cos 60pt A t 3 Example 1.5 ale29559_ch01.qxd 07/08/2008 10:38 AM Page 12
  • 45. 1.5 Power and Energy 13 Solution: (a) The voltage is hence, the power is At ms, (b) We find the voltage and the power as At ms, 14137.167 sin 32.4 cos 32.4 6.396 kW p 4500p sin 0.18p cos 0.18p W t 3 p vi 4500p sin 60pt cos 60pt W v 3 di dt 3(60p)5 sin 60pt 900p sin 60pt V p 75 cos2 (60p 3 103 ) 75 cos2 0.18p 53.48 W t 3 p vi 75 cos2 60pt W v 3i 15 cos 60pt; Practice Problem 1.5 Find the power delivered to the element in Example 1.5 at ms if the current remains the same but the voltage is: (a) V, (b) V. Answer: (a) 17.27 W, (b) 29.7 W. v a10 5 t 0 i dtb v 2i t 5 Example 1.6 How much energy does a 100-W electric bulb consume in two hours? Solution: This is the same as w pt 100 W 2 h 200 Wh 720,000 J 720 kJ w pt 100 (W) 2 (h) 60 (min/h) 60 (s/min) Practice Problem 1.6 A stove element draws 15 A when connected to a 240-V line. How long does it take to consume 60 kJ? Answer: 16.667 s. ale29559_ch01.qxd 07/08/2008 10:38 AM Page 13
  • 46. 14 Chapter 1 Basic Concepts Smithsonian Institution. Historical 1884 Exhibition In the United States, nothing promoted the future of electricity like the 1884 International Electrical Exhibition. Just imagine a world without electricity, a world illuminated by candles and gaslights, a world where the most common transportation was by walk- ing and riding on horseback or by horse-drawn carriage. Into this world an exhibition was created that highlighted Thomas Edison and reflected his highly developed ability to promote his inventions and products. His exhibit featured spectacular lighting displays powered by an impres- sive 100-kW “Jumbo” generator. Edward Weston’s dynamos and lamps were featured in the United States Electric Lighting Company’s display. Weston’s well known col- lection of scientific instruments was also shown. Other prominent exhibitors included Frank Sprague, Elihu Thompson, and the Brush Electric Company of Cleveland. The American Institute of Electrical Engineers (AIEE) held its first technical meeting on Octo- ber 7–8 at the Franklin Institute during the exhibit. AIEE merged with the Institute of Radio Engineers (IRE) in 1964 to form the Institute of Electrical and Electronics Engineers (IEEE). ale29559_ch01.qxd 07/08/2008 10:38 AM Page 14
  • 47. Circuit Elements As we discussed in Section 1.1, an element is the basic building block of a circuit. An electric circuit is simply an interconnection of the ele- ments. Circuit analysis is the process of determining voltages across (or the currents through) the elements of the circuit. There are two types of elements found in electric circuits: pas- sive elements and active elements. An active element is capable of generating energy while a passive element is not. Examples of pas- sive elements are resistors, capacitors, and inductors. Typical active elements include generators, batteries, and operational amplifiers. Our aim in this section is to gain familiarity with some important active elements. The most important active elements are voltage or current sources that generally deliver power to the circuit connected to them. There are two kinds of sources: independent and dependent sources. An ideal independent source is an active element that provides a specified voltage or current that is completely independent of other circuit elements. In other words, an ideal independent voltage source delivers to the circuit whatever current is necessary to maintain its terminal volt- age. Physical sources such as batteries and generators may be regarded as approximations to ideal voltage sources. Figure 1.11 shows the symbols for independent voltage sources. Notice that both symbols in Fig. 1.11(a) and (b) can be used to represent a dc volt- age source, but only the symbol in Fig. 1.11(a) can be used for a time-varying voltage source. Similarly, an ideal independent current source is an active element that provides a specified current com- pletely independent of the voltage across the source. That is, the cur- rent source delivers to the circuit whatever voltage is necessary to maintain the designated current. The symbol for an independent cur- rent source is displayed in Fig. 1.12, where the arrow indicates the direction of current i. An ideal dependent (or controlled) source is an active element in which the source quantity is controlled by another voltage or current. Dependent sources are usually designated by diamond-shaped symbols, as shown in Fig. 1.13. Since the control of the dependent source is achieved by a voltage or current of some other element in the circuit, and the source can be voltage or current, it follows that there are four possible types of dependent sources, namely: 1. A voltage-controlled voltage source (VCVS). 2. A current-controlled voltage source (CCVS). 3. A voltage-controlled current source (VCCS). 4. A current-controlled current source (CCCS). 1.6 1.6 Circuit Elements 15 Figure 1.11 Symbols for independent voltage sources: (a) used for constant or time-varying volt- age, (b) used for constant voltage (dc). V (b) + − v (a) + − i Figure 1.12 Symbol for independent current source. (a) (b) v + − i Figure 1.13 Symbols for: (a) dependent voltage source, (b) dependent current source. ale29559_ch01.qxd 07/08/2008 10:38 AM Page 15
  • 48. Dependent sources are useful in modeling elements such as transis- tors, operational amplifiers, and integrated circuits. An example of a current-controlled voltage source is shown on the right-hand side of Fig. 1.14, where the voltage of the voltage source depends on the current i through element C. Students might be surprised that the value of the dependent voltage source is V (and not A) because it is a voltage source. The key idea to keep in mind is that a voltage source comes with polarities in its symbol, while a current source comes with an arrow, irrespective of what it depends on. It should be noted that an ideal voltage source (dependent or inde- pendent) will produce any current required to ensure that the terminal voltage is as stated, whereas an ideal current source will produce the necessary voltage to ensure the stated current flow. Thus, an ideal source could in theory supply an infinite amount of energy. It should also be noted that not only do sources supply power to a circuit, they can absorb power from a circuit too. For a voltage source, we know the voltage but not the current supplied or drawn by it. By the same token, we know the current supplied by a current source but not the voltage across it. ( ) 10i 10i 10i 16 Chapter 1 Basic Concepts Calculate the power supplied or absorbed by each element in Fig. 1.15. Solution: We apply the sign convention for power shown in Figs. 1.8 and 1.9. For , the 5-A current is out of the positive terminal (or into the negative terminal); hence, For and , the current flows into the positive terminal of the ele- ment in each case. For , we should note that the voltage is 8 V (positive at the top), the same as the voltage for since both the passive element and the dependent source are connected to the same terminals. (Remember that voltage is always measured across an element in a circuit.) Since the current flows out of the positive terminal, We should observe that the 20-V independent voltage source and dependent current source are supplying power to the rest of the network, while the two passive elements are absorbing power. Also, In agreement with Eq. (1.8), the total power supplied equals the total power absorbed. p1 p2 p3 p4 100 60 48 8 0 0.2I p4 8(0.2I) 8(0.2 5) 8 W Supplied power p3, p4 p3 8(6) 48 W Absorbed power p2 12(5) 60 W Absorbed power p3 p2 p1 20(5) 100 W Supplied power p1 Example 1.7 p2 p3 I = 5 A 20 V 6 A 8 V 0.2I 12 V + − + − + − p1 p4 Figure 1.15 For Example 1.7. i A B C 10i 5 V + − + − Figure 1.14 The source on the right-hand side is a current-controlled voltage source. ale29559_ch01.qxd 07/08/2008 10:38 AM Page 16
  • 49. Applications2 In this section, we will consider two practical applications of the concepts developed in this chapter. The first one deals with the TV picture tube and the other with how electric utilities determine your electric bill. 1.7.1 TV Picture Tube One important application of the motion of electrons is found in both the transmission and reception of TV signals. At the transmission end, a TV camera reduces a scene from an optical image to an electrical signal. Scanning is accomplished with a thin beam of electrons in an iconoscope camera tube. At the receiving end, the image is reconstructed by using a cathode- ray tube (CRT) located in the TV receiver.3 The CRT is depicted in Fig. 1.17. Unlike the iconoscope tube, which produces an electron beam of constant intensity, the CRT beam varies in intensity accord- ing to the incoming signal. The electron gun, maintained at a high potential, fires the electron beam. The beam passes through two sets of plates for vertical and horizontal deflections so that the spot on the screen where the beam strikes can move right and left and up and down. When the electron beam strikes the fluorescent screen, it gives off light at that spot. Thus, the beam can be made to “paint” a picture on the TV screen. 1.7 1.7 Applications 17 Compute the power absorbed or supplied by each component of the circuit in Fig. 1.16. Answer: W, W, W, W. p4 15 p3 9 p2 16 p1 40 8 A 5 V 3 V 2 V 3 A I = 5 A 0.6I + − + − + − + − + − p2 p1 p3 p4 Figure 1.16 For Practice Prob. 1.7. 2 The dagger sign preceding a section heading indicates the section that may be skipped, explained briefly, or assigned as homework. 3 Modern TV tubes use a different technology. Vertical deflection plates Horizontal deflection plates Electron trajectory Bright spot on fluorescent screen Electron gun Figure 1.17 Cathode-ray tube. D. E. Tilley, Contemporary College Physics Menlo Park, CA: Benjamin/ Cummings, 1979, p. 319. Practice Problem 1.7 ale29559_ch01.qxd 07/08/2008 10:38 AM Page 17